med surg exam 4

¡Supera tus tareas y exámenes ahora con Quizwiz!

The nurse is administering eye drops to a patient with glaucoma. After instilling the patient's first medication, how long should the nurse wait before instilling the patient's second medication into the same eye? A) 30 seconds B) 1 minute C) 3 minutes D) 5 minutes

D Feedback: A 5-minute interval between successive eye drop administrations allows for adequate drug retention and absorption. Any time frame less than 5 minutes will not allow adequate absorption.

The nurse is assessing a patient with a suspected stroke. What assessment finding is most suggestive of a stroke? A) Facial droop B) Dysrhythmias C) Periorbital edema D) Projectile vomiting

A Feedback: Facial drooping or asymmetry is a classic abnormal finding on a physical assessment that may be associated with a stroke. Facial edema is not suggestive of a stroke and patients less commonly experience dysrhythmias or vomiting.

The nurse is admitting a 55-year-old male patient diagnosed with a retinal detachment in his left eye. While assessing this patient, what characteristic symptom would the nurse expect to find? A) Flashing lights in the visual field B) Sudden eye pain C) Loss of color vision D) Colored halos around lights

A Feedback: Flashing lights in the visual field is a common symptom of retinal detachment. Patients may also report spots or floaters or the sensation of a curtain being pulled across the eye. Retinal detachment is not associated with eye pain, loss of color vision, or colored halos around lights.

A patient's fracture is healing and callus is being deposited in the bone matrix. This process characterizes what phase of the bone healing process? A) The reparative phase B) The reactive phase C) The remodeling phase D) The revascularization phase

Ans: A Feedback: Callus formation takes place during the reparative phase of bone healing. The reactive phase occurs immediately after injury and the remodeling phase builds on the reparative phase. There is no discrete revascularization phase.

The nurse is providing care for a patient who has benefited from a cochlear implant. The nurse should understand that this patient's health history likely includes which of the following? Select all that apply. A) The patient was diagnosed with sensorineural hearing loss. B) The patient's hearing did not improve appreciably with the use of hearing aids. C) The patient has deficits in peripheral nervous function. D) The patient's hearing deficit is likely accompanied by a cognitive deficit. E) The patient is unable to lip-read.

A, B Feedback: A cochlear implant is an auditory prosthesis used for people with profound sensorineural hearing loss bilaterally who do not benefit from conventional hearing aids. The need for a cochlear implant is not associated with deficits in peripheral nervous function, cognitive deficits, or an inability to lip-read.

A patient is scheduled for enucleation and the nurse is providing anticipatory guidance about postoperative care. What aspects of care should the nurse describe to the patient? Select all that apply. A) Application of topical antibiotic ointment B) Maintenance of a supine position for the first 48 hours postoperative C) Fluid restriction to prevent orbital edema D) Administration of loop diuretics to prevent orbital edema E) Use of an ocular pressure dressing

A, E Feedback: Patients who undergo eye removal need to know that they will usually have a large ocular pressure dressing, which is typically removed after a week, and that an ophthalmic topical antibiotic ointment is applied in the socket three times daily. Fluid restriction, supine positioning, and diuretics are not indicated.

A nurse prepares to teach a client who has experienced damage to the left temporal lobe of the brain. Which action should the nurse take when providing education about newly prescribed medications to this client? a. Help the client identify each medication by its color. b. Provide written materials with large print size. c. Sit on the client's right side and speak into the right ear. d. Allow the client to use a white board to ask questions.

ANS: C The temporal lobe contains the auditory center for sound interpretation. The client's hearing will be impaired in the left ear. The nurse should sit on the client's right side and speak into the right ear. The other interventions do not address the client's left temporal lobe damage.

A nursing educator is reviewing the risk factors for osteoporosis with a group of recent graduates. What risk factor of the following should the educator describe? A) Recurrent infections and prolonged use of NSAIDs B) High alcohol intake and low body mass index C) Small frame, female gender, and Caucasian ethnicity D) Male gender, diabetes, and high protein intake

Ans: C Feedback: Small-framed, nonobese Caucasian women are at greatest risk for osteoporosis. Diabetes, high protein intake, alcohol use, and infections are not among the most salient risk factors for osteoporosis.

A patient who suffered a spinal cord injury is experiencing an exaggerated autonomic response. What aspect of the patient's current health status is most likely to have precipitated this event? A) The patient received a blood transfusion. B) The patient's analgesia regimen was recent changed. C) The patient was not repositioned during the night shift. D) The patient's urinary catheter became occluded.

Ans: D Feedback: A distended bladder is the most common cause of autonomic dysreflexia. Infrequent positioning is a less likely cause, although pressure ulcers or tactile stimulation can cause it. Changes in mediations or blood transfusions are unlikely causes.

While assessing a patient, the patient tells the nurse that she is experiencing rhythmic muscle contractions when the nurse performs passive extension of her wrist. What is this pattern of muscle contraction referred to as? A) Fasciculations B) Contractures C) Effusion D) Clonus

Ans: D Feedback: Clonus may occur when the ankle is dorsiflexed or the wrist is extended. It is characterized as rhythmic contractions of the muscle. Fasciculation is involuntary twitching of muscle fiber groups. Contractures are prolonged tightening of muscle groups and an effusion is the pathologic escape of body fluid.

A nurse is caring for a patient diagnosed with Ménière's disease. While completing a neurologic examination on the patient, the nurse assesses cranial nerve VIII. The nurse would be correct in identifying the function of this nerve as what? A) Movement of the tongue B) Visual acuity C) Sense of smell D) Hearing and equilibrium

Ans: D Feedback: Cranial nerve VIII (acoustic) is responsible for hearing and equilibrium. Cranial nerve XII (hypoglossal) is responsible for movement of the tongue. Cranial nerve II (optic) is responsible for visual acuity and visual fields. Cranial nerve I (olfactory) functions in sense of smell.

The nurse is conducting a focused neurologic assessment. When assessing the patient's cranial nerve function, the nurse would include which of the following assessments? A) Assessment of hand grip B) Assessment of orientation to person, time, and place C) Assessment of arm drift D) Assessment of gag reflex

Ans: D Feedback: The gag reflex is governed by the glossopharyngeal nerve, one of the cranial nerves. Hand grip and arm drifting are part of motor function assessment. Orientation is an assessment parameter related to a mental status examination.

A nurse admits a patient who has a fracture of the nose that has resulted in a skin tear and involvement of the mucous membranes of the nasal passages. The orthopedic nurse is aware that this description likely indicates which type of fracture? A) Compression B) Compound C) Impacted D) Transverse

B Feedback: A compound fracture involves damage to the skin or mucous membranes and is also called an open fracture. A compression fracture involves compression of bone and is seen in vertebral fractures. An impacted fracture occurs when a bone fragment is driven into another bone fragment. A transverse fracture occurs straight across the bone shaft.

Assessment is crucial to the care of patients with neurologic dysfunction. What does accurate and appropriate assessment require? Select all that apply. A) The ability to select mediations for the neurologic dysfunction B) Understanding of the tests used to diagnose neurologic disorders C) Knowledge of nursing interventions related to assessment and diagnostic testing D) Knowledge of the anatomy of the nervous system E) The ability to interpret the results of diagnostic tests

B, C, D Feedback: Assessment requires knowledge of the anatomy and physiology of the nervous system and an understanding of the array of tests and procedures used to diagnose neurologic disorders. Knowledge about the nursing implications and interventions related to assessment and diagnostic testing is also essential. Selecting medications and interpreting diagnostic tests are beyond the normal scope of the nurse.

After administering a medication that stimulates the sympathetic division of the autonomic nervous system, the nurse assesses the client. For which clinical manifestations should the nurse assess? (Select all that apply.) a. Decreased respiratory rate b. Increased heart rate c. Decreased level of consciousness d. Increased force of contraction e. Decreased blood pressure

B, D Stimulation of the sympathetic nervous system initiates the fight-or-flight response, increasing both the heart rate and the force of contraction. A medication that stimulates the sympathetic nervous system would also increase the client's respiratory rate, blood pressure, and level of consciousness.

A nurse assesses a client with early-onset multiple sclerosis (MS). Which clinical manifestation should the nurse expect to find? a. Hyperresponsive reflexes b. Excessive somnolence c. Nystagmus d. Heat intolerance

C Early signs and symptoms of MS include changes in motor skills, vision, and sensation. Hyperresponsive reflexes, excessive somnolence, and heat intolerance are later manifestations of MS.

A nurse is providing an educational class to a group of older adults at a community senior center. In an effort to prevent osteoporosis, the nurse should encourage participants to ensure that they consume the recommended adequate intake of what nutrients? Select all that apply. A) Vitamin B12 B) Potassium C) Calcitonin D) Calcium E) Vitamin D

D, E Feedback: A diet rich in calcium and vitamin D protects against skeletal demineralization. Intake of vitamin B12and potassium does not directly influence the risk for osteoporosis. Calcitonin is not considered to be a dietary nutrient.

A nurse plans care for a client with Parkinson disease. Which intervention should the nurse include in this client's plan of care? a. Ambulate the client in the hallway twice a day. b. Ensure a fluid intake of at least 3 liters per day. c. Teach the client pursed-lip breathing techniques. d. Keep the head of the bed at 30 degrees or greater.

Elevation of the head of the bed will help prevent aspiration. The other options will not prevent aspiration, which is the greatest respiratory complication of Parkinson disease, nor do these interventions address any of the complications of Parkinson disease. Ambulation in the hallway is usually implemented to prevent venous thrombosis. Increased fluid intake flushes out toxins from the client's blood. Pursed-lip breathing increases exhalation of carbon dioxide.

A nurse witnesses a client with late-stage Alzheimer's disease eat breakfast. Afterward the client states, "I am hungry and want breakfast." How should the nurse respond? a. "I see you are still hungry. I will get you some toast." b. "You ate your breakfast 30 minutes ago." c. "It appears you are confused this morning." d. "Your family will be here soon. Let's get you dressed."

A Use of validation therapy with clients who have Alzheimer's disease involves acknowledgment of the client's feelings and concerns. This technique has proved more effective in later stages of the disease, when using reality orientation only increases agitation. Telling the client that he or she already ate breakfast may agitate the client. The other statements do not validate the client's concerns.

A nurse cares for a client with advanced Alzheimer's disease. The client's caregiver states, "She is always wandering off. What can I do to manage this restless behavior?" How should the nurse respond? a. "This is a sign of fatigue. The client would benefit from a daily nap." b. "Engage the client in scheduled activities throughout the day." c. "It sounds like this is difficult for you. I will consult the social worker." d. "The provider can prescribe a mild sedative for restlessness."

B Several strategies may be used to cope with restlessness and wandering. One strategy is to engage the client in structured activities. Another is to take the client for frequent walks. Daily naps and a mild sedative will not be as effective in the management of restless behavior. Consulting the social worker does not address the caregiver's concern.

A patient is recovering from intracranial surgery that was performed using the transsphenoidal approach. The nurse should be aware that the patient may have required surgery on what neurologic structure? A) Cerebellum B) Hypothalamus C) Pituitary gland D) Pineal gland

C Feedback: The transsphenoidal approach (through the mouth and nasal sinuses) is often used to gain access to the pituitary gland. This surgical approach would not allow for access to the pineal gland, cerebellum, or hypothalamus.

A nurse assesses a client with a spinal cord injury at level T5. The client's blood pressure is 184/95 mm Hg, and the client presents with a flushed face and blurred vision. Which action should the nurse take first? a. Initiate oxygen via a nasal cannula. b. Place the client in a supine position. c. Palpate the bladder for distention. d. Administer a prescribed beta blocker.

C The client is manifesting symptoms of autonomic dysreflexia. Common causes include bladder distention, tight clothing, increased room temperature, and fecal impaction. If persistent, the client could experience neurologic injury. Precipitating conditions should be eliminated and the physician notified. The other actions would not be appropriate.

A client has a platelet count of 25,000/mm3. What actions does the nurse delegate to the unlicensed assistive personnel (UAP)? (Select all that apply.) a. Assist with oral hygiene using a firm toothbrush. b. Give the client an enema if he or she is constipated. c. Help the client choose soft foods from the menu. d. Shave the male client with an electric razor. e. Use a lift sheet when needed to re-position the client.

C, D, E This client has thrombocytopenia and requires bleeding precautions. These include oral hygiene with a soft-bristled toothbrush or swabs, avoiding rectal trauma, eating soft foods, shaving with an electric razor, and using a lift sheet to re-position the client.

An older adult with a recent history of mixed hearing loss has been diagnosed with a cholesteatoma. What should this patient be taught about this diagnosis? Select all that apply A) Cholesteatomas are benign and self-limiting, and hearing loss will resolve spontaneously. B) Cholesteatomas are usually the result of metastasis from a distant tumor site. C) Cholesteatomas are often the result of chronic otitis media. D) Cholesteatomas, if left untreated, result in intractable neuropathic pain. E) Cholesteatomas usually must be removed surgically.

C, E Feedback: Cholesteatoma is a tumor of the external layer of the eardrum into the middle ear, often resulting from chronic otitis media. They usually do not cause pain; however, if treatment or surgery is delayed, they may burst or destroy the mastoid bone. They are not normally the result of metastasis and are not self-limiting.

A nurse is preparing to hang a blood transfusion. Which action is most important? a. Documenting the transfusion b. Placing the client on NPO status c. Placing the client in isolation d. Putting on a pair of gloves

D To prevent bloodborne illness, the nurse should don a pair of gloves prior to hanging the blood. Documentation is important but not the priority at this point. NPO status and isolation are not needed.

A nurse prepares to provide perineal care to a client with meningococcal meningitis. Which personal protective equipment should the nurse wear? (Select all that apply.) a. Particulate respirator b. Isolation gown c. Shoe covers d. Surgical mask e. Gloves

D, E Meningeal meningitis is spread via saliva and droplets, and Droplet Precautions are necessary. Caregivers should wear a surgical mask when within 6 feet of the client and should continue to use Standard Precautions, including gloves. A particulate respirator, an isolation gown, and shoe covers are not necessary for Droplet Precautions.

A patient's electronic health record notes that the patient has hallux valgus. What signs and symptoms would the nurse expect this patient to manifest? A) Deviation of a great toe laterally B) Abnormal flexion of the great toe C) An exaggerated arch of the foot D) Fusion of the toe joints

A Feedback: A deformity in which the great toe deviates laterally and there is a marked prominence of the medial aspect of the first metatarsal-phalangeal joint and exostosis is referred to as hallux valgus (bunion). Hallux valgus does not result in abnormal flexion, abnormalities of the arch, or joint fusion.

A patient is scheduled for a myelogram and the nurse explains to the patient that this is an invasive procedure, which assesses for any lesions in the spinal cord. The nurse should explain that the preparation is similar to which of the following neurologic tests? A) Lumbar puncture B) MRI C) Cerebral angiography D) EEG

A Feedback: A myelogram is an x-ray of the spinal subarachnoid space taken after the injection of a contrast agent into the spinal subarachnoid space through a lumbar puncture. Patient preparation for a myelogram would be similar to that for lumbar puncture. The other listed diagnostic tests do not involve lumbar puncture.

A patient who has undergone a lower limb amputation is preparing to be discharged home. What outcome is necessary prior to discharge? A) Patient can demonstrate safe use of assistive devices. B) Patient has a healed, nontender, nonadherent scar. C) Patient can perform activities of daily living independently. D) Patients free of pain.

A Feedback: A patient should be able to use assistive devices appropriately and safely prior to discharge. Scar formation will not be complete at the time of hospital discharge. It is anticipated that the patient will require some assistance with ADLs postdischarge. Pain should be well managed, but may or may not be wholly absent.

An elderly female with osteoporosis has been hospitalized. Prior to discharge, when teaching the patient, the nurse should include information about which major complication of osteoporosis? A) Bone fracture B) Loss of estrogen C) Negative calcium balance D) Dowager's hump

A Feedback: Bone fracture is a major complication of osteoporosis that results when loss of calcium and phosphate increases the fragility of bones. Estrogen deficiencies result from menopause, not osteoporosis. Calcium and vitamin D supplements may be used to support normal bone metabolism, but a negative calcium balance is not a complication of osteoporosis. Dowager's hump results from bone fractures. It develops when repeated vertebral fractures increase spinal curvature.

The nurse is discussing the results of a patient's diagnostic testing with the nurse practitioner. What Weber test result would indicate the presence of a sensorineural loss? A) The sound is heard better in the ear in which hearing is better. B) The sound is heard equally in both ears. C) The sound is heard better in the ear in which hearing is poorer. D) The sound is heard longer in the ear in which hearing is better.

A Feedback: A patient with sensorineural hearing loss hears the sound better in the ear in which hearing is better. The Weber test assesses bone conduction of sound and is used for assessing unilateral hearing loss. A tuning fork is used. A patient with normal hearing hears the sound equally in both ears or describes the sound as centered in the middle of the head. A patient whose hearing loss is conductive hears the sound better in the affected ear.

A patient with a C5 spinal cord injury is tetraplegic. After being moved out of the ICU, the patient complains of a severe throbbing headache. What should the nurse do first? A) Check the patient's indwelling urinary catheter for kinks to ensure patency. B) Lower the HOB to improve perfusion. C) Administer analgesia. D) Reassure the patient that headaches are expected after spinal cord injuries.

A Feedback: A severe throbbing headache is a common symptom of autonomic dysreflexia, which occurs after injuries to the spinal cord above T6. The syndrome is usually brought on by sympathetic stimulation, such as bowel and bladder distention. Lowering the HOB can increase ICP. Before administering analgesia, the nurse should check the patient's catheter, record vital signs, and perform an abdominal assessment. A severe throbbing headache is a dangerous symptom in this patient and is not expected.

A patient who just suffered a suspected ischemic stroke is brought to the ED by ambulance. On what should the nurse's primary assessment focus? A) Cardiac and respiratory status B) Seizure activity C) Pain D) Fluid and electrolyte balance

A Feedback: Acute care begins with managing ABCs. Patients may have difficulty keeping an open and clear airway secondary to decreased LOC. Neurologic assessment with close monitoring for signs of increased neurologic deficit and seizure activity occurs next. Fluid and electrolyte balance must be controlled carefully with the goal of adequate hydration to promote perfusion and decrease further brain activity.

A patient was fitted with an arm cast after fracturing her humerus. Twelve hours after the application of the cast, the patient tells the nurse that her arm hurts. Analgesics do not relieve the pain. What would be the most appropriate nursing action? A) Prepare the patient for opening or bivalving of the cast. B) Obtain an order for a different analgesic. C) Encourage the patient to wiggle and move the fingers. D) Petal the edges of the patient's cast.

A Feedback: Acute compartment syndrome involves a sudden and severe decrease in blood flow to the tissues distal to an area of injury that results in ischemic necrosis if prompt, decisive intervention does not occur. Removing or bivalving the cast is necessary to relieve pressure. Ordering different analgesics does not address the underlying problem. Encouraging the patient to move the fingers or perform range-of-motion exercises will not treat or prevent compartment syndrome. Petaling the edges of a cast with tape prevents abrasions and skin breakdown, not compartment syndrome.

A patient with lower back pain is scheduled for myelography using metrizamide (a water-soluble contrast dye). After the test, the nurse should prioritize what action? A) Positioning the patient with the head of the bed elevated 45 degrees B) Administering IV morphine sulfate to prevent headache C) Limiting fluids for the next 12 hours D) Helping the patient perform deep breathing and coughing exercises

A Feedback: After myelography, the patient lies in bed with the head of the bed elevated 30 to 45 degrees. The patient is advised to remain in bed in the recommended position for 3 hours or as prescribed. Drinking liberal amounts of fluid for rehydration and replacement of CSF may decrease the incidence of post-lumbar puncture headache. Deep breathing and coughing exercises are not normally necessary since there is no consequent risk of atelectasis.

A patient was brought to the emergency department after a fall. The patient is taken to the operating room to receive a right hip prosthesis. In the immediate postoperative period, what health education should the nurse emphasize? A) Make sure you don't bring your knees close together. B) Try to lie as still as possible for the first few days. C) Try to avoid bending your knees until next week. D) Keep your legs higher than your chest whenever you can.

A Feedback: After receiving a hip prosthesis, the affected leg should be kept abducted. Mobility should be encouraged within safe limits. There is no need to avoid knee flexion and the patient's legs do not need to be higher than the level of the chest.

The nurse is helping to set up Buck's traction on an orthopedic patient. How often should the nurse assess circulation to the affected leg? A) Within 30 minutes, then every 1 to 2 hours B) Within 30 minutes, then every 4 hours C) Within 30 minutes, then every 8 hours D) Within 30 minutes, then every shift

A Feedback: After skin traction is applied, the nurse assesses circulation of the foot or hand within 15 to 30 minutes and then every 1 to 2 hours.

A rehabilitation nurse is working with a patient who has had a below-the-knee amputation. The nurse knows the importance of the patient's active participation in self-care. In order to determine the patient's ability to be an active participant in self-care, the nurse should prioritize assessment of what variable? A) The patient's attitude B) The patient's learning style C) The patient's nutritional status D) The patient's presurgical level of function

A Feedback: Amputation of an extremity affects the patient's ability to provide adequate self-care. The patient is encouraged to be an active participant in self-care. The patient and the nurse need to maintain positive attitudes and to minimize fatigue and frustration during the learning process. Balanced nutrition and the patient's learning style are important variables in the rehabilitation process but the patient's attitude is among the most salient variables. The patient's presurgical level of function may or may not affect participation in rehabilitation.

A nurse is caring for a patient whose cancer metastasis has resulted in bone pain. Which of the following are typical characteristics of bone pain? A) A dull, deep ache that is boring in nature B) Soreness or aching that may include cramping C) Sharp, piercing pain that is relieved by immobilization D) Spastic or sharp pain that radiates

A Feedback: Bone pain is characteristically described as a dull, deep ache that is boring in nature, whereas muscular pain is described as soreness or aching and is referred to as muscle cramps. Fracture pain is sharp and piercing and is relieved by immobilization. Sharp pain may also result from bone infection with muscle spasm or pressure on a sensory nerve.

A patient who has experienced an ischemic stroke has been admitted to the medical unit. The patient's family in adamant that she remain on bed rest to hasten her recovery and to conserve energy. What principle of care should inform the nurse's response to the family? A) The patient should mobilize as soon as she is physically able. B) To prevent contractures and muscle atrophy, bed rest should not exceed 4 weeks. C) The patient should remain on bed rest until she expresses a desire to mobilize. D) Lack of mobility will greatly increase the patient's risk of stroke recurrence.

A Feedback: As soon as possible, the patient is assisted out of bed and an active rehabilitation program is started. Delaying mobility causes complications, but not necessarily stroke recurrence. Mobility should not be withheld until the patient initiates.

A patient comes to the ophthalmology clinic for an eye examination. The patient tells the nurse that he often sees floaters in his vision. How should the nurse best interpret this subjective assessment finding? A) This is a normal aging process of the eye. B) Glasses will minimize this phenomenon. C) The patient may be exhibiting signs of glaucoma. D) This may be a result of weakened ciliary muscles.

A Feedback: As the body ages, the perfect gel-like characteristics of the vitreous humor are gradually lost, and various cells and fibers cast shadows that the patient perceives as floaters. This is a normal aging process.

The nurse is caring for a patient whose recent health history includes an altered LOC. What should be the nurse's first action when assessing this patient? A) Assessing the patient's verbal response B) Assessing the patient's ability to follow complex commands C) Assessing the patient's judgment D) Assessing the patient's response to pain

A Feedback: Assessment of the patient with an altered LOC often starts with assessing the verbal response through determining the patient's orientation to time, person, and place. In most cases, this assessment will precede each of the other listed assessments, even though each may be indicated.

A nurse is providing discharge teaching for a patient who underwent foot surgery. The nurse is collaborating with the occupational therapist and discussing the use of assistive devices. On what variables does the choice of assistive devices primarily depend? A) Patient's general condition, balance, and weight-bearing prescription B) Patient's general condition, strength, and gender C) Patient's motivation, age, and weight-bearing prescription D) Patient's occupation, motivation, and age

A Feedback: Assistive devices (e.g., crutches, walker) may be needed. The choice of the devices depends on the patient's general condition and balance, and on the weight-bearing prescription. The patient's strength, motivation, and weight restrictions are not what the choice of assistive devices is based on.

The school nurse has been called to the football field where player is immobile on the field after landing awkwardly on his head during a play. While awaiting an ambulance, what action should the nurse perform? A) Ensure that the player is not moved. B) Obtain the player's vital signs, if possible. C) Perform a rapid assessment of the player's range of motion. D) Assess the player's reflexes.

A Feedback: At the scene of the injury, the patient must be immobilized on a spinal (back) board, with the head and neck maintained in a neutral position, to prevent an incomplete injury from becoming complete. This is a priority over determining the patient's vital signs. It would be inappropriate to test ROM or reflexes.

The nurse is caring for a patient whose spinal cord injury has caused recent muscle spasticity. What medication should the nurse expect to be ordered to control this? A) Baclofen (Lioresal) B) Dexamethasone (Decadron) C) Mannitol (Osmitrol) D) Phenobarbital (Luminal)

A Feedback: Baclofen is classified as an antispasmodic agent in the treatment of muscles spasms related to spinal cord injury. Decadron is an anti-inflammatory medication used to decrease inflammation in both SCI and head injury. Mannitol is used to decrease cerebral edema in patients with head injury. Phenobarbital is an anticonvulsant that is used in the treatment of seizure activity.

19. A nurse is planning the care of an older adult patient with osteomalacia. What action should the nurse recommend in order to promote vitamin D synthesis? A) Ensuring adequate exposure to sunlight B) Eating a low-purine diet C) Performing cardiovascular exercise while avoiding weight-bearing exercises D) Taking thyroid supplements as ordered

A Feedback: Because sunlight is necessary for synthesizing vitamin D, patients should be encouraged to spend some time in the sun. A low-purine diet is not a relevant action and thyroid supplements do not directly affect bone function. Action must be taken to prevent fractures, but weight-bearing exercise within safe parameters is not necessarily contraindicated.

An older adult woman's current medication regimen includes alendronate (Fosamax). What outcome would indicate successful therapy? A) Increased bone mass B) Resolution of infection C) Relief of bone pain D) Absence of tumor spread

A Feedback: Bisphosphonates such as Fosamax increase bone mass and decrease bone loss by inhibiting osteoclast function. These drugs do not treat infection, pain, or tumors.

A patient who is being treated in the hospital for a spinal cord injury is advocating for the removal of his urinary catheter, stating that he wants to try to resume normal elimination. What principle should guide the care team's decision regarding this intervention? A) Urinary retention can have serious consequences in patients with SCIs. B) Urinary function is permanently lost following an SCI. C) Urinary catheters should not remain in place for more than 7 days. D) Overuse of urinary catheters can exacerbate nerve damage.

A Feedback: Bladder distention, a major cause of autonomic dysreflexia, can also cause trauma. For this reason, removal of a urinary catheter must be considered with caution. Extended use of urinary catheterization is often necessary following SCI. The effect of a spinal cord lesion on urinary function depends on the level of the injury. Catheter use does not cause nerve damage, although it is a major risk factor for UTIs.

An older adult patient has symptoms of osteoporosis and is being assessed during her annual physical examination. The assessment shows that the patient will require further testing related to a possible exacerbation of her osteoporosis. The nurse should anticipate what diagnostic test? A) Bone densitometry B) Hip bone radiography C) Computed tomography (CT) D) Magnetic resonance imaging (MRI)

A Feedback: Bone densitometry is considered the most accurate test for osteoporosis and for predicting a fracture. As such, it is more likely to be used than CT, MRI, or x-rays.

A patient is admitted to the neurologic ICU with a suspected diffuse axonal injury. What would be the primary neuroimaging diagnostic tool used on this patient to evaluate the brain structure? A) MRI B) PET scan C) X-ray D) Ultrasound

A Feedback: CT and MRI scans, the primary neuroimaging diagnostic tools, are useful in evaluating the brain structure. Ultrasound would not show the brain nor would an x-ray. A PET scan shows brain function, not brain structure.

A patient tells the nurse that he has pain and numbness to his thumb, first finger, and second finger of the right hand. The nurse discovers that the patient is employed as an auto mechanic, and that the pain is increased while working. This may indicate that the patient could possibly have what health problem? A) Carpel tunnel syndrome B) Tendonitis C) Impingement syndrome D) Dupuytren's contracture

A Feedback: Carpel tunnel syndrome may be manifested by numbness, pain, paresthesia, and weakness along the median nerve. Tendonitis is inflammation of muscle tendons. Impingement syndrome is a general term that describes all lesions that involve the rotator cuff of the shoulder. Dupuytren's contracture is a slowly progressive contracture of the palmar fascia.

17. A patient has developed diabetes insipidus after having increased ICP following head trauma. What nursing assessment best addresses this complication? A) Vigilant monitoring of fluid balance B) Continuous BP monitoring C) Serial arterial blood gases (ABGs) D) Monitoring of the patient's airway for patency

A Feedback: Diabetes insipidus requires fluid and electrolyte replacement, along with the administration of vasopressin, to replace and slow the urine output. Because of these alterations in fluid balance, careful monitoring is necessary. None of the other listed assessments directly addresses the major manifestations of diabetes insipidus.

A patient has been experiencing an unexplained decline in knee function and has consequently been scheduled for arthrography. The nurse should teach the patient about what process? A) Injection of a contrast agent into the knee joint prior to ROM exercises B) Aspiration of synovial fluid for serologic testing C) Injection of corticosteroids into the patient's knee joint to facilitate ROM D) Replacement of the patient's synovial fluid with a synthetic substitute

A Feedback: During arthrography, a radiopaque contrast agent or air is injected into the joint cavity to visualize the joint structures such as the ligaments, cartilage, tendons, and joint capsule. The joint is put through its range of motion to distribute the contrast agent while a series of x-rays are obtained. Synovial fluid is not aspirated or replaced and corticosteroids are not administered.

A hearing-impaired patient is scheduled to have an MRI. What would be important for the nurse to remember when caring for this patient? A) Patient is likely unable to hear the nurse during test. B) A person adept in sign language must be present during test. C) Lip reading will be the method of communication that is necessary. D) The nurse should interact with the patient like any other patient.

A Feedback: During health care and screening procedures, the practitioner (e.g., dentist, physician, nurse) must be aware that patients who are deaf or hearing-impaired are unable to read lips, see a signer, or read written materials in the dark rooms required during some diagnostic tests. The same situation exists if the practitioner is wearing a mask or not in sight (e.g., x-ray studies, MRI, colonoscopy).

A patient with a T2 injury is in spinal shock. The nurse will expect to observe what assessment finding? A) Absence of reflexes along with flaccid extremities B) Positive Babinski's reflex along with spastic extremities C) Hyperreflexia along with spastic extremities D) Spasticity of all four extremities

A Feedback: During the period immediately following a spinal cord injury, spinal shock occurs. In spinal shock, all reflexes are absent and the extremities are flaccid. When spinal shock subsides, the patient demonstrates a positive Babinski's reflex, hyperreflexia, and spasticity of all four extremities.

The nurse has created a plan of care for a patient who is at risk for increased ICP. The patient's care plan should specify monitoring for what early sign of increased ICP? A) Disorientation and restlessness B) Decreased pulse and respirations C) Projectile vomiting D) Loss of corneal reflex

A Feedback: Early indicators of ICP include disorientation and restlessness. Later signs include decreased pulse and respirations, projectile vomiting, and loss of brain stem reflexes, such as the corneal reflex.

When preparing to discharge a patient home, the nurse has met with the family and warned them that the patient may exhibit unexpected emotional responses. The nurse should teach the family that these responses are typically a result of what cause? A) Frustration around changes in function and communication B) Unmet physiologic needs C) Changes in brain activity during sleep and wakefulness D) Temporary changes in metabolism

A Feedback: Emotional problems associated with stroke are often related to the new challenges around ADLs and communication. These challenges are more likely than metabolic changes, unmet physiologic needs, or changes in brain activity, each of which should be ruled out.

14. A nurse is teaching a patient with glaucoma how to administer eye drops to achieve maximum absorption. The nurse should teach the patient to perform what action? A) Instill the medication in the conjunctival sac. B) Maintain a supine position for 10 minutes after administration. C) Keep the eyes closed for 1 to 2 minutes after administration. D) Apply the medication evenly to the sclera

A Feedback: Eye drops should be instilled into the conjunctival sac, where absorption can best take place, rather than distributed over the sclera. It is unnecessary to keep the eyes closed or to maintain a supine position after administration.

A patient is postoperative day 6 following tympanoplasty and mastoidectomy. The patient has phoned the surgical unit and states that she is experiencing occasional sharp, shooting pains in her affected ear. How should the nurse best interpret this patient's complaint? A) These pains are an expected finding during the first few weeks of recovery. B) The patient's complaints are suggestive of a postoperative infection. C) The patient may have experienced a spontaneous rupture of the tympanic membrane. D) The patient's surgery may have been unsuccessful.

A Feedback: For 2 to 3 weeks after surgery, the patient may experience sharp, shooting pains intermittently as the eustachian tube opens and allows air to enter the middle ear. Constant, throbbing pain accompanied by fever may indicate infection and should be reported to the primary care provider. The patient's pain does not suggest tympanic perforation or unsuccessful surgery

The nurse is planning the care of a patient with a T1 spinal cord injury. The nurse has identified the diagnosis of "risk for impaired skin integrity." How can the nurse best address this risk? A) Change the patient's position frequently. B) Provide a high-protein diet. C) Provide light massage at least daily. D) Teach the patient deep breathing and coughing exercises.

A Feedback: Frequent position changes are among the best preventative measures against pressure ulcers. A high-protein diet can benefit wound healing, but does not necessarily prevent skin breakdown. Light massage and deep breathing do not protect or restore skin integrity.

The health care team is caring for a patient with osteomalacia. It has been determined that the osteomalacia is caused by malabsorption. What is the usual treatment for osteomalacia caused by malabsorption? A) Supplemental calcium and increased doses of vitamin D B) Exogenous parathyroid hormone and multivitamins C) Colony-stimulating factors and calcitonin D) Supplemental potassium and pancreatic enzymes

A Feedback: If osteomalacia is caused by malabsorption, increased doses of vitamin D, along with supplemental calcium, are usually prescribed.

A nurse is planning the care of a patient who will require a prolonged course of skeletal traction. When planning this patient's care, the nurse should prioritize interventions related to which of the following risk nursing diagnoses? A) Risk for Impaired Skin Integrity B) Risk for Falls C) Risk for Imbalanced Fluid Volume D) Risk for Aspiration

A Feedback: Impaired skin integrity is a high-probability risk in patients receiving traction. Falls are not a threat, due to the patient's immobility. There are not normally high risks of fluid imbalance or aspiration associated with traction.

A nurse is planning the care of a patient who has undergone orthopedic surgery. What main goal should guide the nurse's choice of interventions? A) Improving the patient's level of function B) Helping the patient come to terms with limitations C) Administering medications safely D) Improving the patient's adherence to treatment

A Feedback: Improving function is the overarching goal after orthopedic surgery. Some patients may need to come to terms with limitations, but this is not true of every patient. Safe medication administration is imperative, but this is not a goal that guides other aspects of care. Similarly, adherence to treatment is important, but this is motivated by the need to improve functional status.

. A nurse is providing care for a patient whose pattern of laboratory testing reveals longstanding hypocalcemia. What other laboratory result is most consistent with this finding? A) An elevated parathyroid hormone level B) An increased calcitonin level C) An elevated potassium level D) A decreased vitamin D level

A Feedback: In the response to low calcium levels in the blood, increased levels of parathyroid hormone prompt the mobilization of calcium and the demineralization of bone. Increased calcitonin levels would exacerbate hypocalcemia. Vitamin D levels do not increase in response to low calcium levels. Potassium levels would likely be unaffected.

The nurse is caring for a patient who sustained a moderate head injury following a bicycle accident. The nurse's most recent assessment reveals that the patient's respiratory effort has increased. What is the nurse's most appropriate response? A) Inform the care team and assess for further signs of possible increased ICP. B) Administer bronchodilators as ordered and monitor the patient's LOC. C) Increase the patient's bed height and reassess in 30 minutes. D) Administer a bolus of normal saline as ordered.

A Feedback: Increased respiratory effort can be suggestive of increasing ICP, and the care team should be promptly informed. A bolus of IV fluid will not address the problem. Repositioning the patient and administering bronchodilators are insufficient responses, even though these actions may later be ordered.

After a major ischemic stroke, a possible complication is cerebral edema. Nursing care during the immediate recovery period from an ischemic stroke should include which of the following? A) Positioning to avoid hypoxia B) Maximizing PaCO2 C) Administering hypertonic IV solution D) Initiating early mobilization

A Feedback: Interventions during this period include measures to reduce ICP, such as administering an osmotic diuretic (e.g., mannitol), maintaining the partial pressure of carbon dioxide (PaCO2) within the range of 30 to 35 mm Hg, and positioning to avoid hypoxia. Hypertonic IV solutions are not used unless sodium depletion is evident. Mobilization would take place after the immediate threat of increased ICP has past.

. A patient has just had an arthroscopy performed to assess a knee injury. What nursing intervention should the nurse implement following this procedure? A) Wrap the joint in a compression dressing. B) Perform passive range of motion exercises. C) Maintain the knee in flexion for up to 30 minutes. D) Apply heat to the knee.

A Feedback: Interventions to perform following an arthroscopy include wrapping the joint in a compression dressing, extending and elevating the joint, and applying ice or cold packs. Passive ROM exercises, static flexion, and heat are not indicated.

The nurse educator on an orthopedic trauma unit is reviewing the safe and effective use of traction with some recent nursing graduates. What principle should the educator promote? A) Knots in the rope should not be resting against pulleys. B) Weights should rest against the bed rails. C) The end of the limb in traction should be braced by the footboard of the bed. D) Skeletal traction may be removed for brief periods to facilitate the patient's independence.

A Feedback: Knots in the rope should not rest against pulleys, because this interferes with traction. Weights are used to apply the vector of force necessary to achieve effective traction and should hang freely at all times. To avoid interrupting traction, the limb in traction should not rest against anything. Skeletal traction is never interrupted.

A patient got a sliver of glass in his eye when a glass container at work fell and shattered. The glass had to be surgically removed and the patient is about to be discharged home. The patient asks the nurse for a topical anesthetic for the pain in his eye. What should the nurse respond? A) Overuse of these drops could soften your cornea and damage your eye. B) You could lose the peripheral vision in your eye if you used these drops too much. C) I'm sorry, this medication is considered a controlled substance and patients cannot take it home. D) I know these drops will make your eye feel better, but I can't let you take them home.

A Feedback: Most patients are not allowed to take topical anesthetics home because of the risk of overuse. Patients with corneal abrasions and erosions experience severe pain and are often tempted to overuse topical anesthetic eye drops. Overuse of these drops results in softening of the cornea. Prolonged use of anesthetic drops can delay wound healing and can lead to permanent corneal opacification and scarring, resulting in visual loss. The nurse must explain the rationale for limiting the home use of these medications.

Diagnostic tests show that a patient's bone density has decreased over the past several years. The patient asks the nurse what factors contribute to bone density decreasing. What would be the nurse's best response? A) For many people, lack of nutrition can cause a loss of bone density. B) Progressive loss of bone density is mostly related to your genes. C) Stress is known to have many unhealthy effects, including reduced bone density. D) Bone density decreases with age, but scientists are not exactly sure why this is the case.

A Feedback: Nutrition has a profound effect on bone density, especially later life. Genetics are also an important factor, but nutrition has a more pronounced effect. The pathophysiology of bone density is well understood and psychosocial stress has a minimal effect.

A child is growing at a rate appropriate for his age. What cells are responsible for the secretion of bone matrix that eventually results in bone growth? A) Osteoblasts B) Osteocytes C) Osteoclasts D) Lamellae

A Feedback: Osteoblasts function in bone formation by secreting bone matrix. Osteocytes are mature bone cells and osteoclasts are multinuclear cells involved in dissolving and resorbing bone. Lamellae are circles of mineralized bone matrix.

A patient is being given a medication that stimulates her parasympathetic system. Following administration of this medication, the nurse should anticipate what effect? A) Constricted pupils B) Dilated bronchioles C) Decreased peristaltic movement D) Relaxed muscular walls of the urinary bladder

A Feedback: Parasympathetic stimulation results in constricted pupils, constricted bronchioles, increased peristaltic movement, and contracted muscular walls of the urinary bladder.

A patient with glaucoma has presented for a scheduled clinic visit and tells the nurse that she has begun taking an herbal remedy for her condition that was recommended by a work colleague. What instruction should the nurse provide to the patient? A) The patient should discuss this new remedy with her ophthalmologist promptly. B) The patient should monitor her IOP closely for the next several weeks. C) The patient should do further research on the herbal remedy. D) The patient should report any adverse effects to her pharmacist.

A Feedback: Patients should discuss any new treatments with an ophthalmologist; this should precede the patient's own further research or reporting adverse effects to the pharmacist. Self-monitoring of IOP is not possible.

A patient who suffered an ischemic stroke now has disturbed sensory perception. What principle should guide the nurse's care of this patient? A) The patient should be approached on the side where visual perception is intact. B) Attention to the affected side should be minimized in order to decrease anxiety. C) The patient should avoid turning in the direction of the defective visual field to minimize shoulder subluxation. D) The patient should be approached on the opposite side of where the visual perception is intact to promote recovery.

A Feedback: Patients with decreased field of vision should first be approached on the side where visual perception is intact. All visual stimuli should be placed on this side. The patient can and should be taught to turn the head in the direction of the defective visual field to compensate for this loss. The nurse should constantly remind the patient of the other side of the body and should later stand at a position that encourages the patient to move or turn to visualize who and what is in the room.

The nurse in the ED is caring for a 4 year-old brought in by his parents who state that the child will not stop crying and pulling at his ear. Based on information collected by the nurse, which of the following statements applies to a diagnosis of external otitis? A) External otitis is characterized by aural tenderness. B) External otitis is usually accompanied by a high fever. C) External otitis is usually related to an upper respiratory infection. D) External otitis can be prevented by using cotton-tipped applicators to clean the ear.

A Feedback: Patients with otitis externa usually exhibit pain, discharge from the external auditory canal, and aural tenderness. Fever and accompanying upper respiratory infection occur more commonly in conjunction with otitis media (infection of the middle ear). Cotton-tipped applicators can actually cause external otitis so their use should be avoided.

A gerontologic nurse educator is providing practice guidelines to unlicensed care providers. Because reaction to painful stimuli is sometimes blunted in older adults, what must be used with caution? A) Hot or cold packs B) Analgesics C) Anti-inflammatory medications D) Whirlpool baths

A Feedback: Reaction to painful stimuli may be decreased with age. Because pain is an important warning signal, caution must be used when hot or cold packs are used. The older patient may be burned or suffer frostbite before being aware of any discomfort. Any medication is used with caution in the elderly, but not because of the decreased sense of heat or cold. Whirlpool baths are generally not a routine treatment ordered for the elderly.

A patient presents to a clinic complaining of a leg ulcer that isn't healing; subsequent diagnostic testing suggests osteomyelitis. The nurse is aware that the most common pathogen to cause osteomyelitis is what? A) Staphylococcus aureus B) Proteus C) Pseudomonas D) Escherichia coli

A Feedback: S. aureus causes over 50% of bone infections. Proteus, Pseudomonas, and E. coli are also causes, but to a lesser extent.

he nursing care plan for a patient in traction specifies regular assessments for venous thromboembolism (VTE). When assessing a patient's lower limbs, what sign or symptom is suggestive of deep vein thrombosis (DVT)? A) Increased warmth of the calf B) Decreased circumference of the calf C) Loss of sensation to the calf D) Pale-appearing calf

A Feedback: Signs of DVT include increased warmth, redness, swelling, and calf tenderness. These findings are promptly reported to the physician for definitive evaluation and therapy. Signs and symptoms of a DVT do not include a decreased circumference of the calf, a loss of sensation in the calf, or a pale-appearing calf.

The nurse is caring for a patient with increased intracranial pressure (ICP) caused by a traumatic brain injury. Which of the following clinical manifestations would suggest that the patient may be experiencing increased brain compression causing brain stem damage? A) Hyperthermia B) Tachycardia C) Hypertension D) Bradypnea

A Feedback: Signs of increasing ICP include slowing of the heart rate (bradycardia), increasing systolic BP, and widening pulse pressure. As brain compression increases, respirations become rapid, BP may decrease, and the pulse slows further. A rapid rise in body temperature is regarded as unfavorable. Hyperthermia increases the metabolic demands of the brain and may indicate brain stem damage.

A patient is postoperative day 1 following intracranial surgery. The nurse's assessment reveals that the patient's LOC is slightly decreased compared with the day of surgery. What is the nurse's best response to this assessment finding? A) Recognize that this may represent the peak of post-surgical cerebral edema. B) Alert the surgeon to the possibility of an intracranial hemorrhage. C) Understand that the surgery may have been unsuccessful. D) Recognize the need to refer the patient to the palliative care team.

A Feedback: Some degree of cerebral edema occurs after brain surgery; it tends to peak 24 to 36 hours after surgery, producing decreased responsiveness on the second postoperative day. As such, there is not necessarily any need to deem the surgery unsuccessful or to refer the patient to palliative care. A decrease in LOC is not evidence of an intracranial hemorrhage.

A patient with otosclerosis has significant hearing loss. What should the nurse do to best facilitate communication with the patient? A) Sit or stand in front of the patient when speaking. B) Use exaggerated lip and mouth movements when talking. C) Stand in front of a light or window when speaking. D) Say the patient's name loudly before starting to talk.

A Feedback: Standing directly in front of a hearing-impaired patient allows him or her to lip-read and see facial expressions that offer clues to what is being said. Using exaggerated lip and mouth movements can make lip-reading more difficult by distorting words. Backlighting can create glare, making it difficult for the patient to lip-read. To get the attention of a hearing-impaired patient, gently touch the patient's shoulder or stand in front of the patient.

The patient scheduled for a Syme amputation is concerned about the ability to eventually stand on the amputated extremity. How should the nurse best respond to the patient's concern? A) You will eventually be able to withstand full weight-bearing after the amputation. B) You will have minimal weight-bearing on this extremity but you'll be taught how to use an assistive device. C) You likely will not be able to use this extremity but you will receive teaching on use of a wheelchair. D) You will be fitted for a prosthesis which may or may not allow you to walk.

A Feedback: Syme amputation (modified ankle disarticulation amputation) is performed most frequently for extensive foot trauma and produces a painless, durable extremity end that can withstand full weight-bearing. Therefore, each of the other teaching statements is incorrect.

An advanced practice nurse has performed a Rinne test on a new patient. During the test, the patient reports that air-conducted sound is louder than bone-conducted sound. How should the nurse best interpret this assessment finding? A) The patient's hearing is likely normal. B) The patient is at risk for tinnitus. C) The patient likely has otosclerosis. D) The patient likely has sensorineural hearing loss.

A Feedback: The Rinne test is useful for distinguishing between conductive and sensorineural hearing loss. A person with normal hearing reports that air-conducted sound is louder than bone-conducted sound.

A nurse is teaching a patient with osteomalacia about the role of diet. What would be the best choice for breakfast for a patient with osteomalacia? A) Cereal with milk, a scrambled egg, and grapefruit B) Poached eggs with sausage and toast C) Waffles with fresh strawberries and powdered sugar D) A bagel topped with butter and jam with a side dish of grapes

A Feedback: The best meal option is the one that contains the highest dietary sources of calcium and vitamin D. The best selection among those listed is cereal with milk, and eggs, as these foods contain calcium and vitamin D in a higher quantity over the other menu options.

A patient exhibiting an uncoordinated gait has presented at the clinic. Which of the following is the most plausible cause of this patient's health problem? A) Cerebellar dysfunction B) A lesion in the pons C) Dysfunction of the medulla D) A hemorrhage in the midbrain

A Feedback: The cerebellum controls fine movement, balance, position sense, and integration of sensory input. Portions of the pons control the heart, respiration, and blood pressure. Cranial nerves IX through XII connect to the brain in the medulla. Cranial nerves III and IV originate in the midbrain.

A patient with mastoiditis is admitted to the post-surgical unit after undergoing a radical mastoidectomy. The nurse should identify what priority of postoperative care? A) Assessing for mouth droop and decreased lateral eye gaze B) Assessing for increased middle ear pressure and perforated ear drum C) Assessing for gradual onset of conductive hearing loss and nystagmus D) Assessing for scar tissue and cerumen obstructing the auditory canal

A Feedback: The facial nerve runs through the middle ear and the mastoid; therefore, there is risk of injuring this nerve during a mastoidectomy. When injury occurs, the patient may display mouth droop and decreased lateral gaze on the operative side. Scar tissue is a long-term complication of tympanoplasty and therefore would not be evident during the immediate postoperative period. Tympanic perforation is not a common complication of this surgery.

A patient has had a sudden loss of vision after head trauma. How should the nurse best describe the placement of items on the dinner tray? A) Explain the location of items using clock cues. B) Explain that each of the items on the tray is clearly separated. C) Describe the location of items from the bottom of the plate to the top. D) Ask the patient to describe the location of items before confirming their location.

A Feedback: The food tray's composition is likened to the face of a clock. It is unreasonable to expect the patient to describe the location of items or to state that items are separated.

A nurse is caring for a patient who is postoperative day 1 right hip replacement. How should the nurse position the patient? A) Keep the patient's hips in abduction at all times. B) Keep hips flexed at no less than 90 degrees. C) Elevate the head of the bed to high Fowler's. D) Seat the patient in a low chair as soon as possible.

A Feedback: The hips should be kept in abduction by an abductor pillow. Hips should not be flexed more than 90 degrees, and the head of bed should not be elevated more than 60 degrees. The patient's hips should be higher than the knees; as such, high seat chairs should be used.

A patient is admitted to the medical unit with an exacerbation of multiple sclerosis. When assessing this patient, the nurse has the patient stick out her tongue and move it back and forth. What is the nurse assessing? A) Function of the hypoglossal nerve B) Function of the vagus nerve C) Function of the spinal nerve D) Function of the trochlear nerve

A Feedback: The hypoglossal nerve is the 12th cranial nerve. It is responsible for movement of the tongue. None of the other listed nerves affects motor function in the tongue.

The patient has been diagnosed with aphasia after suffering a stroke. What can the nurse do to best make the patient's atmosphere more conducive to communication? A) Provide a board of commonly used needs and phrases. B) Have the patient speak to loved ones on the phone daily. C) Help the patient complete his or her sentences. D) Speak in a loud and deliberate voice to the patient.

A Feedback: The inability to talk on the telephone or answer a question or exclusion from conversation causes anger, frustration, fear of the future, and hopelessness. A common pitfall is for the nurse or other health care team member to complete the thoughts or sentences of the patient. This should be avoided because it may cause the patient to feel more frustrated at not being allowed to speak and may deter efforts to practice putting thoughts together and completing a sentence. The patient may also benefit from a communication board, which has pictures of commonly requested needs and phrases. The board may be translated into several languages.

The patient in the ED has just had a diagnostic lumbar puncture. To reduce the incidence of a post-lumbar puncture headache, what is the nurse's most appropriate action? A) Position the patient prone. B) Position the patient supine with the head of bed flat. C) Position the patient left side-lying. D) Administer acetaminophen as ordered.

A Feedback: The lumbar puncture headache may be avoided if a small-gauge needle is used and if the patient remains prone after the procedure. Acetaminophen is not administered as a preventative measure for post-lumbar puncture headaches.

The nurse should recognize the greatest risk for the development of blindness in which of the following patients? A) A 58-year-old Caucasian woman with macular degeneration B) A 28-year-old Caucasian man with astigmatism C) A 58-year-old African American woman with hyperopia D) A 28-year-old African American man with myopia

A Feedback: The most common causes of blindness and visual impairment among adults 40 years of age or older are diabetic retinopathy, macular degeneration, glaucoma, and cataracts. The 58-year-old Caucasian woman with macular degeneration has the greatest risk for the development of blindness related to her age and the presence of macular degeneration. Individuals with hyperopia, astigmatism, and myopia are not in a risk category for blindness.

When caring for a patient with increased ICP the nurse knows the importance of monitoring for possible secondary complications, including syndrome of inappropriate antidiuretic hormone (SIADH). What nursing interventions would the nurse most likely initiate if the patient developed SIADH? A) Fluid restriction B) Transfusion of platelets C) Transfusion of fresh frozen plasma (FFP) D) Electrolyte restriction

A Feedback: The nurse also assesses for complications of increased ICP, including diabetes insipidus, and SIADH. SIADH requires fluid restriction and monitoring of serum electrolyte levels. Transfusions are unnecessary.

A nurse is caring for a patient who had a right below-the-knee amputation (BKA). The nurse recognizes the importance of implementing measures that focus on preventing flexion contracture of the hip and maintaining proper positioning. Which of the following measures will best achieve these goals? A) Encouraging the patient to turn from side to side and to assume a prone position B) Initiating ROM exercises of the hip and knee 10 to 12 weeks after the amputation C) Minimizing movement of the flexor muscles of the hip D) Encouraging the patient to sit in a chair for at least 8 hours a day

A Feedback: The nurse encourages the patient to turn from side to side and to assume a prone position, if possible, to stretch the flexor muscles and to prevent flexion contracture of the hip. Postoperative ROM exercises are started early, because contracture deformities develop rapidly. ROM exercises include hip and knee exercises for patients with BKAs. The nurse also discourages sitting for prolonged periods of time.

A 6-year-old child is brought to the pediatric clinic for the assessment of redness and discharge from the eye and is diagnosed with viral conjunctivitis. What is the most important information to discuss with the parents and child? A) Handwashing can prevent the spread of the disease to others. B) The importance of compliance with antibiotic therapy C) Signs and symptoms of complications, such as meningitis and septicemia D) The likely need for surgery to prevent scarring of the conjunctiva

A Feedback: The nurse must inform the parents and child that viral conjunctivitis is highly contagious and instructions should emphasize the importance of handwashing and avoiding sharing towels, face cloths, and eye drops. Viral conjunctivitis is not responsive to any treatment, including antibiotic therapy. Patients with gonococcal conjunctivitis are at risk for meningitis and generalized septicemia; these conditions do not apply to viral conjunctivitis. Surgery to prevent scarring of the conjunctiva is not associated with viral conjunctivitis.

A patient has returned to the postsurgical unit from the PACU after an above-the-knee amputation of the right leg. Results of the nurse's initial postsurgical assessment were unremarkable but the patient has called out. The nurse enters the room and observes copious quantities of blood at the surgical site. What should be the nurse's initial action? A) Apply a tourniquet. B) Elevate the residual limb. C) Apply sterile gauze. D) Call the surgeon.

A Feedback: The nurse should apply a tourniquet in the event of postsurgical hemorrhage. Elevating the limb and applying sterile gauze are likely insufficient to stop the hemorrhage. The nurse should attempt to control the immediate bleeding before contacting the surgeon.

The nurse has taken shift report on her patients and has been told that one patient has an ocular condition that has primarily affected the rods in his eyes. Considering this information, what should the nurse do while caring for the patient? A) Ensure adequate lighting in the patient's room. B) Provide a dimly lit room to aid vision by limiting contrast. C) Carefully point out color differences for the patient. D) Carefully point out fine details for the patient.

A Feedback: The nurse should provide adequate lighting in the patient's room, as the rods are mainly responsible for night vision or vision in low light. If the patient's rods are impaired, the patient will have difficulty seeing in dim light. The cones in the eyes provide best vision for bright light, color vision, and fine detail.

1. A nurse on the orthopedic unit is assessing a patient's peroneal nerve. The nurse will perform this assessment by doing which of the following actions? A) Pricking the skin between the great and second toe B) Stroking the skin on the sole of the patient's foot C) Pinching the skin between the thumb and index finger D) Stroking the distal fat pad of the small finger

A Feedback: The nurse will evaluate the sensation of the peroneal nerve by pricking the skin centered between the great and second toe. None of the other listed actions elicits the function of one of the peripheral nerves.

A nurse is caring for an adult patient diagnosed with a back strain. What health education should the nurse provide to this patient? A) Avoid lifting more than one-third of body weight without assistance. B) Focus on using back muscles efficiently when lifting heavy objects. C) Lift objects while holding the object a safe distance from the body. D) Tighten the abdominal muscles and lock the knees when lifting of an object.

A Feedback: The nurse will instruct the patient on the safe and correct way to lift objectsusing the strong quadriceps muscles of the thighs, with minimal use of the weak back muscles. To prevent recurrence of acute low back pain, the nurse may instruct the patient to avoid lifting more than one-third of his weight without help. The patient should be informed to place the feet a hip-width apart to provide a wide base of support, the person should bend the knees, tighten the abdominal muscles, and lift the object close to the body with a smooth motion, avoiding twisting and jerking.

The nurse is caring for a patient with permanent neurologic impairments resulting from a traumatic head injury. When working with this patient and family, what mutual goal should be prioritized? A) Achieve as high a level of function as possible. B) Enhance the quantity of the patient's life. C) Teach the family proper care of the patient. D) Provide community assistance.

A Feedback: The overarching goals of care are to achieve as high a level of function as possible and to enhance the quality of life for the patient with neurologic impairment and his or her family. This goal encompasses family and community participation.

A nurse is writing a care plan for a patient admitted to the emergency department (ED) with an open fracture. The nurse will assign priority to what nursing diagnosis for a patient with an open fracture of the radius? A) Risk for Infection B) Risk for Ineffective Role Performance C) Risk for Perioperative Positioning Injury D) Risk for Powerlessness

A Feedback: The patient has a significant risk for osteomyelitis and tetanus due to the fact that the fracture is open. Powerlessness and ineffective role performance are psychosocial diagnoses that may or may not apply, and which would be superseded by immediate physiologic threats such as infection. Surgical positioning injury is not plausible, since surgery is not likely indicated.

Following diagnostic testing, a patient has been admitted to the ICU and placed on cerebral aneurysm precautions. What nursing action should be included in patient's plan of care? A) Supervise the patient's activities of daily living closely. B) Initiate early ambulation to prevent complications of immobility. C) Provide a high-calorie, low-protein diet. D) Perform all of the patient's hygiene and feeding.

A Feedback: The patient is placed on immediate and absolute bed rest in a quiet, nonstressful environment, because activity, pain, and anxiety elevate BP, which increases the risk for bleeding. As such, independent ADLs and ambulation are contraindicated. There is no need for a high-calorie or low-protein diet.

A 32-year-old patient comes to the clinic complaining of shoulder tenderness, pain, and limited movement. Upon assessment the nurse finds edema. An MRI shows hemorrhage of the rotator cuff tendons and the patient is diagnosed with impingement syndrome. What action should the nurse recommend in order to promote healing? A) Support the affected arm on pillows at night. B) Take prescribed corticosteroids as ordered. C) Put the shoulder through its full range of motion 3 times daily. D) Keep the affected arm in a sling for 2 to 4 weeks.

A Feedback: The patient should support the affected arm on pillows while sleeping to keep from turning onto the shoulder. Corticosteroids are not commonly prescribed and a sling is not normally necessary. ROM exercises are indicated, but putting the arm through its full ROM may cause damage during the healing process.

A patient is recovering from intracranial surgery performed approximately 24 hours ago and is complaining of a headache that the patient rates at 8 on a 10-point pain scale. What nursing action is most appropriate? A) Administer morphine sulfate as ordered. B) Reposition the patient in a prone position. C) Apply a hot pack to the patient's scalp. D) Implement distraction techniques.

A Feedback: The patient usually has a headache after a craniotomy as a result of stretching and irritation of nerves in the scalp during surgery. Morphine sulfate may also be used in the management of postoperative pain in patients who have undergone a craniotomy. Prone positioning is contraindicated due to the consequent increase in ICP. Distraction would likely be inadequate to reduce pain and a hot pack may cause vasodilation and increased pain.

The nurse is providing discharge education for a patient with a new diagnosis of Ménière's disease. What food should the patient be instructed to limit or avoid? A) Sweet pickles B) Frozen yogurt C) Shellfish D) Red meat

A Feedback: The patient with Ménière's disease should avoid foods high in salt and/or sugar; sweet pickles are high in both. Milk products are not contraindicated. Any type of meat, fish, or poultry is permitted, with the exception of canned or pickled varieties. In general, the patient with Ménière's disease should avoid or limit canned and processed foods.

A nurse is collaborating with the physical therapist to plan the care of a patient with osteomyelitis. What principle should guide the management of activity and mobility in this patient? A) Stress on the weakened bone must be avoided. B) Increased heart rate enhances perfusion and bone healing. C) Bed rest results in improved outcomes in patients with osteomyelitis. D) Maintenance of baseline ADLs is the primary goal during osteomyelitis treatment.

A Feedback: The patient with osteomyelitis has bone that is weakened by the infective process and must be protected by avoidance of stress on the bone.This risk guides the choice of activity in a patient with osteomyelitis. Bed rest is not normally indicated, however. Maintenance of prediagnosis ADLs may be an unrealistic short-term goal for many patients.

The nurse is providing health education to a patient who has a C6 spinal cord injury. The patient asks why autonomic dysreflexia is considered an emergency. What would be the nurse's best answer? A) "The sudden increase in BP can raise the ICP or rupture a cerebral blood vessel." B) "The suddenness of the onset of the syndrome tells us the body is struggling to maintain its normal state." C) "Autonomic dysreflexia causes permanent damage to delicate nerve fibers that are healing." D) "The sudden, severe headache increases muscle tone and can cause further nerve damage."

A Feedback: The sudden increase in BP may cause a rupture of one or more cerebral blood vessels or lead to increased ICP. Autonomic dysreflexia does not directly cause nerve damage.

A patient's ocular tumor has necessitated enucleation and the patient will be fitted with a prosthesis. The nurse should address what nursing diagnosis when planning the patient's discharge education? A) Disturbed body image B) Chronic pain C) Ineffective protection D) Unilateral neglect

A Feedback: The use of an ocular prosthesis is likely to have a significant impact on a patient's body image. Prostheses are not associated with chronic pain or ineffective protection. The patient experiences a change in vision, but is usually able to accommodate such changes and prevent unilateral neglect.

patient is brought by ambulance to the ED after suffering what the family thinks is a stroke. The nurse caring for this patient is aware that an absolute contraindication for thrombolytic therapy is what? A) Evidence of hemorrhagic stroke B) Blood pressure of ³ 180/110 mm Hg C) Evidence of stroke evolution D) Previous thrombolytic therapy within the past 12 months

A Feedback: Thrombolytic therapy would exacerbate a hemorrhagic stroke with potentially fatal consequences. Stroke evolution, high BP, or previous thrombolytic therapy does not contraindicate its safe and effective use.

12. A patient diagnosed with arthritis has been taking aspirin and now reports experiencing tinnitus and hearing loss. What should the nurse teach this patient? A) The hearing loss will likely resolve with time after the drug is discontinued. B) The patient's hearing loss and tinnitus are irreversible at this point. C) The patient's tinnitus is likely multifactorial, and not directly related to aspirin use. D) The patient's tinnitus will abate as tolerance to aspirin develops.

A Feedback: Tinnitus and hearing loss are signs of ototoxicity, which is associated with aspirin use. In most cases, this will resolve upon discontinuing the aspirin. Many other drugs cause irreversible ototoxicity.

A nurse is caring for a patient who is 12 hours postoperative following foot surgery. The nurse assesses the presence of edema in the foot. What nursing measure will the nurse implement to control the edema? A) Elevate the foot on several pillows. B) Apply warm compresses intermittently to the surgical area. C) Administer a loop diuretic as ordered. D) Increase circulation through frequent ambulation.

A Feedback: To control the edema in the foot of a patient who experienced foot surgery, the nurse will elevate the foot on several pillows when the patient is sitting or lying. Diuretic therapy is not an appropriate intervention for edema related to inflammation. Intermittent ice packs should be applied to the surgical area during the first 24 to 48 hours after surgery to control edema and provide some pain relief. Ambulation will gradually be resumed based on the guidelines provided by the surgeon.

A hospital patient has experienced a seizure. In the immediate recovery period, what action best protects the patient's safety? A) Place the patient in a side-lying position. B) Pad the patient's bed rails. C) Administer antianxiety medications as ordered. D) Reassure the patient and family members.

A Feedback: To prevent complications, the patient is placed in the side-lying position to facilitate drainage of oral secretions. Suctioning is performed, if needed, to maintain a patent airway and prevent aspiration. None of the other listed actions promotes safety during the immediate recovery period.

An 82-year-old man is admitted for observation after a fall. Due to his age, the nurse knows that the patient is at increased risk for what complication of his injury? A) Hematoma B) Skull fracture C) Embolus D) Stroke

A Feedback: Two major factors place older adults at increased risk for hematomas. First, the dura becomes more adherent to the skull with increasing age. Second, many older adults take aspirin and anticoagulants as part of routine management of chronic conditions. Because of these factors, the patient's risk of a hematoma is likely greater than that of stroke, embolism, or skull fracture.

A patient broke his arm in a sports accident and required the application of a cast. Shortly following application, the patient complained of an inability to straighten his fingers and was subsequently diagnosed with Volkmann contracture. What pathophysiologic process caused this complication? A) Obstructed arterial blood flow to the forearm and hand B) Simultaneous pressure on the ulnar and radial nerves C) Irritation of Merkel cells in the patient's skin surfaces D) Uncontrolled muscle spasms in the patient's forearm

A Feedback: Volkmann contracture occurs when arterial blood flow is restricted to the forearm and hand and results in contractures of the fingers and wrist. It does not result from nerve pressure, skin irritation, or spasms.

The advanced practice nurse is attempting to examine the patient's ear with an otoscope. Because of impacted cerumen, the tympanic membrane cannot be visualized. The nurse irrigates the patient's ear with a solution of hydrogen peroxide and water to remove the impacted cerumen. What nursing intervention is most important to minimize nausea and vertigo during the procedure? A) Maintain the irrigation fluid at a warm temperature. B) Instill short, sharp bursts of fluid into the ear canal. C) Follow the procedure with insertion of a cerumen curette to extract missed ear wax. D) Have the patient stand during the procedure.

A Feedback: Warm water (never cold or hot) and gentle, not forceful, irrigation should be used to remove cerumen. Too forceful irrigation can cause perforation of the tympanic membrane, and ice water causes vomiting. Cerumen curettes should not be routinely used by the nurse. Special training is required to use a curette safely. It is unnecessary to have the patient stand during the procedure.

The nurse is performing an assessment of a patient's musculoskeletal system and is appraising the patient's bone integrity. What action should the nurse perform during this phase of assessment? A) Compare parts of the body symmetrically. B) Assess extremities when in motion rather than at rest. C) Percuss as many joints as are accessible. D) Administer analgesia 30 to 60 minutes before assessment.

A Feedback: When assessing bone integrity, symmetric parts of the body, such as extremities, are compared. Analgesia should not be necessary and percussion is not a clinically useful assessment technique. Bone integrity is best assessed when the patient is not moving.

An 80-year-old man in a long-term care facility has a chronic leg ulcer and states that the area has become increasingly painful in recent days. The nurse notes that the site is now swollen and warm to the touch. The patient should undergo diagnostic testing for what health problem? A) Osteomyelitis B) Osteoporosis C) Osteomalacia D) Septic arthritis

A Feedback: When osteomyelitis develops from the spread of an adjacent infection, no signs of septicemia are present, but the area becomes swollen, warm, painful, and tender to touch. Osteoporosis is the most prevalent bone disease in the world. Osteomalacia is a metabolic bone disease characterized by inadequate mineralization of bone. Septicarthritis occurs when joints become infected through spread of infection from other parts of the body (hematogenous spread) or directly through trauma or surgical instrumentation.

14. An emergency department nurse is assessing a 17-year-old soccer player who presented with a knee injury. The patient's description of the injury indicates that his knee was struck medially while his foot was on the ground. The nurse knows that the patient likely has experienced what injury? A) Lateral collateral ligament injury B) Medial collateral ligament injury C) Anterior cruciate ligament injury D) Posterior cruciate ligament injury

A Feedback: When the knee is struck medially, damage may occur to the lateral collateral ligament. If the knee is struck laterally, damage may occur to the medial collateral ligament. The ACL and PCL are not typically injured in this way.

A client presents to the emergency department in sickle cell crisis. What intervention by the nurse takes priority? a. Administer oxygen. b. Apply an oximetry probe. c. Give pain medication. d. Start an IV line.

A All actions are appropriate, but remembering the ABCs, oxygen would come first. The main problem in a sickle cell crisis is tissue and organ hypoxia, so providing oxygen helps halt the process.

A nurse prepares a client for lumbar puncture (LP). Which assessment finding should alert the nurse to contact the health care provider? a. Shingles on the client's back b. Client is claustrophobic c. Absence of intravenous access d. Paroxysmal nocturnal dyspnea

A An LP should not be performed if the client has a skin infection at or near the puncture site because of the risk of infection. A nurse would want to notify the health care provider if shingles were identified on the client's back. If a client has shortness of breath when lying flat, the LP can be adapted to meet the client's needs. Claustrophobia, absence of IV access, and paroxysmal nocturnal dyspnea have no impact on whether an LP can be performed.

A nurse caring for a client with sickle cell disease (SCD) reviews the client's laboratory work. Which finding should the nurse report to the provider? a. Creatinine: 2.9 mg/dL b. Hematocrit: 30% c. Sodium: 147 mEq/L d. White blood cell count: 12,000/mm3

A An elevated creatinine indicates kidney damage, which occurs in SCD. A hematocrit level of 30% is an expected finding, as is a slightly elevated white blood cell count. A sodium level of 147 mEq/L, although slightly high, is not concerning.

A client in sickle cell crisis is dehydrated and in the emergency department. The nurse plans to start an IV. Which fluid choice is best? a. 0.45% normal saline b. 0.9% normal saline c. Dextrose 50% (D50) d. Lactated Ringer's solution

A Because clients in sickle cell crisis are often dehydrated, the fluid of choice is a hypotonic solution such as 0.45% normal saline. 0.9% normal saline and lactated Ringer's solution are isotonic. D50 is hypertonic and not used for hydration.

A nurse teaches a client who is scheduled for a positron emission tomography scan of the brain. Which statement should the nurse include in this client's teaching? a. "Avoid caffeine-containing substances for 12 hours before the test." b. "Drink at least 3 liters of fluid during the first 24 hours after the test." c. "Do not take your cardiac medication the morning of the test." d. "Remove your dentures and any metal before the test begins."

A Caffeine-containing liquids and foods are central nervous system stimulants and may alter the test results. No contrast is used; therefore, the client does not need to increase fluid intake. The client should take cardiac medications as prescribed. Metal does not have to be removed; this is done for magnetic resonance imaging.

A nurse assesses a client recovering from a cerebral angiography via the client's right femoral artery. Which assessment should the nurse complete? a. Palpate bilateral lower extremity pulses. b. Obtain orthostatic blood pressure readings. c. Perform a funduscopic examination. d. Assess the gag reflex prior to eating.

A Cerebral angiography is performed by threading a catheter through the femoral or brachial artery. The extremity is kept immobilized after the procedure. The nurse checks the extremity for adequate circulation by noting skin color and temperature, presence and quality of pulses distal to the injection site, and capillary refill. Clients usually are on bedrest; therefore, orthostatic blood pressure readings cannot be performed. The funduscopic examination would not be affected by cerebral angiography. The client is given analgesics but not conscious sedation; therefore, the client's gag reflex would not be compromised.

A nurse delegates care for a client with Parkinson disease to an unlicensed assistive personnel (UAP). Which statement should the nurse include when delegating this client's care? a. "Allow the client to be as independent as possible with activities." b. "Assist the client with frequent and meticulous oral care." c. "Assess the client's ability to eat and swallow before each meal." d. "Schedule appointments early in the morning to ensure rest in the afternoon."

A Clients with Parkinson disease do not move as quickly and can have functional problems. The client should be encouraged to be as independent as possible and provided time to perform activities without rushing. Although oral care is important for all clients, instructing the UAP to provide frequent and meticulous oral is not a priority for this client. This statement would be a priority if the client was immune-compromised or NPO. The nurse should assess the client's ability to eat and swallow; this should not be delegated. Appointments and activities should not be scheduled early in the morning because this may cause the client to be rushed and discourage the client from wanting to participate in activities of daily living.

A client hospitalized with sickle cell crisis frequently asks for opioid pain medications, often shortly after receiving a dose. The nurses on the unit believe the client is drug seeking. When the client requests pain medication, what action by the nurse is best? a. Give the client pain medication if it is time for another dose. b. Instruct the client not to request pain medication too early. c. Request the provider leave a prescription for a placebo. d. Tell the client it is too early to have more pain medication.

A Clients with sickle cell crisis often have severe pain that is managed with up to 48 hours of IV opioid analgesics. Even if the client is addicted and drug seeking, he or she is still in extreme pain. If the client can receive another dose of medication, the nurse should provide it. The other options are judgmental and do not address the client's pain. Giving placebos is unethical.

A nurse delegates care to the unlicensed assistive personnel (UAP). Which statement should the nurse include when delegating care for a client with cranial nerve II impairment? a. "Tell the client where food items are on the breakfast tray." b. "Place the client in a high-Fowler's position for all meals." c. "Make sure the client's food is visually appetizing." d. "Assist the client by placing the fork in the left hand."

A Cranial nerve II, the optic nerve, provides central and peripheral vision. A client who has cranial nerve II impairment will not be able to see, so the UAP should tell the client where different food items are on the meal tray. The other options are not appropriate for a client with cranial nerve II impairment.

A nurse promotes the prevention of lower back pain by teaching clients at a community center. Which instruction should the nurse include in this education? a. "Participate in an exercise program to strengthen muscles." b. "Purchase a mattress that allows you to adjust the firmness." c. "Wear flat instead of high-heeled shoes to work each day." d. "Keep your weight within 20% of your ideal body weight."

A Exercise can strengthen back muscles, reducing the incidence of low back pain. The other options will not prevent low back pain.

The nurse is caring for a client with leukemia who has the priority problem of fatigue. What action by the client best indicates that an important goal for this problem has been met? a. Doing activities of daily living (ADLs) using rest periods b. Helping plan a daily activity schedule c. Requesting a sleeping pill at night d. Telling visitors to leave when fatigued

A Fatigue is a common problem for clients with leukemia. This client is managing his or her own ADLs using rest periods, which indicates an understanding of fatigue and how to control it. Helping to plan an activity schedule is a lesser indicator. Requesting a sleeping pill does not help control fatigue during the day. Asking visitors to leave when tired is another lesser indicator. Managing ADLs using rest periods demonstrates the most comprehensive management strategy.

A nurse performs an assessment of pain discrimination on an older adult client. The client correctly identifies, with eyes closed, a sharp sensation on the right hand when touched with a pin. Which action should the nurse take next? a. Touch the pin on the same area of the left hand. b. Contact the provider with the assessment results. c. Ask the client about current medications. d. Continue the assessment on the client's feet.

A If testing is begun on the right hand and the client correctly identifies the pain stimulus, the nurse should continue the assessment on the left hand. This is a normal finding and does not need to be reported to the provider, but instead documented in the client's chart. Medications do not need to be assessed in response to this finding. The nurse should assess the left hand prior to assessing the feet.

A client has Crohn's disease. What type of anemia is this client most at risk for developing? a. Folic acid deficiency b. Fanconi's anemia c. Hemolytic anemia d. Vitamin B12 anemia

A Malabsorption syndromes such as Crohn's disease leave a client prone to folic acid deficiency. Fanconi's anemia, hemolytic anemia, and vitamin B12 anemia are not related to Crohn's disease.

A nurse obtains a focused health history for a client who is suspected of having bacterial meningitis. Which question should the nurse ask? a. "Do you live in a crowded residence?" b. "When was your last tetanus vaccination?" c. "Have you had any viral infections recently?" d. "Have you traveled out of the country in the last month?"

A Meningococcal meningitis tends to occur in multiple outbreaks. It is most likely to occur in areas of high-density population, such as college dormitories, prisons, and military barracks. A tetanus vaccination would not place the client at increased risk for meningitis or protect the client from meningitis. A viral infection would not lead to bacterial meningitis but could lead to viral meningitis. Simply knowing if the client traveled out of the country does not provide enough information. The nurse should ask about travel to specific countries in which the disease is common, for example, sub-Saharan Africa.

After teaching a client with a spinal cord injury, the nurse assesses the client's understanding. Which client statement indicates a correct understanding of how to prevent respiratory problems at home? a. "I'll use my incentive spirometer every 2 hours while I'm awake." b. "I'll drink thinned fluids to prevent choking." c. "I'll take cough medicine to prevent excessive coughing." d. "I'll position myself on my right side so I don't aspirate."

A Often, the person with a spinal cord injury will have weak intercostal muscles and is at higher risk for developing atelectasis and stasis pneumonia. Using an incentive spirometer every 2 hours helps the client expand the lungs more fully and prevents atelectasis. Clients should drink fluids that they can tolerate; usually thick fluids are easier to tolerate. The client should be encouraged to cough and clear secretions. Clients should be placed in high-Fowler's position to prevent aspiration.

A nurse assesses a client who is recovering from anterior cervical diskectomy and fusion. Which complication should alert the nurse to urgently communicate with the health care provider? a. Auscultated stridor b. Weak pedal pulses c. Difficulty swallowing d. Inability to shrug shoulders

A Postoperative swelling can narrow the trachea, cause a partial airway obstruction, and manifest as stridor. The client may also have trouble swallowing, but maintaining an airway takes priority. Weak pedal pulses and an inability to shrug the shoulders are not complications of this surgery.

A nurse assesses a client and notes the client's position as indicated in the illustration below: How should the nurse document this finding? a. Decorticate posturing b. Decerebrate posturing c. Atypical hyperreflexia d. Spinal cord degeneration

A The client is demonstrating decorticate posturing, which is seen with interruption in the corticospinal pathway. This finding is abnormal and is a sign that the client's condition has deteriorated. The physician, the charge nurse, and other health care team members should be notified immediately of this change in status. Decerebrate posturing consists of external rotation and extension of the extremities. Hyperreflexes present as increased reflex responses. Spinal cord degeneration presents frequently with pain and discomfort.

A nurse cares for a client who is recovering from a single-photon emission computed tomography (SPECT) with a radiopharmaceutical agent. Which statement should the nurse include when discussing the plan of care with this client? a. "You may return to your previous activity level immediately." b. "You are radioactive and must use a private bathroom." c. "Frequent assessments of the injection site will be completed." d. "We will be monitoring your renal functions closely."

A The client may return to his or her previous activity level immediately. Radioisotopes will be eliminated in the urine after SPECT, but no monitoring or special precautions are required. The injection site will not need to be assessed after the procedure is complete.

A nurse is caring for four clients with leukemia. After hand-off report, which client should the nurse see first? a. Client who had two bloody diarrhea stools this morning b. Client who has been premedicated for nausea prior to chemotherapy c. Client with a respiratory rate change from 18 to 22 breaths/min d. Client with an unchanged lesion to the lower right lateral malleolus

A The client who had two bloody diarrhea stools that morning may be hemorrhaging in the gastrointestinal (GI) tract and should be assessed first. The client with the change in respiratory rate may have an infection or worsening anemia and should be seen next. The other two clients are not a priority at this time.

A nurse assesses a client who demonstrates a positive Romberg's sign with eyes closed but not with eyes open. Which condition does the nurse associate with this finding? a. Difficulty with proprioception b. Peripheral motor disorder c. Impaired cerebellar function d. Positive pronator drift

A The client who sways with eyes closed (positive Romberg's sign) but not with eyes open most likely has a disorder of proprioception and uses vision to compensate for it. The other options do not describe a positive Romberg's sign.

A client has been admitted after sustaining a humerus fracture that occurred when picking up the family cat. What test result would the nurse correlate to this condition? a. Bence-Jones protein in urine b. Epstein-Barr virus: positive c. Hemoglobin: 18 mg/dL d. Red blood cell count: 8.2/mm3

A This client has possible multiple myeloma. A positive Bence-Jones protein finding would correlate with this condition. The Epstein-Barr virus is a herpesvirus that causes infectious mononucleosis and some cancers. A hemoglobin of 18 mg/dL is slightly high for a male and somewhat high for a female; this can be caused by several conditions, and further information would be needed to correlate this value with a specific medical condition. A red blood cell count of 8.2/mm3 is also high, but again, more information would be needed to correlate this finding with a specific medical condition.

A nurse assesses the left plantar reflexes of an adult client and notes the response shown in the photograph below: Which action should the nurse take next? a. Contact the provider with this abnormal finding. b. Assess bilateral legs for temperature and edema. c. Ask the client about pain in the lower leg and calf. d. Document the finding and continue the assessment.

A This finding indicates Babinski's sign. In clients older than 2 years of age, Babinski's sign is considered abnormal and indicates central nervous system disease. The nurse should notify the health care provider and other members of the health care team because further investigation is warranted. This finding does not relate to perfusion of the leg or to pain. This is an abnormal assessment finding and should be addressed immediately.

A client has frequent hospitalizations for leukemia and is worried about functioning as a parent to four small children. What action by the nurse would be most helpful? a. Assist the client to make "sick day" plans for household responsibilities. b. Determine if there are family members or friends who can help the client. c. Help the client inform friends and family that they will have to help out. d. Refer the client to a social worker in order to investigate respite child care.

A While all options are reasonable choices, the best option is to help the client make sick day plans, as that is more comprehensive and inclusive than the other options, which focus on a single item.

A client has a platelet count of 9000/mm3. The nurse finds the client confused and mumbling. What action takes priority? a. Calling the Rapid Response Team b. Delegating taking a set of vital signs c. Instituting bleeding precautions d. Placing the client on bedrest

A With a platelet count this low, the client is at high risk of spontaneous bleeding. The most disastrous complication would be intracranial bleeding. The nurse needs to call the Rapid Response Team as this client has manifestations of a sudden neurologic change. The nurse should not delegate the vital signs as the client is no longer stable. Bleeding precautions will not address the immediate situation. Placing the client on bedrest or putting the client back into bed is important, but the critical action is to call for immediate medical attention.

The nurse is assessing a patient for dietary factors that may influence her risk for osteoporosis. The nurse should question the patient about her intake of what nutrients? Select all that apply. A) Calcium B) Simple carbohydrates C) Vitamin D D) Protein E) Soluble fiber

A, C Feedback: A patient's risk for osteoporosis is strongly influenced by vitamin D and calcium intake. Carbohydrate, protein, and fiber intake do not have direct effect on the development of osteoporosis.

During a patient's recovery from stroke, the nurse should be aware of predictors of stroke outcome in order to help patients and families set realistic goals. What are the predictors of stroke outcome? Select all that apply. A) National Institutes of Health Stroke Scale (NIHSS) score B) Race C) LOC at time of admission D) Gender E) Age

A, C, E Feedback: It is helpful for clinicians to be knowledgeable about the relative importance of predictors of stroke outcome (age, NIHSS score, and LOC at time of admission) to provide stroke survivors and their families with realistic goals. Race and gender are not predictors of stroke outcome.

A patient has been admitted to the medical unit for the treatment of Paget's disease. When reviewing the medication administration record, the nurse should anticipate what medications? Select all that apply. A) Calcitonin B) Bisphosphonates C) Alkaline phosphatase D) Calcium gluconate E) Estrogen

A, B Feedback: Bisphosphonates are the cornerstone of Paget therapy in that they stabilize the rapid bone turnover. Calcitonin is also used because it retards bone resorption by decreasing the number and availability of osteoclasts. Alkaline phosphatase is a naturally occurring enzyme, not a drug. Calcium gluconate and estrogen are not used in the treatment of Paget's disease.

A nurse is preparing to administer a blood transfusion to an older adult. Understanding age-related changes, what alterations in the usual protocol are necessary for the nurse to implement? (Select all that apply.) a. Assess vital signs more often. b. Hold other IV fluids running. c. Premedicate to prevent reactions. d. Transfuse smaller bags of blood. e. Transfuse each unit over 8 hours.

A, B The older adult needs vital signs monitored as often as every 15 minutes for the duration of the transfusion because changes may be the only indication of a transfusion-related problem. To prevent fluid overload, the nurse obtains a prescription to hold other running IV fluids during the transfusion. The other options are not warranted.

A patient with spinal cord injury is ready to be discharged home. A family member asks the nurse to review potential complications one more time. What are the potential complications that should be monitored for in this patient? Select all that apply. A) Orthostatic hypotension B) Autonomic dysreflexia C) DVT D) Salt-wasting syndrome E) Increased ICP

A, B, C Feedback: For a spinal cord-injured patient, based on the assessment data, potential complications that may develop include DVT, orthostatic hypotension, and autonomic dysreflexia. Salt-wasting syndrome or increased ICP are not typical complications following the immediate recovery period.

The nurse is admitting a patient to the unit who is scheduled for removal of an intracranial mass. What diagnostic procedures might be included in this patient's admission orders? Select all that apply. A) Transcranial Doppler flow study B) Cerebral angiography C) MRI D) Cranial radiography E) Electromyelography (EMG)

A, B, C Feedback: Preoperative diagnostic procedures may include a CT scan to demonstrate the lesion and show the degree of surrounding brain edema, the ventricular size, and the displacement. An MRI scan provides information similar to that of a CT scan with improved tissue contrast, resolution, and anatomic definition. Cerebral angiography may be used to study a tumor's blood supply or to obtain information about vascular lesions. Transcranial Doppler flow studies are used to evaluate the blood flow within intracranial blood vessels. Regular x-rays of the skull would not be diagnostic for an intracranial mass. An EMG would not be ordered prior to intracranial surgery to remove a mass.

A nurse is explaining a patient's decreasing bone density in terms of the balance between bone resorption and formation. What dietary nutrients and hormones play a role in the resorption and formation of adult bones? Select all that apply. A) Thyroid hormone B) Growth hormone C) Estrogen D) Vitamin B12 E) Luteinizing hormone

A, B, C Feedback: The balance between bone resorption and formation is influenced by the following factors: physical activity; dietary intake of certain nutrients, especially calcium; and several hormones, including calcitriol (i.e., activated vitamin D), parathyroid hormone (PTH), calcitonin, thyroid hormone, cortisol, growth hormone, and the sex hormones estrogen and testosterone. Luteinizing hormone and vitamin B12 do not play a role in bone formation or resorption.

A nurse is caring for a client who is prescribed a computed tomography (CT) scan with iodine-based contrast. Which actions should the nurse take to prepare the client for this procedure? (Select all that apply.) a. Ensure that an informed consent is present. b. Ask the client about any allergies. c. Evaluate the client's renal function. d. Auscultate bilateral breath sounds. e. Assess hematocrit and hemoglobin levels.

A, B, C A client who is scheduled to receive iodine-based contrast should be asked about allergies, especially allergies to iodine or shellfish. The client's kidney function should also be evaluated to determine if it is safe to administer contrast during the procedure. Finally, the nurse should ensure that an informed consent is present because all clients receiving iodine-based contrast must give consent. The CT will have no impact on the client's breath sounds or hematocrit and hemoglobin levels. Findings from these assessments will not influence the client's safety during the procedure.

A nurse assesses clients on a medical-surgical unit. Which clients should the nurse identify as at risk for secondary seizures? (Select all that apply.) a. A 26-year-old woman with a left temporal brain tumor b. A 38-year-old male client in an alcohol withdrawal program c. A 42-year-old football player with a traumatic brain injury d. A 66-year-old female client with multiple sclerosis e. A 72-year-old man with chronic obstructive pulmonary disease

A, B, C Clients at risk for secondary seizures include those with a brain lesion from a tumor or trauma, and those who are experiencing a metabolic disorder, acute alcohol withdrawal, electrolyte disturbances, and high fever. Clients with a history of stroke, heart disease, and substance abuse are also at risk. Clients with multiple sclerosis or chronic obstructive pulmonary disease are not at risk for secondary seizures.

A nurse is providing care for a patient who has a recent diagnosis of Paget's disease. When planning this patient's nursing care, interventions should address what nursing diagnoses? Select all that apply. A) Impaired Physical Mobility B) Acute Pain C) Disturbed Auditory Sensory Perception D) Risk for Injury E) Risk for Unstable Blood Glucose

A, B, C, D Feedback: Patient's with Paget's disease are at risk of decreased mobility, pain, hearing loss, and injuries resulting from decreased bone density. Paget's disease does not affect blood glucose levels.

An adult patient has sought care for the treatment of headaches that have become increasingly severe and frequent over the past several months. Which of the following questions addresses potential etiological factors? Select all that apply? A) "Are you exposed to any toxins or chemicals at work?" B) "How would you describe your ability to cope with stress?" C) "What medications are you currently taking?" D) "When was the last time you were hospitalized?" E) "Does anyone else in your family struggle with headaches?"

A, B, C, E Feedback: Headaches are multifactorial, and may involve medications, exposure to toxins, family history, and stress. Hospitalization is an unlikely contributor to headaches.

A nurse is planning the care of an older adult patient who will soon be discharged home after treatment for a fractured hip. In an effort to prevent future fractures, the nurse should encourage which of the following? Select all that apply. A) Regular bone density testing B) A high-calcium diet C) Use of falls prevention precautions D) Use of corticosteroids as ordered E) Weight-bearing exercise

A, B, C, E Feedback: Health promotion measures after an older adult's hip fracture include weight-bearing exercise, promotion of a healthy diet, falls prevention, and bone density testing. Corticosteroids have the potential to reduce bone density and increase the risk for fractures.

An older adult patient experienced a fall and required treatment for a fractured hip on the orthopedic unit. Which of the following are contributory factors to the incidence of falls and fractured hips among the older adult population? Select all that apply. A) Loss of visual acuity B) Adverse medication effects C) Slowed reflexes D) Hearing loss E) Muscle weakness

A, B, C, E Feedback: Older adults are generally vulnerable to falls and have a high incidence of hip fracture. Weak quadriceps muscles, medication effects, vision loss, and slowed reflexes are among the factors that contribute to the incidence of falls. Decreased hearing is not noted to contribute to the incidence of falls.

A student nurse is helping a registered nurse with a blood transfusion. Which actions by the student are most appropriate? (Select all that apply.) a. Hanging the blood product using normal saline and a filtered tubing set b. Taking a full set of vital signs prior to starting the blood transfusion c. Telling the client someone will remain at the bedside for the first 5 minutes d. Using gloves to start the client's IV if needed and to handle the blood product e. Verifying the client's identity, and checking blood compatibility and expiration time

A, B, D Correct actions prior to beginning a blood transfusion include hanging the product with saline and the correct filtered blood tubing, taking a full set of vital signs prior to starting, and using gloves. Someone stays with the client for the first 15 to 30 minutes of the transfusion. Two registered nurses must verify the client's identity and blood compatibility.

A nurse delegates care for an older adult client to the unlicensed assistive personnel (UAP). Which statements should the nurse include when delegating this client's care? (Select all that apply.) a. "Plan to bathe the client in the evening when the client is most alert." b. "Encourage the client to use a cane when ambulating." c. "Assess the client for symptoms related to pain and discomfort." d. "Remind the client to look at foot placement when walking." e. "Schedule additional time for teaching about prescribed therapies."

A, B, D The nurse should tell the UAP to schedule activities when the client is normally awake, encourage the client to use a cane when ambulating, and remind the client to look where feet are placed when walking. The nurse should assess the client for symptoms of pain and should provide sufficient time for older adults to process information, including new teaching. These are not items the nurse can delegate.

A client has heparin-induced thrombocytopenia (HIT). The student nurse asks how this is treated. About what drugs does the nurse instructor teach? (Select all that apply.) a. Argatroban (Argatroban) b. Bivalirudin (Angiomax) c. Clopidogrel (Plavix) d. Lepirudin (Refludan) e. Methylprednisolone (Solu-Medrol)

A, B, D The standard drugs used to treat HIT are argatroban, bivalirudin, and lepirudin. The other drugs are not used. Clopidogrel is an antiplatelet agent used to reduce the likelihood of stroke or myocardial infarction. Methylprednisolone is a steroid used to reduce inflammation.

A nurse plans care for a client with a halo fixator. Which interventions should the nurse include in this client's plan of care? (Select all that apply.) a. Tape a halo wrench to the client's vest. b. Assess the pin sites for signs of infection. c. Loosen the pins when sleeping. d. Decrease the client's oral fluid intake. e. Assess the chest and back for skin breakdown.

A, B, E A special halo wrench should be taped to the client's vest in case of a cardiopulmonary emergency. The nurse should assess the pin sites for signs of infection or loose pins and for complications from the halo. The nurse should also increase fluids and fiber to decrease bowel straining and assess the client's chest and back for skin breakdown from the halo vest.

After teaching a client with a spinal cord tumor, the nurse assesses the client's understanding. Which statements by the client indicate a correct understanding of the teaching? (Select all that apply.) a. "Even though turning hurts, I will remind you to turn me every 2 hours." b. "Radiation therapy can shrink the tumor but also can cause more problems." c. "Surgery will be scheduled to remove the tumor and reverse my symptoms." d. "I put my affairs in order because this type of cancer is almost always fatal." e. "My family is moving my bedroom downstairs for when I am discharged home."

A, B, E Although surgery may relieve symptoms by reducing pressure on the spine and debulking the tumor, some motor and sensory deficits may remain. Spinal tumors usually cause disability but are not usually fatal. Radiation therapy is often used to shrink spinal tumors but can cause progressive spinal cord degeneration and neurologic deficits. The client should be turned every 2 hours to prevent skin breakdown and arrangements should be made at home so that the client can complete activities of daily living without needing to go up and down stairs.

A nurse assesses a client who is experiencing a cluster headache. Which clinical manifestations should the nurse expect to find? (Select all that apply.) a. Ipsilateral tearing of the eye b. Miosis c. Abrupt loss of consciousness d. Neck and shoulder tenderness e. Nasal congestion f. Exophthalmos

A, B, E Cluster headache is usually accompanied by ipsilateral tearing, miosis, rhinorrhea or nasal congestion, ptosis, eyelid edema, and facial sweating. Abrupt loss of consciousness, neck and shoulder tenderness, and exophthalmos are not associated with cluster headaches.

A nurse assesses a client with a brain tumor. Which newly identified assessment findings should alert the nurse to urgently communicate with the health care provider? (Select all that apply.) a. Glasgow Coma Scale score of 8 b. Decerebrate posturing c. Reactive pupils d. Uninhibited speech e. Diminished cognition

A, B, E The nurse should urgently communicate changes in a client's neurologic status, including a decrease in the Glasgow Coma Scale score, abnormal flexion or extension, changes in cognition or speech, and pinpointed, dilated, and nonreactive pupils.

A nurse is caring for a client with meningitis. Which laboratory values should the nurse monitor to identify potential complications of this disorder? (Select all that apply.) a. Sodium level b. Liver enzymes c. Clotting factors d. Cardiac enzymes e. Creatinine level

A, C Inflammation associated with meningitis can stimulate the hypothalamus and result in excessive production of antidiuretic hormone. The nurse should monitor sodium levels for early identification of syndrome of inappropriate antidiuretic hormone. A systemic inflammatory response (SIR) can also occur with meningitis. A SIR can result in a coagulopathy that leads to disseminated intravascular coagulation. The nurse should monitor clotting factors to identify this complication. The other laboratory values are not specific to complications of meningitis.

A patient has had a brace prescribed to facilitate recovery from a knee injury. What are the potential therapeutic benefits of a brace? Select all that apply. A) Preventing additional injury B) Immobilizing prior to surgery C) Providing support D) Controlling movement E) Promoting bone remodeling

A, C, D Feedback: Braces (i.e., orthoses) are used to provide support, control movement, and prevent additional injury. They are not used to immobilize body parts or to facilitate bone remodeling.

A nurse evaluates the results of diagnostic tests on a client's cerebrospinal fluid (CSF). Which fluid results alerts the nurse to possible viral meningitis? (Select all that apply.) a. Clear b. Cloudy c. Increased protein level d. Normal glucose level e. Bacterial organisms present f. Increased white blood cells

A, C, D In viral meningitis, CSF fluid is clear, protein levels are slightly increased, and glucose levels are normal. Viral meningitis does not cause cloudiness or increased turbidity of CSF. In bacterial meningitis, the presence of bacteria and white blood cells causes the fluid to be cloudy.

A nurse assesses a client who experienced a spinal cord injury at the T5 level 12 hours ago. Which manifestations should the nurse correlate with neurogenic shock? (Select all that apply.) a. Heart rate of 34 beats/min b. Blood pressure of 185/65 mm Hg c. Urine output less than 30 mL/hr d. Decreased level of consciousness e. Increased oxygen saturation

A, C, D Neurogenic shock with acute spinal cord injury manifests with decreased oxygen saturation, symptomatic bradycardia, decreased level of consciousness, decreased urine output, and hypotension.

2. An emergency department nurse assesses a client who was struck in the temporal lobe with a baseball. For which clinical manifestations that are related to a temporal lobe injury should the nurse assess? (Select all that apply.) a. Memory loss b. Personality changes c. Difficulty with sound interpretation d. Speech difficulties e. Impaired taste

A, C, D Wernicke's area (language area) is located in the temporal lobe and enables the processing of words into coherent thought as well as the understanding of written or spoken words. The temporal lobe also is responsible for the auditory center's interpretation of sound and complicated memory patterns. Personality changes are related to frontal lobe injury. Impaired taste is associated with injury to the parietal lobe.

A nurse is caring for a patient who is being assessed following complaints of severe and persistent low back pain. The patient is scheduled for diagnostic testing in the morning. Which of the following are appropriate diagnostic tests for assessing low back pain? that apply. A) Computed tomography (CT) B) Angiography C) Magnetic resonance imaging (MRI) D) Ultrasound E) X-ray

A, C, D, E Feedback: A variety of diagnostic tests can be used to address lower back pain, including CT, MRI, ultrasound, and X-rays. Angiography is not related to the etiology of back pain.

The nurse caring for a patient in a persistent vegetative state is regularly assessing for potential complications. Complications of neurologic dysfunction for which the nurse should assess include which of the following? Select all that apply. A) Contractures B) Hemorrhage C) Pressure ulcers D) Venous thromboembolism E) Pneumonia

A, C, D, E Feedback: Based on the assessment data, potential complications may include respiratory distress or failure, pneumonia, aspiration, pressure ulcer, deep vein thrombosis (DVT), and contractures. The pathophysiology of decreased LOC does not normally create a heightened risk for hemorrhage.

A nurse working with clients with sickle cell disease (SCD) teaches about self-management to prevent exacerbations and sickle cell crises. What factors should clients be taught to avoid? (Select all that apply.) a. Dehydration b. Exercise c. Extreme stress d. High altitudes e. Pregnancy

A, C, D, E Several factors cause red blood cells to sickle in SCD, including dehydration, extreme stress, high altitudes, and pregnancy. Strenuous exercise can also cause sickling, but not unless it is very vigorous.

A nurse assesses a client who recently experienced a traumatic spinal cord injury. Which assessment data should the nurse obtain to assess the client's coping strategies? (Select all that apply.) a. Spiritual beliefs b. Level of pain c. Family support d. Level of independence e. Annual income f. Previous coping strategies

A, C, D, F Information about the client's preinjury psychosocial status, usual methods of coping with illness, difficult situations, and disappointments should be obtained. Determine the client's level of independence or dependence and his or her comfort level in discussing feelings and emotions with family members or close friends. Clients who are emotionally secure and have a positive self-image, a supportive family, and financial and job security often adapt to their injury. Information about the client's spiritual and religious beliefs or cultural background also assists the nurse in developing the plan of care. The other options do not supply as much information about coping.

A public health nurse is teaching a health promotion workshop that focuses on vision and eye health. What should this nurse cite as the most common causes of blindness and visual impairment among adults over the age of 40? Select all that apply. A) Diabetic retinopathy B) Trauma C) Macular degeneration D) Cytomegalovirus E) Glaucoma

A, C, E Feedback: The most common causes of blindness and visual impairment among adults 40 years of age or older are diabetic retinopathy, macular degeneration, glaucoma, and cataracts. Therefore, trauma and cytomegalovirus are incorrect.

A student studying leukemias learns the risk factors for developing this disorder. Which risk factors does this include? (Select all that apply.) a. Chemical exposure b. Genetically modified foods c. Ionizing radiation exposure d. Vaccinations e. Viral infections

A, C, E Chemical and ionizing radiation exposure and viral infections are known risk factors for developing leukemia. Eating genetically modified food and receiving vaccinations are not known risk factors.

A student nurse is learning about blood transfusion compatibilities. What information does this include? (Select all that apply.) a. Donor blood type A can donate to recipient blood type AB. b. Donor blood type B can donate to recipient blood type O. c. Donor blood type AB can donate to anyone. d. Donor blood type O can donate to anyone. e. Donor blood type A can donate to recipient blood type B.

A, D Blood type A can be donated to people who have blood types A or AB. Blood type O can be given to anyone. Blood type B can be donated to people who have blood types B or AB. Blood type AB can only go to recipients with blood type AB.

The school nurse is giving a presentation on preventing spinal cord injuries (SCI). What should the nurse identify as prominent risk factors for SCI? Select all that apply. A) Young age B) Frequent travel C) African American race D) Male gender E) Alcohol or drug use

A, D, E Feedback: The predominant risk factors for SCI include young age, male gender, and alcohol and drug use. Ethnicity and travel are not risk factors.

A nurse plans care for a client with epilepsy who is admitted to the hospital. Which interventions should the nurse include in this client's plan of care? (Select all that apply.) a. Have suction equipment at the bedside. b. Place a padded tongue blade at the bedside. c. Permit only clear oral fluids. d. Keep bed rails up at all times. e. Maintain the client on strict bedrest. f. Ensure that the client has IV access.

A, D, F Oxygen and suctioning equipment with an airway must be readily available. The bed rails should be up at all times while the client is in the bed to prevent injury from a fall if the client has a seizure. If the client does not have an IV access, insert a saline lock, especially for those clients who are at significant risk for generalized tonic-clonic seizures. The saline lock provides ready access if IV drug therapy must be given to stop the seizure. Padded tongue blades may pose a danger to the client during a seizure and should not be used. Dietary restrictions and strict bedrest are not interventions associated with epilepsy. The client should be encouraged to eat a well-balanced diet and ambulate while in the hospital.

After teaching a male client with a spinal cord injury at the T4 level, the nurse assesses the client's understanding. Which client statements indicate a correct understanding of the teaching related to sexual effects of this injury? (Select all that apply.) a. "I will explore other ways besides intercourse to please my partner." b. "I will not be able to have an erection because of my injury." c. "Ejaculation may not be as predictable as before." d. "I may urinate with ejaculation but this will not cause infection." e. "I should be able to have an erection with stimulation."

ANS: C, D, E Men with injuries above T6 often are able to have erections by stimulating reflex activity. For example, stroking the penis will cause an erection. Ejaculation is less predictable and may be mixed with urine. However, urine is sterile, so the client's partner will not get an infection.

A nurse assesses a client with paraplegia from a spinal cord injury and notes reddened areas over the client's hips and sacrum. Which actions should the nurse take? (Select all that apply.) a. Apply a barrier cream to protect the skin from excoriation. b. Perform range-of-motion (ROM) exercises for the hip joint. c. Re-position the client off of the reddened areas. d. Get the client out of bed and into a chair once a day. e. Obtain a low-air-loss mattress to minimize pressure.

ANS: C, E Appropriate interventions to relieve pressure on these areas include frequent re-positioning and a low-air-loss mattress. Reddened areas should not be rubbed because this action could cause more extensive damage to the already fragile capillary system. Barrier cream will not protect the skin from pressure wounds. ROM exercises are used to prevent contractures. Sitting the client in a chair once a day will decrease the client's risk of respiratory complications but will not decrease pressure on the client's hips and sacrum.

A nurse is caring for a patient who has just had an arthroscopy as an outpatient and is getting ready to go home. The nurse should teach the patient to monitor closely for what postprocedure complication? A) Fever B) Crepitus C) Fasciculations D) Synovial fluid leakage

Ans: A Feedback: Following arthroscopy, the patient and family are informed of complications to watch for, including fever. Synovial fluid leakage is unlikely and crepitus would not develop as a postprocedure complication. Fasciculations are muscle twitches and do not involve joint integrity or function.

The surgical nurse is admitting a patient from postanesthetic recovery following the patient's below-the-knee amputation. The nurse recognizes the patient's high risk for postoperative hemorrhage and should keep which of the following at the bedside? A) A tourniquet B) A syringe preloaded with vitamin K C) A unit of packed red blood cells, placed on ice D) A dose of protamine sulfate

Ans: A Feedback: Immediate postoperative bleeding may develop slowly or may take the form of massive hemorrhage resulting from a loosened suture. A large tourniquet should be in plain sight at the patient's bedside so that, if severe bleeding occurs, it can be applied to the residual limb to control the hemorrhage. PRBCs cannot be kept at the bedside. Vitamin K and protamine sulfate are antidotes to warfarin and heparin, but are not administered to treat active postsurgical bleeding.

A 25-year-old man is involved in a motorcycle accident and injures his arm. The physician diagnoses the man with an intra-articular fracture and splints the injury. The nurse implements the teaching plan developed for this patient. What sequela of intra-articular fractures should the nurse describe regarding this patient? A) Post-traumatic arthritis B) Fat embolism syndrome (FES) C) Osteomyelitis D) Compartment syndrome

Ans: A Feedback: Intra-articular fractures often lead to post-traumatic arthritis. Research does not indicate a correlation between intra-articular fractures and FES, osteomyelitis, or compartment syndrome.

A nurse is assessing a patient who reports a throbbing, burning sensation in the right foot. The patient states that the pain is worst during the day but notes that the pain is relieved with rest. The nurse should recognize the signs and symptoms of what health problem? A) Morton's neuroma B) Pescavus C) Hallux valgus D) Onychocryptosis

Ans: A Feedback: Morton's neuroma is a swelling of the third (lateral) branch of the median plantar nerve, which causes a throbbing, burning pain, usually relieved with rest. Pescavus refers to a foot with an abnormally high arch and a fixed equinus deformity of the forefoot. Hallux valgus (bunion) is a deformity in which the great toe deviates laterally and there is a marked prominence of the medial aspect of the first metatarsal-phalangeal joint and exostosis. Onychocryptosis (ingrown toenail) occurs when the free edge of a nail plate penetrates the surrounding skin, laterally or anteriorly.

A nurse is admitting a patient to the unit who presented with a lower extremity fracture. What signs and symptoms would suggest to the nurse that the patient may have aperoneal nerve injury? A) Numbness and burning of the foot B) Pallor to the dorsal surface of the foot C) Visible cyanosis in the toes D) Inadequate capillary refill to the toes

Ans: A Feedback: Peroneal nerve injury may result in numbness, tingling, and burning in the feet. Cyanosis, pallor, and decreased capillary refill are signs of inadequate circulation.

A nurse is taking a health history on a patient with musculoskeletal dysfunction. What is the primary focus of this phase of the nurse's assessment? A) Evaluating the effects of the musculoskeletal disorder on the patient's function B) Evaluating the patient's adherence to the existing treatment regimen C) Evaluating the presence of genetic risk factors for further musculoskeletal disorders D) Evaluating the patient's active and passive range of motion

Ans: A Feedback: The nursing assessment of the patient with musculoskeletal dysfunction includes an evaluation of the effects of the musculoskeletal disorder on the patient. This is a vital focus of the health history and supersedes the assessment of genetic risk factors and adherence to treatment, though these are both valid inclusions to the interview. Assessment of ROM occurs during the physical assessment, not the interview.

A patient is receiving ongoing nursing care for the treatment of Parkinson's disease. When assessing this patient's gait, what finding is most closely associated with this health problem? A) Spastic hemiparesis gait B) Shuffling gait C) Rapid gait D) Steppage gait

Ans: B Feedback: A variety of neurologic conditions are associated with abnormal gaits, such as a spastic hemiparesis gait (stroke), steppage gait (lower motor neuron disease), and shuffling gait (Parkinson's disease). A rapid gait is not associated with Parkinson's disease.

The orthopedic surgeon has prescribed balanced skeletal traction for a patient. What advantage is conferred by balanced traction? A) Balanced traction can be applied at night and removed during the day. B) Balanced traction allows for greater patient movement and independence than other forms of traction. C) Balanced traction is portable and may accompany the patient's movements. D) Balanced traction facilitates bone remodeling in as little as 4 days.

Ans: B Feedback: Often, skeletal traction is balanced traction, which supports the affected extremity, allows for some patient movement, and facilitates patient independence and nursing care while maintaining effective traction. It is not portable, however, and it cannot be removed. Bone remodeling takes longer than 4 days.

A patient scheduled for magnetic resonance imaging (MRI) has arrived at the radiology department. The nurse who prepares the patient for the MRI should prioritize which of the following actions? A) Withholding stimulants 24 to 48 hours prior to exam B) Removing all metal-containing objects C) Instructing the patient to void prior to the MRI D) Initiating an IV line for administration of contrast

Ans: B Feedback: Patient preparation for an MRI consists of removing all metal-containing objects prior to the examination. Withholding stimulants would not affect an MRI; this relates to an electroencephalography (EEG). Instructing the patient to void is patient preparation for a lumbar puncture. Initiating an IV line for administration of contrast would be done if the patient was having a CT scan with contrast.

Which of the following is the most appropriate nursing intervention to facilitate healing in a patient who has suffered a hip fracture? A) Administer analgesics as required. B) Place a pillow between the patient's legs when turning. C) Maintain prone positioning at all times. D) Encourage internal and external rotation of the affected leg.

Ans: B Feedback: Placing a pillow between the patient's legs when turning prevents adduction and supports the patient's legs. Administering analgesics addresses pain but does not directly protect bone remodeling and promote healing. Rotation of the affected leg can cause dislocation and must be avoided. Prone positioning does not need to be maintained at all times.

The orthopedic nurse should assess for signs and symptoms of Volkmann's contracture if a patient has fractured which of the following bones? A) Femur B) Humerus C) Radial head D) Clavicle

Ans: B Feedback: The most serious complication of a supracondylar fracture of the humerus is Volkmann's ischemic contracture, which results from antecubital swelling or damage to the brachial artery. This complication is specific to humeral fractures.

A nurse is caring for a patient who is in skeletal traction. To prevent the complication of skin breakdown in a patient with skeletal traction, what action should be included in the plan of care? A) Apply occlusive dressings to the pin sites. B) Encourage the patient to push up with the elbows when repositioning. C) Encourage the patient to perform isometric exercises once a shift. D) Assess the pin insertion site every 8 hours.

Ans: D Feedback: The pin insertion site should be assessed every 8 hours for inflammation and infection. Loose cover dressings should be applied to pin sites. The patient should be encouraged to use the overhead trapeze to shift weight for repositioning. Isometric exercises should be done 10 times an hour while awake.

The nurse is participating in the care of a patient with increased ICP. What diagnostic test is contraindicated in this patient's treatment? A) Computed tomography (CT) scan B) Lumbar puncture C) Magnetic resonance imaging (MRI) D) Venous Doppler studies

B Feedback: A lumbar puncture in a patient with increased ICP may cause the brain to herniate from the withdrawal of fluid and change in pressure during the lumbar puncture. Herniation of the brain is a dire and frequently fatal event. CT, MRI, and venous Doppler are considered noninvasive procedures and they would not affect the ICP itself.

A nurse is caring for a patient who experiences debilitating cluster headaches. The patient should be taught to take appropriate medications at what point in the course of the onset of a new headache? A) As soon as the patient's pain becomes unbearable B) As soon as the patient senses the onset of symptoms C) Twenty to 30 minutes after the onset of symptoms D) When the patient senses his or her symptoms peaking

B Feedback: A migraine or a cluster headache in the early phase requires abortive medication therapy instituted as soon as possible. Delaying medication administration would lead to unnecessary pain.

3. A nurse's assessment of a patient's knee reveals edema, tenderness, muscle spasms, and ecchymosis. The patient states that 2 days ago he ran 10 miles and now it really hurts to stand up. The nurse should plan care based on the belief that the patient has experienced what? A) A first-degree strain B) A second-degree strain C) A first-degree sprain D) A second-degree sprain

B Feedback: A second-degree strain involves tearing of muscle fibers and is manifested by notable loss of load-bearing strength with accompanying edema, tenderness, muscle spasm, and ecchymosis. A first-degree strain reflects tearing of a few muscle fibers and is accompanied by minor edema, tenderness, and mild muscle spasm, without noticeable loss of function. However, this patient states a loss of function. A sprain normally involves twisting, which is inconsistent with the patient's overuse injury.

The nurse is caring for a patient who exhibits abnormal results of the Weber test and Rinne test. The nurse should suspect dysfunction involving what cranial nerve? A) Trigeminal B) Acoustic C) Hypoglossal D) Trochlear

B Feedback: Abnormal hearing can correlate with damage to cranial nerve VIII (acoustic). The acoustic nerve functions in hearing and equilibrium. The trigeminal nerve functions in facial sensation, corneal reflex, and chewing. The hypoglossal nerve moves the tongue. The trochlear nerve controls muscles that move the eye.

A physician writes an order to discontinue skeletal traction on an orthopedic patient. The nurse should anticipate what subsequent intervention? A) Application of a walking boot B) Application of a cast C) Education on how to use crutches D) Passive range of motion exercises

B Feedback: After skeletal traction is discontinued, internal fixation, casts, or splints are then used to immobilize and support the healing bone. The use of a walking boot, crutches, or ROM exercises could easily damage delicate, remodeled bone.

A clinic nurse is caring for a patient diagnosed with migraine headaches. During the patient teaching session, the patient questions the nurse regarding alcohol consumption. What would the nurse be correct in telling the patient about the effects of alcohol? A) Alcohol causes hormone fluctuations. B) Alcohol causes vasodilation of the blood vessels. C) Alcohol has an excitatory effect on the CNS. D) Alcohol diminishes endorphins in the brain.

B Feedback: Alcohol causes vasodilation of the blood vessels and may exacerbate migraine headaches. Alcohol has a depressant effect on the CNS. Alcohol does not cause hormone fluctuations, nor does it decrease endorphins (morphine-like substances produced by the body) in the brain.

A hospitalized patient with impaired vision must get a picture in his or her mind of the hospital room and its contents in order to mobilize independently and safely. What must the nurse monitor in the patient's room? A) That a commode is always available at the bedside B) That all furniture remains in the same position C) That visitors do not leave items on the bedside table D) That the patient's slippers stay under the bed

B Feedback: All articles and furniture must remain in the same positions throughout the patient's hospitalization. This will reduce the patient's risks for falls. Visual impairment does not necessarily indicate a need for a commode. Keeping slippers under the bed and keeping the bedside table clear are also appropriate, but preventing falls by maintaining the room arrangement is a priority.

When caring for a patient who had a hemorrhagic stroke, close monitoring of vital signs and neurologic changes is imperative. What is the earliest sign of deterioration in a patient with a hemorrhagic stroke of which the nurse should be aware? A) Generalized pain B) Alteration in level of consciousness (LOC) C) Tonicclonic seizures D) Shortness of breath

B Feedback: Alteration in LOC is the earliest sign of deterioration in a patient after a hemorrhagic stroke, such as mild drowsiness, slight slurring of speech, and sluggish papillary reaction. Sudden headache may occur, but generalized pain is less common. Seizures and shortness of breath are not identified as early signs of hemorrhagic stroke.

An elderly woman found with a head injury on the floor of her home is subsequently admitted to the neurologic ICU. What is the best rationale for the following physician orders: elevate the HOB; keep the head in neutral alignment with no neck flexion or head rotation; avoid sharp hip flexion? A) To decrease cerebral arterial pressure B) To avoid impeding venous outflow C) To prevent flexion contractures D) To prevent aspiration of stomach contents

B Feedback: Any activity or position that impedes venous outflow from the head may contribute to increased volume inside the skull and possibly increase ICP. Cerebral arterial pressure will be affected by the balance between oxygen and carbon dioxide. Flexion contractures are not a priority at this time. Stomach contents could still be aspirated in this position.

A nurse is caring for a critically ill patient with autonomic dysreflexia. What clinical manifestations would the nurse expect in this patient? A) Respiratory distress and projectile vomiting B) Bradycardia and hypertension C) Tachycardia and agitation D) Third-spacing and hyperthermia

B Feedback: Autonomic dysreflexia is characterized by a pounding headache, profuse sweating, nasal congestion, piloerection ("goose bumps"), bradycardia, and hypertension. It occurs in cord lesions above T6 after spinal shock has resolved; it does not result in vomiting, tachycardia, or third-spacing.

The nurse is caring for a patient recovering from an ischemic stroke. What intervention best addresses a potential complication after an ischemic stroke? A) Providing frequent small meals rather than three larger meals B) Teaching the patient to perform deep breathing and coughing exercises C) Keeping a urinary catheter in situ for the full duration of recovery D) Limiting intake of insoluble fiber

B Feedback: Because pneumonia is a potential complication of stroke, deep breathing and coughing exercises should be encouraged unless contraindicated. No particular need exists to provide frequent meals and normally fiber intake should not be restricted. Urinary catheters should be discontinued as soon as possible.

Ans: A Feedback: The nursing assessment of the patient with musculoskeletal dysfunction includes an evaluation of the effects of the musculoskeletal disorder on the patient. This is a vital focus of the health history and supersedes the assessment of genetic risk factors and adherence to treatment, though these are both valid inclusions to the interview. Assessment of ROM occurs during the physical assessment, not the interview.

B Feedback: Before the scan, the nurse asks the patient to empty the bladder, because a full bladder interferes with accurate scanning of the pelvic bones. Bowel cleansing and fasting are not indicated for a bone scan and an allergy to penicillins is not a contraindication.

A nurse is reviewing a patient's activities of daily living prior to discharge from total hip replacement. The nurse should identify what activity as posing a potential risk for hip dislocation? A) Straining during a bowel movement B) Bending down to put on socks C) Lifting items above shoulder level D) Transferring from a sitting to standing position

B Feedback: Bending to put on socks or shoes can cause hip dislocation. None of the other listed actions poses a serious threat to the integrity of the new hip.

A bone biopsy has just been completed on a patient with suspected bone metastases. What assessment should the nurse prioritize in the immediate recovery period? A) Assessment for dehiscence at the biopsy site B) Assessment for pain C) Assessment for hematoma formation D) Assessment for infection

B Feedback: Bone biopsy can be painful and the nurse should prioritize relevant assessments. Dehiscence is not a possibility, since the incision is not linear. Signs and symptoms of infection would not be evident in the immediate recovery period and hematoma formation is not a common complication.

A preceptor is discussing stroke with a new nurse on the unit. The preceptor would tell the new nurse which cardiac dysrhythmia is associated with cardiogenic embolic strokes? A) Ventricular tachycardia B) Atrial fibrillation C) Supraventricular tachycardia D) Bundle branch block

B Feedback: Cardiogenic embolic strokes are associated with cardiac dysrhythmias, usually atrial fibrillation. The other listed dysrhythmias are less commonly associated with this type of stroke.

A patient has been admitted to the ICU after being recently diagnosed with an aneurysm and the patient's admission orders include specific aneurysm precautions. What nursing action will the nurse incorporate into the patient's plan of care? A) Elevate the head of the bed to 45 degrees. B) Maintain the patient on complete bed rest. C) Administer enemas when the patient is constipated. D) Avoid use of thigh-high elastic compression stockings.

B Feedback: Cerebral aneurysm precautions are implemented for the patient with a diagnosis of aneurysm to provide a nonstimulating environment, prevent increases in ICP, and prevent further bleeding. The patient is placed on immediate and absolute bed rest in a quiet, nonstressful environment because activity, pain, and anxiety elevate BP, which increases the risk for bleeding. Visitors, except for family, are restricted. The head of the bed is elevated 15 to 30 degrees to promote venous drainage and decrease ICP. Some neurologists, however, prefer that the patient remains flat to increase cerebral perfusion. No enemas are permitted, but stool softeners and mild laxatives are prescribed. Thigh-high elastic compression stockings or sequential compression boots may be ordered to decrease the patient's risk for deep vein thrombosis (DVT).

A patient presents at the ED after receiving a chemical burn to the eye. What would be the nurse's initial intervention for this patient? A) Generously flush the affected eye with a dilute antibiotic solution. B) Generously flush the affected eye with normal saline or water. C) Apply a patch to the affected eye. D) Apply direct pressure to the affected eye.

B Feedback: Chemical burns of the eye should be immediately irrigated with water or normal saline to flush the chemical from the eye. Antibiotic solutions, lubricant drops, and other prescription drops may be prescribed at a later time. Application of direct pressure may extend the damage to the eye tissue and should be avoided. Patching will be incorporated into the treatment plan at a later time to assist with the process of re-epithelialization, but at this point in the care of the patient, patching will prevent irrigation of the eye.

The nurse on the medicalsurgical unit is reviewing discharge instructions with a patient who has a history of glaucoma. The nurse should anticipate the use of what medications? A) Potassium-sparing diuretics B) Cholinergics C) Antibiotics D) Loop diuretics

B Feedback: Cholinergics are used in the treatment of glaucoma. The action of this medication is to increase aqueous fluid outflow by contracting the ciliary muscle and causing miosis and opening the trabecular meshwork. Diuretics and antibiotics are not used in the management of glaucoma.

A patient has been admitted to the hospital with a spontaneous vertebral fracture related to osteoporosis. Which of the following nursing diagnoses must be addressed in the plan of care? A) Risk for Aspiration Related to Vertebral Fracture B) Constipation Related to Vertebral Fracture C) Impaired Swallowing Related to Vertebral Fracture D) Decreased Cardiac Output Related to Vertebral Fracture

B Feedback: Constipation is a problem related to immobility and medications used to treat vertebral fractures. The patient's risks of aspiration, dysphagia, and decreased cardiac output are not necessarily heightened.

A patient has returned to the unit after undergoing limb-sparing surgery to remove a metastatic bone tumor. The nurse providing postoperative care in the days following surgery assesses for what complication from surgery? A) Deficient fluid volume B) Delayed wound healing C) Hypocalcemia D) Pathologic fractures

B Feedback: Delayed wound healing is a complication of surgery due to tissue trauma from the surgery. Nutritional deficiency is usually due to the effects of chemotherapy and radiation therapy, which may cause weight loss. Pathologic fractures are not a complication of surgery.

A patient with diabetes is attending a class on the prevention of associated diseases. What action should the patient perform to reduce the risk of osteomyelitis? A) Increase calcium and vitamin intake. B) Perform meticulous foot care. C) Exercise 3 to 4 times weekly for at least 30 minutes. D) Take corticosteroids as ordered.

B Feedback: Diabetic foot ulcers have a high potential for progressing to osteomyelitis. Meticulous foot care can help mitigate this risk. Corticosteroids can exacerbate the risk of osteomyelitis. Increased intake of calcium and vitamins as well as regular exercise are beneficial health promotion exercises, but they do not directly reduce the risk of osteomyelitis.

The nurse is doing an initial assessment on a patient newly admitted to the unit with a diagnosis of cerebrovascular accident (CVA). The patient has difficulty copying a figure that the nurse has drawn and is diagnosed with visual-receptive aphasia. What brain region is primarily involved in this deficit? A) Temporal lobe B) Parietal-occipital area C) Inferior posterior frontal areas D) Posterior frontal area

B Feedback: Difficulty copying a figure that the nurse has drawn would be considered visual-receptive aphasia, which involves the parietal-occipital area. Expressive aphasia, the inability to express oneself, is often associated with damage to the frontal area. Receptive aphasia, the inability to understand what someone else is saying, is often associated with damage to the temporal lobe area.

A patient has recently been admitted to the orthopedic unit following total hip arthroplasty. The patient has a closed suction device in place and the nurse has determined that there were 320 mL of output in the first 24 hours. How should the nurse best respond to this assessment finding? A) Inform the primary care provider promptly. B) Document this as an expected assessment finding. C) Limit the patient's fluid intake to 2 liters for the next 24 hours. D) Administer a loop diuretic as ordered.

B Feedback: Drainage of 200 to 500 mL in the first 24 hours is expected. Consequently, the nurse does not need to inform the physician. Fluid restriction and medication administration are not indicated.

A patient has recently begun mobilizing during the recovery from an ischemic stroke. To protect the patient's safety during mobilization, the nurse should perform what action? A) Support the patient's full body weight with a waist belt during ambulation. B) Have a colleague follow the patient closely with a wheelchair. C) Avoid mobilizing the patient in the early morning or late evening. D) Ensure that the patient's family members do not participate in mobilization.

B Feedback: During mobilization, a chair or wheelchair should be readily available in case the patient suddenly becomes fatigued or feels dizzy. The family should be encouraged to participate, as appropriate, and the nurse should not have to support the patient's full body weight. Morning and evening activity are not necessarily problematic.

17. The nurse is planning the care of a patient who is adapting to the use of a hearing aid for the first time. What is the most significant challenge experienced by a patient with hearing loss who is adapting to using a hearing aid for the first time? A) Regulating the tone and volume B) Learning to cope with amplification of background noise C) Constant irritation of the external auditory canal D) Challenges in keeping the hearing aid clean while minimizing exposure to moisture

B Feedback: Each of the answers represents a common problem experienced by patients using a hearing aid for the first time. However, amplification of background noise is a difficult problem to manage and is the major reason why patients stop using their hearing aid. All patients learning to use a hearing aid require support and coaching by the nurse and other members of the health care team. Patients should be encouraged to discuss their adaptation to the hearing aid with their audiologist.

A nurse is caring for a patient diagnosed with a hemorrhagic stroke. When creating this patient's plan of care, what goal should be prioritized? A) Prevent complications of immobility. B) Maintain and improve cerebral tissue perfusion. C) Relieve anxiety and pain. D) Relieve sensory deprivation.

B Feedback: Each of the listed goals is appropriate in the care of a patient recovering from a stroke. However, promoting cerebral perfusion is a priority physiologic need, on which the patient's survival depends.

A patient is scheduled to have an electronystagmography as part of a diagnostic workup for Ménière's disease. What question is it most important for the nurse to ask the patient in preparation for this test? A) Have you ever experienced claustrophobia or feelings of anxiety while in enclosed spaces? B) Do you currently take any tranquilizers or stimulants on a regular basis? C) Do you have a history of falls or problems with loss of balance? D) Do you have a history of either high or low blood pressure?

B Feedback: Electronystagmography measures changes in electrical potentials created by eye movements during induced nystagmus. Medications such as tranquilizers, stimulants, or antivertigo agents are withheld for 5 days before the test. Claustrophobia is not a significant concern associated with this test; rather, it is most often a concern for patients undergoing magnetic resonance imaging (MRI). Balance is impaired by Ménière's disease; therefore, a patient history of balance problems is important, but is not relevant to test preparation. Hypertension or hypotension, while important health problems, should not be affected by this test.

When caring for a patient who has had a stroke, a priority is reduction of ICP. What patient position is most consistent with this goal? A) Head turned slightly to the right side B) Elevation of the head of the bed C) Position changes every 15 minutes while awake D) Extension of the neck

B Feedback: Elevation of the head of the bed promotes venous drainage and lowers ICP; the nurse should avoid flexing or extending the neck or turning the head side to side. The head should be in a neutral midline position. Excessively frequent position changes are unnecessary.

A patient has just returned to the surgical floor after undergoing a retinal detachment repair. The postoperative orders specify that the patient should be kept in a prone position until otherwise ordered. What should the nurse do? A) Call the physician and ask for the order to be confirmed. B) Follow the order because this position will help keep the retinal repair intact. C) Instruct the patient to maintain this position to prevent bleeding. D) Reposition the patient after the first dressing change.

B Feedback: For pneumatic retinopexy, postoperative positioning of the patient is critical because the injected bubble must float into a position overlying the area of detachment, providing consistent pressure to reattach the sensory retina. The patient must maintain a prone position that would allow the gas bubble to act as a tamponade for the retinal break. Patients and family members should be made aware of these special needs beforehand so that the patient can be made as comfortable as possible. It would be inappropriate to deviate from this order and there is no obvious need to confirm the order.

An older adult patient has fallen in her home and is brought to the emergency department by ambulance with a suspected fractured hip. X-rays confirm a fracture of the left femoral neck. When planning assessments during the patient's presurgical care, the nurse should be aware of the patient's heightened risk of what complication? A) Osteomyelitis B) Avascular necrosis C) Phantom pain D) Septicemia

B Feedback: Fractures of the neck of the femur may damage the vascular system that supplies blood to the head and the neck of the femur, and the bone may become ischemic. For this reason, AVN is common in patients with femoral neck fractures. Infections are not immediate complications and phantom pain applies to patients with amputations, not hip fractures.

The nurse is providing health education to a patient newly diagnosed with glaucoma. The nurse teaches the patient that this disease has a familial tendency. The nurse should encourage the patient's immediate family members to undergo clinical examinations how often? A) At least monthly B) At least once every 2 years C) At least once every 5 years D) At least once every 10 years

B Feedback: Glaucoma has a family tendency and family members should be encouraged to undergo examinations at least once every 2 years to detect glaucoma early. Testing on a monthly basis is not necessary and excessive.

A patient has been diagnosed with glaucoma and the nurse is preparing health education regarding the patient's medication regimen. The patient states that she is eager to beat this disease and looks forward to the time that she will no longer require medication. How should the nurse best respond? A) You have a great attitude. This will likely shorten the amount of time that you need medications. B) In fact, glaucoma usually requires lifelong treatment with medications. C) Most people are treated until their intraocular pressure goes below 50 mm Hg. D) You can likely expect a minimum of 6 months of treatment.

B Feedback: Glaucoma requires lifelong pharmacologic treatment. Normal intraocular pressure is between 10 and 21 mm Hg.

A nurse is caring for an older adult who has been diagnosed with geriatric failure to thrive. This patient's prolonged immobility creates a risk for what complication? A) Muscle clonus B) Muscle atrophy C) Rheumatoid arthritis D) Muscle fasciculations

B Feedback: If a muscle is in disuse for an extended period of time, it is at risk of developing atrophy, which is the decrease in size. Clonus is a pattern of rhythmic muscle contractions and fasciculation is the involuntary twitch of muscle fibers; neither results from immobility. Lack of exercise is a risk factor for rheumatoid arthritis.

A patient is brought to the trauma center by ambulance after sustaining a high cervical spinal cord injury 1½ hours ago. Endotracheal intubation has been deemed necessary and the nurse is preparing to assist. What nursing diagnosis should the nurse associate with this procedure? A) Risk for impaired skin integrity B) Risk for injury C) Risk for autonomic dysreflexia D) Risk for suffocation

B Feedback: If endotracheal intubation is necessary, extreme care is taken to avoid flexing or extending the patient's neck, which can result in extension of a cervical injury. Intubation does not directly cause autonomic dysreflexia and the threat to skin integrity is a not a primary concern. Intubation does not carry the potential to cause suffocation.

Following a spinal cord injury a patient is placed in halo traction. While performing pin site care, the nurse notes that one of the traction pins has become detached. The nurse would be correct in implementing what priority nursing action? A) Complete the pin site care to decrease risk of infection. B) Notify the neurosurgeon of the occurrence. C) Stabilize the head in a lateral position. D) Reattach the pin to prevent further head trauma.

B Feedback: If one of the pins became detached, the head is stabilized in neutral position by one person while another notifies the neurosurgeon. Reattaching the pin as a nursing intervention would not be done due to risk of increased injury. Pin site care would not be a priority in this instance. Prevention of neurologic injury is the priority.

The nurse's assessment of a patient with significant visual losses reveals that the patient cannot count fingers. How should the nurse proceed with assessment of the patient's visual acuity? A) Assess the patient's vision using a Snellen chart. B) Determine whether the patient is able to see the nurse's hand motion. C) Perform a detailed examination of the patient's external eye structures. D) Palpate the patient's periocular regions.

B Feedback: If the patient cannot count fingers, the examiner raises one hand up and down or moves it side to side and asks in which direction the hand is moving. An inability to count fingers precludes the use of a Snellen chart. Palpation and examination cannot ascertain visual acuity.

The nurse is performing a neurologic assessment of a patient whose injuries have rendered her unable to follow verbal commands. How should the nurse proceed with assessing the patient's level of consciousness (LOC)? A) Assess the patient's vital signs and correlate these with the patient's baselines. B) Assess the patient's eye opening and response to stimuli. C) Document that the patient currently lacks a level of consciousness. D) Facilitate diagnostic testing in an effort to obtain objective data.

B Feedback: If the patient is not alert or able to follow commands, the examiner observes for eye opening; verbal response and motor response to stimuli, if any; and the type of stimuli needed to obtain a response. Vital signs and diagnostic testing are appropriate, but neither will allow the nurse to gauge the patient's LOC. Inability to follow commands does not necessarily denote an absolute lack of consciousness.

A rehabilitation nurse caring for a patient who has had a stroke is approached by the patient's family and asked why the patient has to do so much for herself when she is obviously struggling. What would be the nurse's best answer? A) We are trying to help her be as useful as she possibly can. B) The focus on care in a rehabilitation facility is to help the patient to resume as much self-care as possible. C) We aren't here to care for her the way the hospital staff did; we are here to help her get better so she can go home. D) Rehabilitation means helping patients do exactly what they did before their stroke.

B Feedback: In both acute care and rehabilitation facilities, the focus is on teaching the patient to resume as much self-care as possible. The goal of rehabilitation is not to be useful, nor is it to return patients to their prestroke level of functioning, which may be unrealistic.

While completing a health history on a patient who has recently experienced a seizure, the nurse would assess for what characteristic associated with the postictal state? A) Epileptic cry B) Confusion C) Urinary incontinence D) Body rigidity

B Feedback: In the postictal state (after the seizure), the patient is often confused and hard to arouse and may sleep for hours. The epileptic cry occurs from the simultaneous contractions of the diaphragm and chest muscles that occur during the seizure. Urinary incontinence and intense rigidity of the entire body are followed by alternating muscle relaxation and contraction (generalized tonic-clonic contraction) during the seizure.

A nurse is assessing a patient who is experiencing peripheral neurovascular dysfunction. What assessment findings are most consistent with this diagnosis? A) Hot skin with a capillary refill of 1 to 2 seconds B) Absence of feeling, capillary refill of 4 to 5 seconds, and cool skin C) Pain, diaphoresis, and erythema D) Jaundiced skin, weakness, and capillary refill of 3 seconds

B Feedback: Indicators of peripheral neurovascular dysfunction include pale, cyanotic, or mottled skin with a cool temperature; capillary refill greater than 3 seconds; weakness or paralysis with motion; and paresthesia, unrelenting pain, pain on passive stretch, or absence of feeling. Jaundice, diaphoresis, and warmth are inconsistent with peripheral neurovascular dysfunction.

A nurse is caring for a patient who is being treated in the hospital for a spontaneous vertebral fracture related to osteoporosis. The nurse should address the nursing diagnosis of Acute Pain Related to Fracture by implementing what intervention? A) Maintenance of high Fowler's positioning whenever possible B) Intermittent application of heat to the patient's back C) Use of a pressure-reducing mattress D) Passive range of motion exercises

B Feedback: Intermittent local heat and back rubs promote muscle relaxation following osteoporotic vertebral fractures. High Fowler's positioning is likely to exacerbate pain. The mattress must be adequately supportive, but pressure reduction is not necessarily required. Passive range of motion exercises to the back would cause pain and impair healing.

A patient with spinal cord injury has a nursing diagnosis of altered mobility and the nurse recognizes the increased the risk of deep vein thrombosis (DVT). Which of the following would be included as an appropriate nursing intervention to prevent a DVT from occurring? A) Placing the patient on a fluid restriction as ordered B) Applying thigh-high elastic stockings C) Administering an antifibrinolyic agent D) Assisting the patient with passive range of motion (PROM) exercises

B Feedback: It is important to promote venous return to the heart and prevent venous stasis in a patient with altered mobility. Applying elastic stockings will aid in the prevention of a DVT. The patient should not be placed on fluid restriction because a dehydrated state will increase the risk of clotting throughout the body. Antifibrinolytic agents cause the blood to clot, which is absolutely contraindicated in this situation. PROM exercises are not an effective protection against the development of DVT.

A patient exhibiting an altered level of consciousness (LOC) due to blunt-force trauma to the head is admitted to the ED. The physician determines the patient's injury is causing increased intracranial pressure (ICP). The nurse should gauge the patient's LOC on the results of what diagnostic tool? A) Monro-Kellie hypothesis B) Glasgow Coma Scale C) Cranial nerve function D) Mental status examination

B Feedback: LOC, a sensitive indicator of neurologic function, is assessed based on the criteria in the Glasgow Coma Scale: eye opening, verbal response, and motor response. The Monro-Kellie hypothesis states that because of the limited space for expansion within the skull, an increase in any one of the components (blood, brain tissue, cerebrospinal fluid) causes a change in the volume of the others. Cranial nerve function and the mental status examination would be part of the neurologic examination for this patient, but would not be the priority in evaluating LOC.

While reviewing the health history of an older adult experiencing hearing loss the nurse notes the patient has had no trauma or loss of balance. What aspect of this patient's health history is most likely to be linked to the patient's hearing deficit? A) Recent completion of radiation therapy for treatment of thyroid cancer B) Routine use of quinine for management of leg cramps C) Allergy to hair coloring and hair spray D) Previous perforation of the eardrum

B Feedback: Long-term, regular use of quinine for management of leg cramps is associated with loss of hearing acuity. Radiation therapy for cancer should not affect hearing; however, hearing can be significantly compromised by chemotherapy. Allergy to hair products may be associated with otitis externa; however, it is not linked to hearing loss. An ear drum that perforates spontaneously due to the sudden drop in altitude associated with a high dive usually heals well and is not likely to become infected. Recurrent otitis media with perforation can affect hearing as a result of chronic inflammation of the ossicles in the middle ear.

The nurse is providing care for a patient who is unconscious. What nursing intervention takes highest priority? A) Maintaining accurate records of intake and output B) Maintaining a patent airway C) Inserting a nasogastric (NG) tube as ordered D) Providing appropriate pain control

B Feedback: Maintaining a patent airway always takes top priority, even though each of the other listed actions is necessary and appropriate.

During discharge teaching the nurse realizes that the patient is not able to read medication bottles accurately and has not been taking her medications consistently at home. How should the nurse intervene most appropriately in this situation? A) Ask the social worker to investigate alternative housing arrangements. B) Ask the social worker to investigate community support agencies. C) Encourage the patient to explore surgical corrections for the vision problem. D) Arrange for referral to a rehabilitation facility for vision training.

B Feedback: Managing low vision involves magnification and image enhancement through the use of low-vision aids and strategies and referrals to social services and community agencies serving those with visual impairment. Community agencies offer services to patients with low vision, which include training in independent living skills and a variety of assistive devices for vision enhancement, orientation, and mobility, preventing patients from needing to enter a nursing facility. A rehabilitation facility is generally not needed by the patients to learn to use the assistive devices or to gain a greater degree of independence. Surgical options may or may not be available to the patient.

The staff educator is precepting a nurse new to the critical care unit when a patient with a T2 spinal cord injury is admitted. The patient is soon exhibiting manifestations of neurogenic shock. In addition to monitoring the patient closely, what would be the nurse's most appropriate action? A) Prepare to transfuse packed red blood cells. B) Prepare for interventions to increase the patient's BP. C) Place the patient in the Trendelenberg position. D) Prepare an ice bath to lower core body temperature.

B Feedback: Manifestations of neurogenic shock include decreased BP and heart rate. Cardiac markers would be expected to rise in cardiogenic shock. Transfusion, repositioning, and ice baths are not indicated interventions.

The nurse is caring for a patient who is in status epilepticus. What medication does the nurse know may be given to halt the seizure immediately? A) Intravenous phenobarbital (Luminal) B) Intravenous diazepam (Valium) C) Oral lorazepam (Ativan) D) Oral phenytoin (Dilantin)

B Feedback: Medical management of status epilepticus includes IV diazepam (Valium) and IV lorazepam (Ativan) given slowly in an attempt to halt seizures immediately. Other medications (phenytoin, phenobarbital) are given later to maintain a seizure-free state. Oral medications are not given during status epilepticus

A patient has suffered a muscle strain and is complaining of pain that she rates at 6 on a 10-point scale. The nurse should recommend what action? A) Taking an opioid analgesic as ordered B) Applying a cold pack to the injured site C) Performing passive ROM exercises D) Applying a heating pad to the affected muscle

B Feedback: Most pain can be relieved by elevating the involved part, applying cold packs, and administering analgesics as prescribed. Heat may exacerbate the pain by increasing blood circulation, and ROM exercises would likely be painful. Analgesia is likely necessary, but NSAIDs would be more appropriate than opioids.

In the course of a focused neurologic assessment, the nurse is palpating the patient's major muscle groups at rest and during passive movement. Data gleaned from this assessment will allow the nurse to describe which of the following aspects of neurologic function? A) Muscle dexterity B) Muscle tone C) Motor symmetry D) Deep tendon reflexes

B Feedback: Muscle tone (the tension present in a muscle at rest) is evaluated by palpating various muscle groups at rest and during passive movement. Data from this assessment do not allow the nurse to ascertain the patient's dexterity, reflexes, or motor symmetry.

A neurologic flow chart is often used to document the care of a patient with a traumatic brain injury. At what point in the patient's care should the nurse begin to use a neurologic flow chart? A) When the patient's condition begins to deteriorate B) As soon as the initial assessment is made C) At the beginning of each shift D) When there is a clinically significant change in the patient's condition

B Feedback: Neurologic parameters are assessed initially and as frequently as the patient's condition requires. As soon as the initial assessment is made, the use of a neurologic flowchart is started and maintained. A new chart is not begun at the start of every shift.

A patient who has sustained a nondepressed skull fracture is admitted to the acute medical unit. Nursing care should include which of the following? A) Preparation for emergency craniotomy B) Watchful waiting and close monitoring C) Administration of inotropic drugs D) Fluid resuscitation

B Feedback: Nondepressed skull fractures generally do not require surgical treatment; however, close observation of the patient is essential. A craniotomy would not likely be needed if the fracture is nondepressed. Even if treatment is warranted, it is unlikely to include inotropes or fluid resuscitation.

The nurse is admitting a patient to the unit who is scheduled to have an ossiculoplasty. What postoperative assessment will best determine whether the procedure has been successful? A) Otoscopy B) Audiometry C) Balance testing D) Culture and sensitivity testing of ear discharge

B Feedback: Ossiculoplasty is the surgical reconstruction of the middle ear bones to restore hearing. Consequently, results are assessed by testing hearing, not by visualizing the ear, testing balance, or culturing ear discharge.

A patient has come to the clinic for a routine annual physical. The nurse practitioner notes a palpable, painless projection of bone at the patient's shoulder. The projection appears to be at the distal end of the humerus. The nurse should suspect the presence of which of the following? A) Osteomyelitis B) Osteochondroma C) Osteomalacia D) Paget's disease

B Feedback: Osteochondroma is the most common benign bone tumor. It usually occurs as a large projection of bone at the end of long bones (at the knee or shoulder). Osteomyelitis, osteomalacia, and Paget's disease do not involve the development of excess bone tissue.

A patient with diabetes has been diagnosed with osteomyelitis. The nurse notes that the patient's right foot is pale and mottled, cool to touch, with a capillary refill of greater than 3 seconds. The nurse should suspect what type of osteomyelitis? A) Hematogenous osteomyelitis B) Osteomyelitis with vascular insufficiency C) Contiguous-focus osteomyelitis D) Osteomyelitis with muscular deterioration

B Feedback: Osteomyelitis is classified as hematogenous osteomyelitis (i.e., due to blood-borne spread of infection); contiguous-focus osteomyelitis, from contamination from bone surgery, open fracture, or traumatic injury (e.g., gunshot wound); and osteomyelitis with vascular insufficiency, seen most commonly among patients with diabetes and peripheral vascular disease, most commonly affecting the feet. Osteomyelitis with muscular deterioration does not exist.

A patient has undergone diagnostic testing and has been diagnosed with otosclerosis? What ear structure is primarily affected by this diagnosis? A) Malleus B) Stapes C) Incus D) Tympanic membrane

B Feedback: Otosclerosis involves the stapes and is thought to result from the formation of new, abnormal bone, especially around the oval window, with resulting fixation of the stapes.

The nurse is planning the care of a patient with Parkinson's disease. The nurse should be aware that treatment will focus on what pathophysiological phenomenon? A) Premature degradation of acetylcholine B) Decreased availability of dopamine C) Insufficient synthesis of epinephrine D) Delayed reuptake of serotonin

B Feedback: Parkinson's disease develops from decreased availability of dopamine, not acetylcholine, epinephrine, or serotonin.

35. A patient is being prepared for a total hip arthroplasty, and the nurse is providing relevant education. The patient is concerned about being on bed rest for several days after the surgery. The nurse should explain what expectation for activity following hip replacement? A) Actually, patients are only on bed rest for 2 to 3 days before they begin walking with assistance. B) The physical therapist will likely help you get up using a walker the day after your surgery. C) Our goal will actually be to have you walking normally within 5 days of your surgery. D) For the first two weeks after the surgery, you can use a wheelchair to meet your mobility needs.

B Feedback: Patients post-THA begin ambulation with the assistance of a walker or crutches within a day after surgery. Wheelchairs are not normally utilized. Baseline levels of mobility are not normally achieved until several weeks after surgery, however.

Which of the following patients should the nurse recognize as being at the highest risk for the development of osteomyelitis? A) A middle-age adult who takes ibuprofen daily for rheumatoid arthritis B) An elderly patient with an infected pressure ulcer in the sacral area C) A 17-year-old football player who had orthopedic surgery 6 weeks prior D) An infant diagnosed with jaundice

B Feedback: Patients who are at high risk of osteomyelitis include those who are poorly nourished, elderly, and obese. The elderly patient with an infected sacral pressure ulcer is at the greatest risk for the development of osteomyelitis, as this patient has two risk factors: age and the presence of a soft-tissue infection that has the potential to extend into the bone. The patient with rheumatoid arthritis has one risk factor and the infant with jaundice has no identifiable risk factors. The patient 6 weeks postsurgery is beyond the usual window of time for the development of a postoperative surgical wound infection.

An elite high school football player has been diagnosed with a shoulder dislocation. The patient has been treated and is eager to resume his role on his team, stating that he is not experiencing pain. What should the nurse emphasize during health education? A) The need to take analgesia regardless of the short-term absence of pain B) The importance of adhering to the prescribed treatment and rehabilitation regimen C) The fact that he has a permanently increased risk of future shoulder dislocations D) The importance of monitoring for intracapsular bleeding once he resumes playing

B Feedback: Patients who have experienced sports-related injuries are often highly motivated to return to their previous level of activity. Adherence to restriction of activities and gradual resumption of activities needs to be reinforced. Appropriate analgesia use must be encouraged, but analgesia does not necessarily have to be taken in the absence of pain. If healing is complete, the patient does not likely have a greatly increased risk of reinjury. Dislocations rarely cause bleeding after the healing process.

A nurse is caring for a patient who has suffered an unstable thoracolumbar fracture. Which of the following is the priority during nursing care? A) Preventing infection B) Maintaining spinal alignment C) Maximizing function D) Preventing increased intracranial pressure

B Feedback: Patients with an unstable fracture must have their spine in alignment at all times in order to prevent neurologic damage. This is a greater threat, and higher priority, than promoting function and preventing infection, even though these are both valid considerations. Increased ICP is not a high risk.

A patient presents at a clinic complaining of pain in his heel so bad that it inhibits his ability to walk. The patient is subsequently diagnosed with plantar fasciitis. This patient's plan of care should include what intervention? A) Wrapping the affected area in lamb's wool or gauze to relieve pressure B) Gently stretching the foot and the Achilles tendon C) Wearing open-toed shoes at all times D) Applying topical analgesic ointment to plantar surface each morning

B Feedback: Plantar fasciitis leads to pain that is localized to the anterior medial aspect of the heel and diminishes with gentle stretching of the foot and Achilles tendon. Dressings of any kind are not of therapeutic benefit and analgesic ointments do not address the pathology of the problem. Open-toed shoes are of no particular benefit.

A nurse is assessing a patient who is receiving traction. The nurse's assessment confirms that the patient is able to perform plantar flexion. What conclusion can the nurse draw from this finding? A) The leg that was assessed is free from DVT. B) The patient's tibial nerve is functional. C) Circulation to the distal extremity is adequate. D) The patient does not have peripheral neurovascular dysfunction.

B Feedback: Plantar flexion demonstrates function of the tibial nerve. It does not demonstrate the absence of DVT and does not allow the nurse to ascertain adequate circulation. The nurse must perform more assessments on more sites in order to determine an absence of peripheral neurovascular dysfunction.

3. A gerontologic nurse planning the neurologic assessment of an older adult is considering normal, age-related changes. Of what phenomenon should the nurse be aware? A) Hyperactive deep tendon reflexes B) Reduction in cerebral blood flow C) Increased cerebral metabolism D) Hypersensitivity to painful stimuli

B Feedback: Reduction in cerebral blood flow (CBF) is a change that occurs in the normal aging process. Deep tendon reflexes can be decreased or, in some cases, absent. Cerebral metabolism decreases as the patient advances in age. Reaction to painful stimuli may be decreased with age. Because pain is an important warning signal, caution must be used when hot or cold packs are used.

34. The physician has ordered a somatosensory evoked responses (SERs) test for a patient for whom the nurse is caring. The nurse is justified in suspecting that this patient may have a history of what type of neurologic disorder? A) Hypothalamic disorder B) Demyelinating disease C) Brainstem deficit D) Diabetic neuropathy

B Feedback: SERs are used to detect deficits in the spinal cord or peripheral nerve conduction and to monitor spinal cord function during surgical procedures. The test is also useful in the diagnosis of demyelinating diseases, such as multiple sclerosis and polyneuropathies, where nerve conduction is slowed. The test is not done to diagnose hypothalamic disorders, brainstem deficits, or diabetic neuropathies.

A patient is undergoing diagnostic testing for suspected Paget's disease. What assessment finding is most consistent with this diagnosis? A) Altered serum magnesium levels B) Altered serum calcium levels C) Altered serum potassium levels D) Altered serum sodium levels

B Feedback: Serum calcium levels are altered in patients with osteomalacia, parathyroid dysfunction, Paget's disease, metastatic bone tumors, or prolonged immobilization. Paget's disease is not directly associated with altered magnesium, potassium, or sodium levels.

A family member brings the patient to the clinic for a follow-up visit after a stroke. The family member asks the nurse what he can do to decrease his chance of having another stroke. What would be the nurse's best answer? A) Have your heart checked regularly. B) Stop smoking as soon as possible. C) Get medication to bring down your sodium levels. D) Eat a nutritious diet.

B Feedback: Smoking is a modifiable and highly significant risk factor for stroke. The significance of smoking, and the potential benefits of quitting, exceed the roles of sodium, diet, and regular medical assessments.

A group of high school students is attending a concert, which will be at a volume of 80 to 90 dB. What is a health consequence of this sound level? A) Hearing will not be affected by a decibel level in this range. B) Hearing loss may occur with a decibel level in this range. C) Sounds in this decibel level are not perceived to be harsh to the ear. D) Ear plugs will have no effect on these decibel levels.

B Feedback: Sound louder than 80 dB is perceived by the human ear to be harsh and can be damaging to the inner ear. Ear protection or plugs do help to minimize the effects of high decibel levels.

A nurse is planning preoperative teaching for a patient with hearing loss due to otosclerosis. The patient is scheduled for a stapedectomy with insertion of a prosthesis. What information is most crucial to include in the patient's preoperative teaching? A) The procedure is an effective, time-tested treatment for sensory hearing loss. B) The patient is likely to experience resolution of conductive hearing loss after the procedure. C) Several months of post-procedure rehabilitation will be needed to maximize benefits. D) The procedure is experimental, but early indications suggest great therapeutic benefits.

B Feedback: Stapedectomy is a very successful time-tested procedure, resulting in the restoration of conductive hearing loss. Lengthy rehabilitation is not normally required.

A school nurse is called to the playground where a 6-year-old girl has been found unresponsive and "staring into space," according to the playground supervisor. How would the nurse document the girl's activity in her chart at school? A) Generalized seizure B) Absence seizure C) Focal seizure D) Unclassified seizure

B Feedback: Staring episodes characterize an absence seizure, whereas focal seizures, generalized seizures, and unclassified seizures involve uncontrolled motor activity.

A trauma patient in the ICU has been declared brain dead. What diagnostic test is used in making the determination of brain death? A) Magnetic resonance imaging (MRI) B) Electroencephalography (EEG) C) Electromyelography (EMG) D) Computed tomography (CT)

B Feedback: The EEG can be used in determining brain death. MRI, CT, and EMG are not normally used in determining brain death.

The nurse and a colleague are performing the Epley maneuver with a patient who has a diagnosis of benign paroxysmal positional vertigo. The nurses should begin this maneuver by performing what action? A) Placing the patient in a prone position B) Assisting the patient into a sitting position C) Instilling 15 mL of warm normal saline into one of the patient's ears D) Assessing the patient's baseline hearing by performing the whisper test

B Feedback: The Epley maneuver is performed by placing the patient in a sitting position, turning the head to a 45-degree angle on the affected side, and then quickly moving the patient to the supine position. Saline is not instilled into the ears and there is no need to assess hearing before the test.

A nurse is reviewing the trend of a patient's scores on the Glasgow Coma Scale (GCS). This allows the nurse to gauge what aspect of the patient's status? A) Reflex activity B) Level of consciousness C) Cognitive ability D) Sensory involvement

B Feedback: The Glasgow Coma Scale (GCS) examines three responses related to LOC: eye opening, best verbal response, and best motor response.

A patient who presents for an eye examination is diagnosed as having a visual acuity of 20/40. The patient asks the nurse what these numbers specifically mean. What is a correct response by the nurse? A) A person whose vision is 20/40 can see an object from 40 feet away that a person with 20/20 vision can see from 20 feet away. B) A person whose vision is 20/40 can see an object from 20 feet away that a person with 20/20 vision can see from 40 feet away. C) A person whose vision is 20/40 can see an object from 40 inches away that a person with 20/20 vision can see from 20 inches away. D) A person whose vision is 20/40 can see an object from 20 inches away that a person with 20/20 vision can see from 40 inches away.

B Feedback: The Snellen chart is a tool used to measure visual acuity. It is composed of a series of progressively smaller rows of letters and is used to test distance vision. The fraction 20/20 is considered the standard of normal vision. Most people can see the letters on the line designated as 20/20 from a distance of 20 feet. A person whose vision is 20/40 can see an object from 20 feet away that a person with 20/20 vision can see from 40 feet away.

While assessing a patient who has had knee replacement surgery, the nurse notes that the patient has developed a hematoma at the surgical site. The affected leg has a decreased pedal pulse. What would be the priority nursing diagnosis for this patient? A) Risk for Infection B) Risk for Peripheral Neurovascular Dysfunction C) Unilateral Neglect D) Disturbed Kinesthetic Sensory Perception

B Feedback: The hematoma may cause an interruption of tissue perfusion, so the most appropriate nursing diagnosis is Risk of Peripheral Neurovascular Dysfunction. There is also an associated risk for infection because of the hematoma, but impaired neurovascular function is a more acute threat. Unilateral neglect and impaired sensation are lower priorities than neurovascular status.

A public health nurse is organizing a campaign that will address the leading cause of musculoskeletal-related disability in the United States. The nurse should focus on what health problem? A) Osteoporosis B) Arthritis C) Hip fractures D) Lower back pain

B Feedback: The leading cause of musculoskeletal-related disability in the United States is arthritis.

A patient with a right tibial fracture is being discharged home after having a cast applied. What instruction should the nurse provide in relationship to the patient's cast care? A) Cover the cast with a blanket until the cast dries. B) Keep your right leg elevated above heart level. C) Use a clean object to scratch itches inside the cast. D) A foul smell from the cast is normal after the first few days.

B Feedback: The leg should be elevated to promote venous return and prevent edema. The cast shouldn't be covered while drying because this will cause heat buildup and prevent air circulation. No foreign object should be inserted inside the cast because of the risk of cutting the skin and causing an infection. A foul smell from a cast is never normal and may indicate an infection.

A patient has lost most of her vision as a result of macular degeneration. When attempting to meet this patient's psychosocial needs, what nursing action is most appropriate? A) Encourage the patient to focus on her use of her other senses. B) Assess and promote the patient's coping skills during interactions with the patient. C) Emphasize that her lifestyle will be unchanged once she adapts to her vision loss. D) Promote the patient's hope for recovery.

B Feedback: The nurse should empathically promote the patient's coping with her loss. Focusing on the remaining senses could easily be interpreted as downplaying the patient's loss, and recovery is not normally a realistic possibility. Even with successful adaptation, the patient's lifestyle will be profoundly affected.

A patient has sustained a long bone fracture and the nurse is preparing the patient's care plan. Which of the following should the nurse include in the care plan? A) Administer vitamin D and calcium supplements as ordered. B) Monitor temperature and pulses of the affected extremity. C) Perform passive range of motion exercises as tolerated. D) Administer corticosteroids as ordered.

B Feedback: The nurse should include monitoring for sufficient blood supply by assessing the color, temperature, and pulses of the affected extremity. Weight-bearing exercises are encouraged, but passive ROM exercises have the potential to cause pain and inhibit healing. Corticosteroids, vitamin D, and calcium are not normally administered.

A patient with a simple arm fracture is receiving discharge education from the nurse. What would the nurse instruct the patient to do? A) Elevate the affected extremity to shoulder level when at rest. B) Engage in exercises that strengthen the unaffected muscles. C) Apply topical anesthetics to accessible skin surfaces as needed. D) Avoid using analgesics so that further damage is not masked.

B Feedback: The nurse will encourage the patient to engage in exercises that strengthen the unaffected muscles. Comfort measures may include appropriate use of analgesics and elevation of the affected extremity to the heart level. Topical anesthetics are not typically used.

A nurse is performing a nursing assessment of a patient suspected of having a musculoskeletal disorder. What is the primary focus of the nursing assessment with a patient who has a musculoskeletal disorder? A) Range of motion B) Activities of daily living C) Gait D) Strength

B Feedback: The nursing assessment is primarily a functional evaluation, focusing on the patient's ability to perform activities of daily living. The nurse also assesses strength, gait, and ROM, but these are assessed to identify their effect on functional status rather than to identify a medical diagnosis.

A nurse is caring for an older adult patient who is preparing for discharge following recovery from a total hip replacement. Which of the following outcomes must be met prior to discharge? A) Patient is able to perform ADLs independently. B) Patient is able to perform transfers safely. C) Patient is able to weight-bear equally on both legs. D) Patient is able to demonstrate full ROM of the affected hip.

B Feedback: The patient must be able to perform transfers and to use mobility aids safely. Each of the other listed goals is unrealistic for the patient who has undergone recent hip replacement.

The nurse is providing discharge education to an adult patient who will begin a regimen of ocular medications for the treatment of glaucoma. How can the nurse best determine if the patient is able to self-administer these medications safely and effectively? A) Assess the patient for any previous inability to self-manage medications. B) Ask the patient to demonstrate the instillation of her medications. C) Determine whether the patient can accurately describe the appropriate method of administering her medications. D) Assess the patient's functional status.

B Feedback: The patient or the caregiver at home should be asked to demonstrate actual eye drop administration. This method of assessment is more accurate than asking the patient to describe the process or determining earlier inabilities to self-administer medications. The patient's functional status will not necessarily determine the ability to administer medication safely.

The neurologic ICU nurse is admitting a patient following a craniotomy using the supratentorial approach. How should the nurse best position the patient? A) Position the patient supine. B) Maintain head of bed (HOB) elevated at 30 to 45 degrees. C) Position patient in prone position. D) Maintain bed in Trendelenberg position.

B Feedback: The patient undergoing a craniotomy with a supratentorial (above the tentorium) approach should be placed with the HOB elevated 30 to 45 degrees, with the neck in neutral alignment. Each of the other listed positions would cause a dangerous elevation in ICP.

A patient is brought to the ER following a motor vehicle accident in which he sustained head trauma. Preliminary assessment reveals a vision deficit in the patient's left eye. The nurse should associate this abnormal finding with trauma to which of the following cerebral lobes? A) Temporal B) Occipital C) Parietal D) Frontal

B Feedback: The posterior lobe of the cerebral hemisphere is responsible for visual interpretation. The temporal lobe contains the auditory receptive areas. The parietal lobe contains the primary sensory cortex, and is essential to an individual's awareness of the body in space, as well as orientation in space and spatial relations. The frontal lobe functions in concentration, abstract thought, information storage or memory, and motor function.

The nurse is caring for a patient who is postoperative following a craniotomy. When writing the plan of care, the nurse identifies a diagnosis of "deficient fluid volume related to fluid restriction and osmotic diuretic use." What would be an appropriate intervention for this diagnosis? A) Change the patient's position as indicated. B) Monitor serum electrolytes. C) Maintain NPO status. D) Monitor arterial blood gas (ABG) values.

B Feedback: The postoperative fluid regimen depends on the type of neurosurgical procedure and is determined on an individual basis. The volume and composition of fluids are adjusted based on daily serum electrolyte values, along with fluid intake and output. Fluids may have to be restricted in patients with cerebral edema. Changing the patient's position, maintaining an NPO status, and monitoring ABG values do not relate to the nursing diagnosis of deficient fluid volume.

The nurse is providing care for a patient who has had a below-the-knee amputation. The nurse enters the patient's room and finds him resting in bed with his residual limb supported on pillow. What is the nurse's most appropriate action? A) Inform the surgeon of this finding. B) Explain the risks of flexion contracture to the patient. C) Transfer the patient to a sitting position. D) Encourage the patient to perform active ROM exercises with the residual limb.

B Feedback: The residual limb should not be placed on a pillow, because a flexion contracture of the hip may result. There is no acute need to contact the patient's surgeon. Encouraging exercise or transferring the patient does not address the risk of flexion contracture.

The nurse has admitted a new patient to the unit. One of the patient's admitting orders is for an adrenergic medication. The nurse knows that this medication will have what effect on the circulatory system? A) Thin, watery saliva B) Increased heart rate C) Decreased BP D) Constricted bronchioles

B Feedback: The term "adrenergic" refers to the sympathetic nervous system. Sympathetic effects include an increased rate and force of the heartbeat. Cholinergic effects, which correspond to the parasympathetic division of the autonomic nervous system, include thin, watery saliva, decreased rate and force of heartbeat, and decreased BP.

A patient has just begun been receiving skeletal traction and the nurse is aware that muscles in the patient's affected limb are spastic. How does this change in muscle tone affect the patient's traction prescription? A) Traction must temporarily be aligned in a slightly different direction. B) Extra weight is needed initially to keep the limb in proper alignment. C) A lighter weight should be initially used. D) Weight will temporarily alternate between heavier and lighter weights.

B Feedback: The traction weights applied initially must overcome the shortening spasms of the affected muscles. As the muscles relax, the traction weight is reduced to prevent fracture dislocation and to promote healing. Weights never alternate between heavy and light.

When assessing a patient's peripheral nerve function, the nurse uses an instrument to prick the fat pad at the top of the patient's small finger. This action will assess which of the following nerves? A) Radial B) Ulnar C) Median D) Tibial

B Feedback: The ulnar nerve is assessed for sensation by pricking the fat pad at the top of the small finger. The radial, median, and tibial nerves are not assessed in this manner.

The nurse is caring for a patient who has undergone a mastoidectomy. In an effort to prevent postoperative infection, what intervention should the nurse implement? A) Teach the patient about the risks of ototoxic medications. B) Instruct the patient to protect the ear from water for several weeks. C) Teach the patient to remove cerumen safely at least once per week. D) Instruct the patient to protect the ear from temperature extremes until healing is complete.

B Feedback: To prevent infection, the patient is instructed to prevent water from entering the external auditory canal for 6 weeks. Ototoxic medications and temperature extremes do not present a risk for infection. Removal of cerumen during the healing process should be avoided due to the possibility of trauma.

A nurse is caring for a patient receiving skeletal traction. Due to the patient's severe limits on mobility, the nurse has identified a risk for atelectasis or pneumonia. What intervention should the nurse provide in order to prevent these complications? A) Perform chest physiotherapy once per shift and as needed. B) Teach the patient to perform deep breathing and coughing exercises. C) Administer prophylactic antibiotics as ordered. D) Administer nebulized bronchodilators and corticosteroids as ordered.

B Feedback: To prevent these complications, the nurse should educate the patient about performing deep-breathing and coughing exercises to aid in fully expanding the lungs and clearing pulmonary secretions. Antibiotics, bronchodilators, and steroids are not used on a preventative basis and chest physiotherapy is unnecessary and implausible for a patient in traction.

The nurse is performing stroke risk screenings at a hospital open house. The nurse has identified four patients who might be at risk for a stroke. Which patient is likely at the highest risk for a hemorrhagic stroke? A) White female, age 60, with history of excessive alcohol intake B) White male, age 60, with history of uncontrolled hypertension C) Black male, age 60, with history of diabetes D) Black male, age 50, with history of smoking

B Feedback: Uncontrolled hypertension is the primary cause of a hemorrhagic stroke. Control of hypertension, especially in individuals over 55 years of age, clearly reduces the risk for hemorrhagic stroke. Additional risk factors are increased age, male gender, and excessive alcohol intake. Another high-risk group includes African Americans, where the incidence of first stroke is almost twice that as in Caucasians.

A patient with a total hip replacement is progressing well and expects to be discharged tomorrow. On returning to bed after ambulating, he complains of a new onset of pain at the surgical site. What is the nurse's best action? A) Administer pain medication as ordered. B) Assess the surgical site and the affected extremity. C) Reassure the patient that pain is a direct result of increased activity. D) Assess the patient for signs and symptoms of systemic infection.

B Feedback: Worsening pain after a total hip replacement may indicate dislocation of the prosthesis. Assessment of pain should include evaluation of the wound and the affected extremity. Assuming he's anxious about discharge and administering pain medication do not address the cause of the pain. Sudden severe pain is not considered normal after hip replacement. Sudden pain is rarely indicative of a systemic infection.

A nurse assesses a client after administering prescribed levetiracetam (Keppra). Which laboratory tests should the nurse monitor for potential adverse effects of this medication? a. Serum electrolyte levels b. Kidney function tests c. Complete blood cell count d. Antinuclear antibodies

B Adverse effects of levetiracetam include coordination problems and renal toxicity. The other laboratory tests are not affected by levetiracetam.

A nurse obtains a focused health history for a client who is scheduled for magnetic resonance angiography. Which priority question should the nurse ask before the test? a. "Have you had a recent blood transfusion?" b. "Do you have allergies to iodine or shellfish?" c. "Are you taking any cardiac medications?" d. "Do you currently use oral contraceptives?"

B Allergies to iodine and/or shellfish need to be explored because the client may have a similar reaction to the dye used in the procedure. In some cases, the client may need to be medicated with antihistamines or steroids before the test is given. A recent blood transfusion or current use of cardiac medications or oral contraceptives would not affect the angiography.

A nurse assesses a client with Huntington disease. Which motor changes should the nurse monitor for in this client? a. Shuffling gait b. Jerky hand movements c. Continuous chewing motions d. Tremors of the hands

B An imbalance between excitatory and inhibitory neurotransmitters leads to uninhibited motor movements, such as brisk, jerky, purposeless movements of the hands, face, tongue, and legs. Shuffling gait, continuous chewing motions, and tremors are associated with Parkinson disease.

A nurse is teaching a client who experiences migraine headaches and is prescribed a beta blocker. Which statement should the nurse include in this client's teaching? a. "Take this drug only when you have prodromal symptoms indicating the onset of a migraine headache." b. "Take this drug as ordered, even when feeling well, to prevent vascular changes associated with migraine headaches." c. "This drug will relieve the pain during the aura phase soon after a headache has started." d. "This medication will have no effect on your heart rate or blood pressure because you are taking it for migraines."

B Beta blockers are prescribed as prophylactic treatment to prevent the vascular changes that initiate migraine headaches. Heart rate and blood pressure will also be affected, and the client should monitor these side effects. The other responses do not discuss appropriate uses of the medication.

A nurse assesses the health history of a client who is prescribed ziconotide (Prialt) for chronic back pain. Which assessment question should the nurse ask? a. "Are you taking a nonsteroidal anti-inflammatory drug?" b. "Do you have a mental health disorder?" c. "Are you able to swallow medications?" d. "Do you smoke cigarettes or any illegal drugs?"

B Clients who have a mental health or behavioral health problem should not take ziconotide. The other questions do not identify a contraindication for this medication.

A nurse teaches a client who is recovering from a spinal fusion. Which statement should the nurse include in this client's postoperative instructions? a. "Only lift items that are 10 pounds or less." b. "Wear your brace whenever you are out of bed." c. "You must remain in bed for 3 weeks after surgery." d. "You are prescribed medications to prevent rejection."

B Clients who undergo spinal fusion are fitted with a brace that they must wear throughout the healing process (usually 3 to 6 months) whenever they are out of bed. The client should not lift anything. The client does not need to remain in bed. Medications for rejection prevention are not necessary for this procedure.

A nurse in a hematology clinic is working with four clients who have polycythemia vera. Which client should the nurse see first? a. Client with a blood pressure of 180/98 mm Hg b. Client who reports shortness of breath c. Client who reports calf tenderness and swelling d. Client with a swollen and painful left great toe

B Clients with polycythemia vera often have clotting abnormalities due to the hyperviscous blood with sluggish flow. The client reporting shortness of breath may have a pulmonary embolism and should be seen first. The client with a swollen calf may have a deep vein thrombosis and should be seen next. High blood pressure and gout symptoms are common findings with this disorder.

A nurse plans care for a client who has a hypoactive response to a test of deep tendon reflexes. Which intervention should the nurse include in this client's plan of care? a. Check bath water temperature with a thermometer. b. Provide the client with assistance when ambulating. c. Place elastic support hose on the client's legs. d. Assess the client's feet for wounds each shift.

B Hypoactive deep tendon reflexes and loss of vibration sense can impair balance and coordination, predisposing the client to falls. The nurse should plan to provide the client with ambulation assistance to prevent injury. The other interventions do not address the client's problem.

A nurse is preparing to administer a blood transfusion. What action is most important? a. Correctly identifying client using two identifiers b. Ensuring informed consent is obtained if required c. Hanging the blood product with Ringer's lactate d. Staying with the client for the entire transfusion

B If the facility requires informed consent for transfusions, this action is most important because it precedes the other actions taken during the transfusion. Correctly identifying the client and blood product is a National Patient Safety Goal, and is the most important action after obtaining informed consent. Ringer's lactate is not used to transfuse blood. The nurse does not need to stay with the client for the duration of the transfusion.

A nurse cares for a client who is experiencing status epilepticus. Which prescribed medication should the nurse prepare to administer? a. Atenolol (Tenormin) b. Lorazepam (Ativan) c. Phenytoin (Dilantin) d. Lisinopril (Prinivil)

B Initially, intravenous lorazepam is administered to stop motor movements. This is followed by the administration of phenytoin. Atenolol, a beta blocker, and lisinopril, an angiotensin-converting enzyme inhibitor, are not administered for seizure activity. These medications are typically administered for hypertension and heart failure.

An emergency department nurse cares for a client who experienced a spinal cord injury 1 hour ago. Which prescribed medication should the nurse prepare to administer? a. Intrathecal baclofen (Lioresal) b. Methylprednisolone (Medrol) c. Atropine sulfate d. Epinephrine (Adrenalin)

B Methylprednisolone (Medrol) should be given within 8 hours of the injury. Clients who receive this therapy usually show improvement in motor and sensory function. The other medications are inappropriate for this client.

The family of a neutropenic client reports the client "is not acting right." What action by the nurse is the priority? a. Ask the client about pain. b. Assess the client for infection. c. Delegate taking a set of vital signs. d. Look at today's laboratory results.

B Neutropenic clients often do not have classic manifestations of infection, but infection is the most common cause of death in neutropenic clients. The nurse should assess for infection. The nurse should assess for pain but this is not the priority. The nurse should take the client's vital signs instead of delegating them since the client has had a change in status. Laboratory results may be inconclusive.

A nurse assesses a client with a history of epilepsy who experiences stiffening of the muscles of the arms and legs, followed by an immediate loss of consciousness and jerking of all extremities. How should the nurse document this activity? a. Atonic seizure b. Tonic-clonic seizure c. Myoclonic seizure d. Absence seizure

B Seizure activity that begins with stiffening of the arms and legs, followed by loss of consciousness and jerking of all extremities, is characteristic of a tonic-clonic seizure. An atonic seizure presents as a sudden loss of muscle tone followed by postictal confusion. A myoclonic seizure presents with a brief jerking or stiffening of extremities that may occur singly or in groups. Absence seizures present with automatisms, and the client is unaware of his or her environment.

A nurse obtains a health history on a client prior to administering prescribed sumatriptan succinate (Imitrex) for migraine headaches. Which condition should alert the nurse to hold the medication and contact the health care provider? a. Bronchial asthma b. Prinzmetal's angina c. Diabetes mellitus d. Chronic kidney disease

B Sumatriptan succinate effectively reduces pain and other associated symptoms of migraine headache by binding to serotonin receptors and triggering cranial vasoconstriction. Vasoconstrictive effects are not confined to the cranium and can cause coronary vasospasm in clients with Prinzmetal's angina. The other conditions would not affect the client's treatment.

A client admitted for sickle cell crisis is distraught after learning her child also has the disease. What response by the nurse is best? a. "Both you and the father are equally responsible for passing it on." b. "I can see you are upset. I can stay here with you a while if you like." c. "It's not your fault; there is no way to know who will have this disease." d. "There are many good treatments for sickle cell disease these days."

B The best response is for the nurse to offer self, a therapeutic communication technique that uses presence. Attempting to assign blame to both parents will not help the client feel better. There is genetic testing available, so it is inaccurate to state there is no way to know who will have the disease. Stating that good treatments exist belittles the client's feelings.

A nurse is teaching a client with multiple sclerosis who is prescribed cyclophosphamide (Cytoxan) and methylprednisolone (Medrol). Which statement should the nurse include in this client's discharge teaching? a. "Take warm baths to promote muscle relaxation." b. "Avoid crowds and people with colds." c. "Relying on a walker will weaken your gait." d. "Take prescribed medications when symptoms occur."

B The client should be taught to avoid people with any type of upper respiratory illness because these medications are immunosuppressive. Warm baths will exacerbate the client's symptoms. Assistive devices may be required for safe ambulation. Medication should be taken at all times and should not be stopped.

A nurse assesses a client who is recovering from a lumbar puncture (LP). Which complication of this procedure should alert the nurse to urgently contact the health care provider? a. Weak pedal pulses b. Nausea and vomiting c. Increased thirst d. Hives on the chest

B The nurse should immediately contact the provider if the client experiences a severe headache, nausea, vomiting, photophobia, or a change in level of consciousness after an LP, which are all signs of increased intracranial pressure. Weak pedal pulses, increased thirst, and hives are not complications of an LP.

A nurse witnesses a client begin to experience a tonic-clonic seizure and loss of consciousness. Which action should the nurse take? a. Start fluids via a large-bore catheter. b. Turn the client's head to the side. c. Administer IV push diazepam. d. Prepare to intubate the client.

B The nurse should turn the client's head to the side to prevent aspiration and allow drainage of secretions. Anticonvulsants are administered on a routine basis if a seizure is sustained. If the seizure is sustained (status epilepticus), the client must be intubated and should be administered oxygen, 0.9% sodium chloride, and IV push lorazepam or diazepam.

A client receiving a blood transfusion develops anxiety and low back pain. After stopping the transfusion, what action by the nurse is most important? a. Documenting the events in the client's medical record b. Double-checking the client and blood product identification c. Placing the client on strict bedrest until the pain subsides d. Reviewing the client's medical record for known allergies

B This client had a hemolytic transfusion reaction, most commonly caused by blood type or Rh incompatibility. The nurse should double-check all identifying information for both the client and blood type. Documentation occurs after the client is stable. Bedrest may or may not be needed. Allergies to medications or environmental items is not related.

client has a serum ferritin level of 8 ng/mL and microcytic red blood cells. What action by the nurse is best? a. Encourage high-protein foods. b. Perform a Hemoccult test on the client's stools. c. Offer frequent oral care. d. Prepare to administer cobalamin (vitamin B12).

B This client has laboratory findings indicative of iron deficiency anemia. The most common cause of this disorder is blood loss, often from the GI tract. The nurse should perform a Hemoccult test on the client's stools. High-protein foods may help the condition, but dietary interventions take time to work. That still does not determine the cause. Frequent oral care is not related. Cobalamin injections are for pernicious anemia.

The nurse educator is reviewing the assessment of cranial nerves. What should the educator identify as the specific instances when cranial nerves should be assessed? Select all that apply. A) When a neurogenic bladder develops B) When level of consciousness is decreased C) With brain stem pathology D) In the presence of peripheral nervous system disease E) When a spinal reflex is interrupted

B, C, D Feedback: Cranial nerves are assessed when level of consciousness is decreased, with brain stem pathology, or in the presence of peripheral nervous system disease. Abnormalities in muscle tone and involuntary movements are less likely to prompt the assessment of cranial nerves, since these nerves do not directly mediate most aspects of muscle tone and movement.

The nurse is caring for a patient who is rapidly progressing toward brain death. The nurse should be aware of what cardinal signs of brain death? Select all that apply. A) Absence of pain response B) Apnea C) Coma D) Absence of brain stem reflexes E) Absence of deep tendon reflexes

B, C, D Feedback: The three cardinal signs of brain death upon clinical examination are coma, the absence of brain stem reflexes, and apnea. Absences of pain response and deep tendon reflexes are not necessarily indicative of brain death.

A nurse in a busy emergency department provides care for many patients who present with contusions, strains, or sprains. Treatment modalities that are common to all of these musculoskeletal injuries include which of the following? Select all that apply. A) Massage B) Applying ice C) Compression dressings D) Resting the affected extremity E) Corticosteroids F) Elevating the injured limb

B, C, D, F Feedback: Treatment of contusions, strains, and sprains consists of resting and elevating the affected part, applying cold, and using a compression bandage. Massage and corticosteroids are not used to treat these injuries.

A client with chronic anemia has had many blood transfusions. What medications does the nurse anticipate teaching the client about adding to the regimen? (Select all that apply.) a. Azacitidine (Vidaza) b. Darbepoetin alfa (Aranesp) c. Decitabine (Dacogen) d. Epoetin alfa (Epogen) e. Methylprednisolone (Solu-Medrol)

B, D Darbepoetin alfa and epoetin alfa are both red blood cell colony-stimulating factors that will help increase the production of red blood cells. Azacitidine and decitabine are used for myelodysplastic syndromes. Methylprednisolone is a steroid and would not be used for this problem.

As a member of the stroke team, the nurse knows that thrombolytic therapy carries the potential for benefit and for harm. The nurse should be cognizant of what contraindications for thrombolytic therapy? Select all that apply. A) INR above 1.0 B) Recent intracranial pathology C) Sudden symptom onset D) Current anticoagulation therapy E) Symptom onset greater than 3 hours prior to admission

B, D, E Feedback: Some of the absolute contraindications for thrombolytic therapy include symptom onset greater than 3 hours before admission, a patient who is anticoagulated (with an INR above 1.7), or a patient who has recently had any type of intracranial pathology (e.g., previous stroke, head injury, trauma).

A nurse assesses a client with an injury to the medulla. Which clinical manifestations should the nurse expect to find? (Select all that apply.) a. Loss of smell b. Impaired swallowing c. Visual changes d. Inability to shrug shoulders e. Loss of gag reflex

B, D, E Cranial nerves IX (glossopharyngeal), X (vagus), XI (accessory), and XII (hypoglossal) emerge from the medulla, as do portions of cranial nerves VII (facial) and VIII (acoustic). Damage to these nerves causes impaired swallowing, inability to shrug shoulders, and loss of the gag reflex. The other manifestations are not associated with damage to the medulla.

A nurse assesses a client who has encephalitis. Which manifestations should the nurse recognize as signs of increased intracranial pressure (ICP), a complication of encephalitis? (Select all that apply.) a. Photophobia b. Dilated pupils c. Headache d. Widened pulse pressure e. Bradycardia

B, D, E Increased ICP is a complication of encephalitis. The nurse should monitor for signs of increased ICP, including dilated pupils, widened pulse pressure, bradycardia, irregular respirations, and less responsive pupils. Photophobia and headache are not related to increased ICP.

A nurse is teaching a client who has chronic headaches. Which statements about headache triggers should the nurse include in this client's plan of care? (Select all that apply.) a. "Increase your intake of caffeinated beverages." b. "Incorporate physical exercise into your daily routine." c. "Avoid all alcoholic beverages." d. "Participate in a smoking cessation program." e. "Increase your intake of fruits and vegetables."

B, D, E Triggers for headaches include caffeine, smoking, and ingestion of pickled foods, so these factors should be avoided. Clients are taught to eat a balanced diet and to get adequate exercise and rest. Alcohol does not trigger chronic headaches but can enhance headaches during the headache period.

A nurse cares for a client with a lower motor neuron injury who is experiencing a flaccid bowel elimination pattern. Which actions should the nurse take to assist in relieving this client's constipation? (Select all that apply.) a. Pour warm water over the perineum. b. Provide a diet high in fluids and fiber. c. Administer daily tap water enemas. d. Implement a consistent daily time for elimination. e. Massage the abdomen from left to right. f. Perform manual disimpaction.

B, D, F For the client with a lower motor neuron injury, the resulting flaccid bowel may require a bowel program for the client that includes stool softeners, increased fluid intake, a high-fiber diet, and a consistent elimination time. If the client becomes impacted, the nurse would need to perform manual disimpaction. Pouring warm water over the perineum, administering daily enemas, and massaging the abdomen would not assist this client.

A nurse assesses an older client. Which assessment findings should the nurse identify as normal changes in the nervous system related to aging? (Select all that apply.) a. Long-term memory loss b. Slower processing time c. Increased sensory perception d. Decreased risk for infection e. Change in sleep patterns

B, E Normal changes in the nervous system related to aging include recent memory loss, slower processing time, decreased sensory perception, an increased risk for infection, changes in sleep patterns, changes in perception of pain, and altered balance and/or decreased coordination.

A nurse assesses a client who is experiencing an absence seizure. For which clinical manifestations should the nurse assess? (Select all that apply.) a. Intermittent rigidity b. Lip smacking c. Sudden loss of muscle tone d. Brief jerking of the extremities e. Picking at clothing f. Patting of the hand on the leg

B, E, F Automatisms are characteristic of absence seizures. These behaviors consist of lip smacking, picking at clothing, and patting. Rigidity of muscles is associated with the tonic phase of a seizure, and jerking of the extremities is associated with the clonic phase of a seizure. Loss of muscle tone occurs with atonic seizures.

A patient has informed the home health nurse that she has recently noticed distortions when she looks at the Amsler grid that she has mounted on her refrigerator. What is the nurse's most appropriate action? A) Reassure the patient that this is an age-related change in vision. B) Arrange for the patient to have her visual acuity assessed. C) Arrange for the patient to be assessed for macular degeneration. D) Facilitate tonometry testing.

C Feedback: 18, The Amsler grid is a test often used for patients with macular problems, such as macular degeneration. Distortions would not be attributed to age-related changes and there is no direct need for testing of intraocular pressure or visual acuity.

A patient has been diagnosed with serous otitis media for the third time in the past year. How should the nurse best interpret this patient's health status? A) For some patients, these recurrent infections constitute an age-related physiologic change. B) The patient would benefit from a temporary mobility restriction to facilitate healing. C) The patient needs to be assessed for nasopharyngeal cancer. D) Blood cultures should be drawn to rule out a systemic infection.

C Feedback: A carcinoma (e.g., nasopharyngeal cancer) obstructing the eustachian tube should be ruled out in adults with persistent unilateral serous otitis media. This phenomenon is not an age-related change and does not indicate a systemic infection. Mobility limitations are unnecessary.

A school nurse is assessing a student who was kicked in the shin during a soccer game. The area of the injury has become swollen and discolored. The triage nurse recognizes that the patient has likely sustained what? A) Sprain B) Strain C) Contusion D) Dislocation

C Feedback: A contusion is a soft-tissue injury that results in bleeding into soft tissues, creating a hematoma and ecchymosis. A sprain is an injury to ligaments caused by wrenching or twisting. A strain is a muscle pull from overuse, overstretching, or excessive stress. A dislocation is a condition in which the articular surfaces of the bones forming a joint are no longer in anatomic contact. Because the injury is not at the site of a joint, the patient has not experienced a sprain, strain, or dislocation.

A child goes to the school nurse and complains of not being able to hear the teacher. What test could the school nurse perform that would preliminarily indicate hearing loss? A) Audiometry B) Rinne test C) Whisper test D) Weber test

C Feedback: A general estimate of hearing can be made by assessing the patient's ability to hear a whispered phrase or a ticking watch, testing one ear at a time. The Rinne and Weber tests distinguish sensorineural from conductive hearing loss. These tests, as well as audiometry, are not usually performed by a registered nurse in a general practice setting.

A patient is admitted to the neurologic ICU with a C4 spinal cord injury. When writing the plan of care for this patient, which of the following nursing diagnoses would the nurse prioritize in the immediate care of this patient? A) Risk for impaired skin integrity related to immobility and sensory loss B) Impaired physical mobility related to loss of motor function C) Ineffective breathing patterns related to weakness of the intercostal muscles D) Urinary retention related to inability to void spontaneously

C Feedback: A nursing diagnosis related to breathing pattern would be the priority for this patient. A C4 spinal cord injury will require ventilatory support, due to the diaphragm and intercostals being affected. The other nursing diagnoses would be used in the care plan, but not designated as a higher priority than ineffective breathing patterns.

A patient has had an ischemic stroke and has been admitted to the medical unit. What action should the nurse perform to best prevent joint deformities? A) Place the patient in the prone position for 30 minutes/day. B) Assist the patient in acutely flexing the thigh to promote movement. C) Place a pillow in the axilla when there is limited external rotation. D) Place patient's hand in pronation.

C Feedback: A pillow in the axilla prevents adduction of the affected shoulder and keeps the arm away from the chest. The prone position with a pillow under the pelvis, not flat, promotes hyperextension of the hip joints, essential for normal gait. To promote venous return and prevent edema, the upper thigh should not be flexed acutely. The hand is placed in slight supination, not pronation, which is its most functional position.

The nurse is caring for a patient who underwent a total hip replacement yesterday. What should the nurse do to prevent dislocation of the new prosthesis? A) Keep the affected leg in a position of adduction. B) Have the patient reposition himself independently. C) Protect the affected leg from internal rotation. D) Keep the hip flexed by placing pillows under the patient's knee.

C Feedback: Abduction of the hip helps to prevent dislocation of a new hip joint. Rotation and adduction should be avoided. While the hip may be flexed slightly, it shouldn't exceed 90 degrees and maintenance of flexion isn't necessary. The patient may not be capable of safe independent repositioning at this early stage of recovery.

When administering a patient's eye drops, the nurse recognizes the need to prevent absorption by the nasolacrimal duct. How can the nurse best achieve this goal? A) Ensure that the patient is well hydrated at all times. B) Encourage self-administration of eye drops. C) Occlude the puncta after applying the medication. D) Position the patient supine before administering eye drops.

C Feedback: Absorption of eye drops by the nasolacrimal duct is undesirable because of the potential systemic side effects of ocular medications. To diminish systemic absorption and minimize the side effects, it is important to occlude the puncta. Self-administration, supine positioning, and adequate hydration do not prevent this adverse effect.

A community health nurse is giving an educational presentation about stroke and heart disease at the local senior citizens center. What nonmodifiable risk factor for stroke should the nurse cite? A) Female gender B) Asian American race C) Advanced age D) Smoking

C Feedback: Advanced age, male gender, and race are well-known nonmodifiable risk factors for stroke. High-risk groups include people older than 55 years of age; the incidence of stroke more than doubles in each successive decade. Men have a higher rate of stroke than that of women. Another high-risk group is African Americans; the incidence of first stroke in African Americans is almost twice that as in Caucasian Americans; Asian American race is not a risk factor. Smoking is a modifiable risk.

A nurse is caring for a patient with a diagnosis of cancer that has metastasized. What laboratory value would the nurse expect to be elevated in this patient? A) Bilirubin B) Potassium C) Alkaline phosphatase D) Creatinine

C Feedback: Alkaline phosphatase is elevated during early fracture healing and in diseases with increased osteoblastic activity (e.g., metastatic bone tumors). Elevated bilirubin, potassium, and creatinine would not be expected in a patient with metastatic bone tumors.

A nurse is planning the care of a patient with osteomyelitis that resulted from a diabetic foot ulcer. The patient requires a transmetatarsal amputation. When planning the patient's postoperative care, which of the following nursing diagnoses should the nurse most likely include in the plan of care? A) Ineffective Thermoregulation B) Risk-Prone Health Behavior C) Disturbed Body Image D) Deficient Diversion Activity

C Feedback: Amputations present a serious threat to any patient's body image. None of the other listed diagnoses is specifically associated with amputation.

The ED nurse is caring for a patient who has been brought in by ambulance after sustaining a fall at home. What physical assessment finding is suggestive of a basilar skull fracture? A) Epistaxis B) Periorbital edema C) Bruising over the mastoid D) Unilateral facial numbness

C Feedback: An area of ecchymosis (bruising) may be seen over the mastoid (Battle's sign) in a basilar skull fracture. Numbness, edema, and epistaxis are not directly associated with a basilar skull fracture.

A patient is brought to the ED by her family after falling off the roof. A family member tells the nurse that when the patient fell she was "knocked out," but came to and "seemed okay." Now she is complaining of a severe headache and not feeling well. The care team suspects an epidural hematoma, prompting the nurse to prepare for which priority intervention? A) Insertion of an intracranial monitoring device B) Treatment with antihypertensives C) Emergency craniotomy D) Administration of anticoagulant therapy

C Feedback: An epidural hematoma is considered an extreme emergency. Marked neurologic deficit or respiratory arrest can occur within minutes. Treatment consists of making an opening through the skull to decrease ICP emergently, remove the clot, and control the bleeding. Antihypertensive medications would not be a priority. Anticoagulant therapy should not be ordered for a patient who has a cranial bleed. This could further increase bleeding activity. Insertion of an intracranial monitoring device may be done during the surgery, but is not priority for this patient.

A patient has presented to the emergency department with an injury to the wrist. The patient is diagnosed with a third-degree strain. Why would the physician order an x-ray of the wrist? A) Nerve damage is associated with third-degree strains. B) Compartment syndrome is associated with third-degree strains. C) Avulsion fractures are associated with third-degree strains. D) Greenstick fractures are associated with third-degree strains.

C Feedback: An x-ray should be obtained to rule out bone injury, because an avulsion fracture (in which a bone fragment is pulled away from the bone by a tendon) may be associated with a third-degree strain. Nerve damage, compartment syndrome, and greenstick fractures are not associated with third-degree strains.

A patient has been experiencing progressive increases in knee pain and diagnostic imaging reveals a worsening effusion in the synovial capsule. The nurse should anticipate which of the following? A) Arthrography B) Knee biopsy C) Arthrocentesis D) Electromyography

C Feedback: Arthrocentesis (joint aspiration) is carried out to obtain synovial fluid for purposes of examination or to relieve pain due to effusion. Arthrography, biopsy, and electromyography would not remove fluid and relieve pressure.

A 56-year-old patient has come to the clinic for his routine eye examination and is told he needs bifocals. The patient asks the nurse what change in his eyes has caused his need for bifocals. How should the nurse respond? A) You know, you are getting older now and we change as we get older. B) The parts of our eyes age, just like the rest of us, and this is nothing to cause you to worry. C) There is a gradual thickening of the lens of the eye and it can limit the eye's ability for accommodation. D) The eye gets shorter, back to front, as we age and it changes how we see things.

C Feedback: As a result of a loss of accommodative power in the lens with age, many adults require bifocals or other forms of visual correction. This is not attributable to a change in the shape of the ocular globe. The nurse should not dismiss or downplay the patient's concerns.

An elderly patient's hip joint is immobilized prior to surgery to correct a femoral head fracture. What is the nurse's priority assessment? A) The presence of leg shortening B) The patient's complaints of pain C) Signs of neurovascular compromise D) The presence of internal or external rotation

C Feedback: Because impaired circulation can cause permanent damage, neurovascular assessment of the affected leg is always a priority assessment. Leg shortening and internal or external rotation are common findings with a fractured hip. Pain, especially on movement, is also common after a hip fracture.

A patient with a new diagnosis of ischemic stroke is deemed to be a candidate for treatment with tissue plasminogen activator (t-PA) and has been admitted to the ICU. In addition to closely monitoring the patient's cardiac and neurologic status, the nurse monitors the patient for signs of what complication? A) Acute pain B) Septicemia C) Bleeding D) Seizures

C Feedback: Bleeding is the most common side effect of t-PA administration, and the patient is closely monitored for any bleeding. Septicemia, pain, and seizures are much less likely to result from thrombolytic therapy.

A clinic nurse is caring for a patient with a history of osteoporosis. Which of the following diagnostic tests best allows the care team to assess the patient's risk of fracture? A) Arthrography B) Bone scan C) Bone densitometry D) Arthroscopy

C Feedback: Bone densitometry is used to detect bone density and can be used to assess the risk of fracture in osteoporosis. Arthrography is used to detect acute or chronic tears of joint capsule or supporting ligaments. Bone scans can be used to detect metastatic and primary bone tumors, osteomyelitis, certain fractures, and aseptic necrosis. Arthroscopy is used to visualize a joint.

A patient is admitted to the unit in traction for a fractured proximal femur and requires traction prior to surgery. What is the most appropriate type of traction to apply to a fractured proximal femur? A) Russell's traction B) Dunlop's traction C) Buck's extension traction D) Cervical head halter

C Feedback: Buck's extension is used for fractures of the proximal femur. Russell's traction is used for lower leg fractures. Dunlop's traction is applied to the upper extremity for supracondylar fractures of the elbow and humerus. Cervical head halters are used to stabilize the neck.

A nurse is reviewing the pathophysiology that may underlie a patient's decreased bone density. What hormone should the nurse identify as inhibiting bone resorption and promoting bone formation? A) Estrogen B) Parathyroid hormone (PTH) C) Calcitonin D) Progesterone

C Feedback: Calcitonin inhibits bone resorption and promotes bone formation, estrogen inhibits bone breakdown, and parathyroid increases bone resorption. Estrogen, which inhibits bone breakdown, decreases with aging. Parathyroid hormone (PTH) increases with aging, increasing bone turnover and resorption. Progesterone is the major naturally occurring human progestogen and plays a role in the female menstrual cycle.

Following a traumatic brain injury, a patient has been in a coma for several days. Which of the following statements is true of this patient's current LOC? A) The patient occasionally makes incomprehensible sounds. B) The patient's current LOC will likely become a permanent state. C) The patient may occasionally make nonpurposeful movements. D) The patient is incapable of spontaneous

C Feedback: Coma is a clinical state of unarousable unresponsiveness in which no purposeful responses to internal or external stimuli occur, although nonpurposeful responses to painful stimuli and brain stem reflexes may be present. Verbal sounds, however, are atypical. Ventilator support may or may not be necessary. Comas are not permanent states.

Which of the following nursing interventions would most likely facilitate effective communication with a hearing-impaired patient? A) Ask the patient to repeat what was said in order to evaluate understanding. B) Stand directly in front of the patient to facilitate lip reading. C) Reduce environmental noise and distractions before communicating. D) Raise the voice to project sound at a higher frequency.

C Feedback: Communication with the hearing impaired can be facilitated by talking in a quiet space free of competing noise stimuli and other distractions. Asking the patient to repeat what was said is likely to provoke frustration in the patient. A more effective strategy would be to repeat the question or statement, choosing different words. Raising the voice to project sound at higher frequency would make understanding more difficult. The nurse cannot assume that the patient reads lips. If the patient does read lips, on average he or she will understand only 50% of words accurately.

A patient is complaining of pain in her casted leg. The nurse has administered analgesics and elevated the limb. Thirty minutes after administering the analgesics, the patient states the pain is unrelieved. The nurse should identify the warning signs of what complication? A) Subcutaneous emphysema B) Skin breakdown C) Compartment syndrome D) Disuse syndrome

C Feedback: Compartment syndrome may manifest as unrelenting, uncontrollable pain. This presentation of pain is not suggestive of disuse syndrome or skin breakdown. Subcutaneous emphysema is not a complication of casting.

A patient with increased ICP has a ventriculostomy for monitoring ICP. The nurse's most recent assessment reveals that the patient is now exhibiting nuchal rigidity and photophobia. The nurse would be correct in suspecting the presence of what complication? A) Encephalitis B) CSF leak C) Meningitis D) Catheter occlusion

C Feedback: Complications of a ventriculostomy include ventricular infectious meningitis and problems with the monitoring system. Nuchal rigidity and photophobia are clinical manifestations of meningitis, but are not suggestive of encephalitis, a CSF leak, or an occluded catheter.

A nurse is discussing conservative management of tendonitis with a patient. Which of the following may be an effective approach to managing tendonitis? A) Weight reduction B) Use of oral opioid analgesics C) Intermittent application of ice and heat D) Passive range of motion exercises

C Feedback: Conservative management of tendonitis includes rest of the extremity, intermittent ice and heat to the joint, and NSAIDs. Weight reduction may prevent future injuries but will not relieve existing tendonitis. Range-of-motion exercises may exacerbate pain. Opioids would not be considered a conservative treatment measure.

A patient had a lumbar puncture performed at the outpatient clinic and the nurse has phoned the patient and family that evening. What does this phone call enable the nurse to determine? A) What are the patient's and family's expectations of the test B) Whether the patient's family had any questions about why the test was necessary C) Whether the patient has had any complications of the test D) Whether the patient understood accurately why the test was done

C Feedback: Contacting the patient and family after diagnostic testing enables the nurse to determine whether they have any questions about the procedure or whether the patient had any untoward results. The other listed information should have been elicited from the patient and family prior to the test.

When caring for a patient with an altered level of consciousness, the nurse is preparing to test cranial nerve VII. What assessment technique would the nurse use to elicit a response from cranial nerve VII? A) Palpate trapezius muscle while patient shrugs should against resistance. B) Administer the whisper or watch-tick test. C) Observe for facial movement symmetry, such as a smile. D) Note any hoarseness in the patient's voice.

C Feedback: Cranial nerve VII is the facial nerve. An appropriate assessment technique for this cranial nerve would include observing for symmetry while the patient performs facial movements: smiles, whistles, elevates eyebrows, and frowns. Palpating and noting strength of the trapezius muscle while the patient shrugs shoulders against resistance would be completed to assess cranial nerve XI (spinal accessory). Assessing cranial nerve VIII (acoustic) would involve using the whisper or watch-tick test to evaluate hearing. Noting any hoarseness in the patient's voice would involve assessment of cranial nerve X (vagus).

A nurse is caring for a patient who has suffered a hip fracture and who will require an extended hospital stay. The nurse should ensure that the patient does which of the following in order to prevent common complications associated with a hip fracture? A) Avoid requesting analgesia unless pain becomes unbearable. B) Use supplementary oxygen when transferring or mobilizing. C) Increase fluid intake and perform prescribed foot exercises. D) Remain on bed rest for 14 days or until instructed by the orthopedic surgeon.

C Feedback: Deep vein thrombosis (DVT) is among the most common complications related to a hip fracture. To prevent DVT, the nurse encourages intake of fluids and ankle and foot exercises. The patient should not be told to endure pain; a proactive approach to pain control should be adopted. While respiratory complications commonly include atelectasis and pneumonia, the use of deep-breathing exercises, changes in position at least every 2 hours, and the use of incentive spirometry help prevent the respiratory complications more than using supplementary oxygen. Bed rest may be indicated in the short term, but is not normally required for 14 days.

The nurse is discharging home a patient who suffered a stroke. He has a flaccid right arm and leg and is experiencing problems with urinary incontinence. The nurse makes a referral to a home health nurse because of an awareness of what common patient response to a change in body image? A) Denial B) Fear C) Depression D) Disassociation

C Feedback: Depression is a common and serious problem in the patient who has had a stroke. It can result from a profound disruption in his or her life and changes in total function, leaving the patient with a loss of independence. The nurse needs to encourage the patient to verbalize feelings to assess the effect of the stroke on self-esteem. Denial, fear, and disassociation are not the most common patient response to a change in body image, although each can occur in some patients.

A patient with low vision has called the clinic and asked the nurse for help with acquiring some low-vision aids. What else can the nurse offer to help this patient manage his low vision? A) The patient uses OTC NSAIDs. B) The patient has a history of stroke. C) The patient has diabetes. D) The patient has Asian ancestry.

C Feedback: Diabetes is a risk factor for glaucoma, but Asian ancestry, NSAIDs, and stroke are not risk factors for the disease.

A nurse is caring for a patient with a bone tumor. The nurse is providing education to help the patient reduce the risk for pathologic fractures. What should the nurse teach the patient? A) Strive to achieve maximum weight-bearing capabilities. B) Gradually strengthen the affected muscles through weight training. C) Support the affected extremity with external supports such as splints. D) Limit reliance on assistive devices in order to build strength.

C Feedback: During nursing care, the affected extremities must be supported and handled gently. External supports (splints) may be used for additional protection. Prescribed weight-bearing restrictions must be followed. Assistive devices should be used to strengthen the unaffected extremities.

A nurse is performing a shift assessment on an elderly patient who is recovering after surgery for a hip fracture. The nurse notes that the patient is complaining of chest pain, has an increased heart rate, and increased respiratory rate. The nurse further notes that the patient is febrile and hypoxic, coughing, and producing large amounts of thick, white sputum. The nurse recognizes that this is a medical emergency and calls for assistance, recognizing that this patient is likely demonstrating symptoms of what complication? A) Avascular necrosis of bone B) Compartment syndrome C) Fat embolism syndrome D) Complex regional pain syndrome

C Feedback: Fat embolism syndrome occurs most frequently in young adults and elderly patients who experience fractures of the proximal femur (i.e., hip fracture). Presenting features of fat embolism syndrome include hypoxia, tachypnea, tachycardia, and pyrexia. The respiratory distress response includes tachypnea, dyspnea, wheezes, precordial chest pain, cough, large amounts of thick, white sputum, and tachycardia. Avascular necrosis (AVN) occurs when the bone loses its blood supply and dies. This does not cause coughing. Complex regional pain syndrome does not have cardiopulmonary involvement.

The human body is designed to protect its vital parts. A fracture of what type of bone may interfere with the protection of vital organs? A) Long bones B) Short bones C) Flat bones D) Irregular bones

C Feedback: Flat bones, such as the sternum, provide vital organ protection. Fractures of the flat bones may lead to puncturing of the vital organs or may interfere with the protection of the vital organs. Long, short, and irregular bones do not usually have this physiologic function.

A patient diagnosed with a hemorrhagic stroke has been admitted to the neurologic ICU. The nurse knows that teaching for the patient and family needs to begin as soon as the patient is settled on the unit and will continue until the patient is discharged. What will family education need to include? A) How to differentiate between hemorrhagic and ischemic stroke B) Risk factors for ischemic stroke C) How to correctly modify the home environment D) Techniques for adjusting the patient's medication dosages at home

C Feedback: For a patient with a hemorrhagic stroke, teaching addresses the use of assistive devices or modification of the home environment to help the patient live with the disability. This is more important to the patient's needs than knowing about risk factors for ischemic stroke. It is not necessary for the family to differentiate between different types of strokes. Medication regimens should never be altered without consultation.

17. The nurse's comprehensive assessment of an older adult involves the assessment of the patient's gait. How should the nurse best perform this assessment? A) Instruct the patient to walk heel-to-toe for 15 to 20 steps. B) Instruct the patient to walk in a straight line while not looking at the floor. C) Instruct the patient to walk away from the nurse for a short distance and then toward the nurse. D) Instruct the patient to balance on one foot for as long as possible and then walk in a circle around the room.

C Feedback: Gait is assessed by having the patient walk away from the examiner for a short distance. The examiner observes the patient's gait for smoothness and rhythm. Looking at the floor is not disallowed and gait is not assessed by observing balance on one leg. Heel-to-toe walking ability is not gauged during an assessment of normal gait.

A patient has experienced a seizure in which she became rigid and then experienced alternating muscle relaxation and contraction. What type of seizure does the nurse recognize? A) Unclassified seizure B) Absence seizure C) Generalized seizure D) Focal seizure

C Feedback: Generalized seizures often involve both hemispheres of the brain, causing both sides of the body to react. Intense rigidity of the entire body may occur, followed by alternating muscle relaxation and contraction (generalized tonic-clonic contraction). This pattern of rigidity does not occur in patients who experience unclassified, absence, or focal seizures.

A patient who has been on long-term phenytoin (Dilantin) therapy is admitted to the unit. In light of the adverse of effects of this medication, the nurse should prioritize which of the following in the patient's plan of care? A) Monitoring of pulse oximetry B) Administration of a low-protein diet C) Administration of thorough oral hygiene D) Fluid restriction as ordered

C Feedback: Gingival hyperplasia (swollen and tender gums) can be associated with long-term phenytoin (Dilantin) use. Thorough oral hygiene should be provided consistently and encouraged after discharge. Fluid and protein restriction are contraindicated and there is no particular need for constant oxygen saturation monitoring.

The nurse is caring for a patient with a brain tumor. What drug would the nurse expect to be ordered to reduce the edema surrounding the tumor? A) Solumedrol B) Dextromethorphan C) Dexamethasone D) Furosemide

C Feedback: If a brain tumor is the cause of the increased ICP, corticosteroids (e.g., dexamethasone) help reduce the edema surrounding the tumor. Solumedrol, a steroid, and furosemide, a loop diuretic, are not the drugs of choice in this instance. Dextromethorphan is used in cough medicines.

The nurse is caring for a patient diagnosed with an ischemic stroke and knows that effective positioning of the patient is important. Which of the following should be integrated into the patient's plan of care? A) The patient's hip joint should be maintained in a flexed position. B) The patient should be in a supine position unless ambulating. C) The patient should be placed in a prone position for 15 to 30 minutes several times a day. D) The patient should be placed in a Trendelenberg position two to three times daily to promote cerebral perfusion.

C Feedback: If possible, the patient is placed in a prone position for 15 to 30 minutes several times a day. A small pillow or a support is placed under the pelvis, extending from the level of the umbilicus to the upper third of the thigh. This helps to promote hyperextension of the hip joints, which is essential for normal gait, and helps prevent knee and hip flexion contractures. The hip joints should not be maintained in flexion and the Trendelenberg position is not indicated.

Paramedics have brought an intubated patient to the RD following a head injury due to acceleration-deceleration motor vehicle accident. Increased ICP is suspected. Appropriate nursing interventions would include which of the following? A) Keep the head of the bed (HOB) flat at all times. B) Teach the patient to perform the Valsalva maneuver. C) Administer benzodiazepines on a PRN basis. D) Perform endotracheal suctioning every hour.

C Feedback: If the patient with a brain injury is very agitated, benzodiazepines are the most commonly used sedatives and do not affect cerebral blood flow or ICP. The HOB should be elevated 30 degrees. Suctioning should be done a limited basis, due to increasing the pressure in the cranium. The Valsalva maneuver is to be avoided. This also causes increased ICP.

A patient is admitted to the neurologic ICU with a spinal cord injury. When assessing the patient the nurse notes there is a sudden depression of reflex activity in the spinal cord below the level of injury. What should the nurse suspect? A) Epidural hemorrhage B) Hypertensive emergency C) Spinal shock D) Hypovolemia

C Feedback: In spinal shock, the reflexes are absent, BP and heart rate fall, and respiratory failure can occur. Hypovolemia, hemorrhage, and hypertension do not cause this sudden change in neurologic function.

A patient has come to the orthopedic clinic for a follow-up appointment 6 weeks after fracturing his ankle. Diagnostic imaging reveals that bone union is not taking place. What factor may have contributed to this complication? A) Inadequate vitamin D intake B) Bleeding at the injury site C) Inadequate immobilization D) Venous thromboembolism (VTE)

C Feedback: Inadequate fracture immobilization can delay or prevent union. A short-term vitamin D deficiency would not likely prevent bone union. VTE is a serious complication but would not be a cause of nonunion. Similarly, bleeding would not likely delay union.

A nurse is caring for a patient who has been scheduled for a bone scan. What should the nurse teach the patient about this diagnostic test? A) The test is brief and requires that you drink a calcium solution 2 hours before the test. B) You will not be allowed fluid for 2 hours before and 3 hours after the test. C) You'll be encouraged to drink water after the administration of the radioisotope injection. D) This is a common test that can be safely performed on anyone.

C Feedback: It is important to encourage the patient to drink plenty of fluids to help distribute and eliminate the isotopic after it is injected. There are important contraindications to the procedure, include pregnancy or an allergy to the radioisotope. The test requires the injection of an intravenous radioisotope and the scan is preformed 2 to 3 hours after the isotope is injected. A calcium solution is not utilized.

A 91-year-old patient is slated for orthopedic surgery and the nurse is integrated gerontologic considerations into the patient's plan of care. What intervention is most justified in the care of this patient? A) Administration of prophylactic antibiotics B) Total parenteral nutrition (TPN) C) Use of a pressure-relieving mattress D) Use of a Foley catheter until discharge

C Feedback: Older adults have a heightened risk of skin breakdown; use of a pressure-reducing mattress addresses this risk. Older adults do not necessarily need TPN and the Foley catheter should be discontinued as soon as possible to prevent urinary tract infections. Prophylactic antibiotics are not a standard infection prevention measure.

A 20 year-old is brought in by ambulance to the emergency department after being involved in a motorcycle accident. The patient has an open fracture of his tibia. The wound is highly contaminated and there is extensive soft-tissue damage. How would this patient's fracture likely be graded? A) Grade I B) Grade II C) Grade III D) Grade IV

C Feedback: Open fractures are graded according to the following criteria. Grade I is a clean wound less than 1 cm long. Grade II is a larger wound without extensive soft-tissue damage. Grade III is highly contaminated, has extensive soft-tissue damage, and is the most severe. There is no grade IV fracture.

Six weeks after an above-the-knee amputation (AKA), a patient returns to the outpatient office for a routine postoperative checkup. During the nurse's assessment, the patient reports symptoms of phantom pain. What should the nurse tell the patient to do to reduce the discomfort of the phantom pain? A) Apply intermittent hot compresses to the area of the amputation. B) Avoid activity until the pain subsides. C) Take opioid analgesics as ordered. D) Elevate the level of the amputation site.

C Feedback: Opioid analgesics may be effective in relieving phantom pain. Heat, immobility, and elevation are not noted to relieve this form of pain.

The nurse is teaching a patient to care for her new ocular prosthesis. What should the nurse emphasize during the patient's health education? A) The need to limit exposure to bright light B) The need to maintain a low Fowler's position when removing the prosthesis C) The need to perform thorough hand hygiene before handling the prosthesis D) The need to apply antiviral ointment to the prosthesis daily

C Feedback: Proper hand hygiene must be observed before inserting and removing an ocular prosthesis. There is no need for a low Fowler's position or for limiting light exposure. Antiviral ointments are not routinely used.

The clinic nurse is assessing a child who has been brought to the clinic with signs and symptoms that are suggestive of otitis externa. What assessment finding is characteristic of otitis externa? A) Tophi on the pinna and ear lobe B) Dark yellow cerumen in the external auditory canal C) Pain on manipulation of the auricle D) Air bubbles visible in the middle ear

C Feedback: Pain when the nurse pulls gently on the auricle in preparation for an otoscopic examination of the ear canal is a characteristic finding in patients with otitis externa. Tophi are deposits of generally painless uric acid crystals; they are a common physical assessment finding in patients diagnosed with gout. Cerumen is a normal finding during assessment of the ear canal. Its presence does not necessarily indicate that inflammation is present. Air bubbles in the middle ear may be visualized with the otoscope; however, these do not indicate a problem involving the ear canal.

The nurse recognizes that a patient with a SCI is at risk for muscle spasticity. How can the nurse best prevent this complication of an SCI? A) Position the patient in a high Fowler's position when in bed. B) Support the knees with a pillow when the patient is in bed. C) Perform passive ROM exercises as ordered. D) Administer NSAIDs as ordered.

C Feedback: Passive ROM exercises can prevent muscle spasticity following SCI. NSAIDs are not used for this purpose. Pillows and sitting upright do not directly address the patient's risk of muscle spasticity.

A patient is admitted to the neurologic ICU with a spinal cord injury. In writing the patient's care plan, the nurse specifies that contractures can best be prevented by what action? A) Repositioning the patient every 2 hours B) Initiating range-of-motion exercises (ROM) as soon as the patient initiates C) Initiating (ROM) exercises as soon as possible after the injury D) Performing ROM exercises once a day

C Feedback: Passive ROM exercises should be implemented as soon as possible after injury. It would be inappropriate to wait for the patient to first initiate exercises. Toes, metatarsals, ankles, knees, and hips should be put through a full ROM at least four, and ideally five, times daily. Repositioning alone will not prevent contractures.

A patient is being discharged home after mastoid surgery. What topic should the nurse address in the patient's discharge education? A) Expected changes in facial nerve function B) The need for audiometry testing every 6 months following recovery C) Safe use of analgesics and antivertiginous agents D) Appropriate use of OTC ear drops

C Feedback: Patients require instruction about medication therapy, such as analgesics and antivertiginous agents (e.g., antihistamines) prescribed for balance disturbance. OTC ear drops are not recommended and changes in facial nerve function are signs of a complication that needs to be addressed promptly. There is no need for serial audiometry testing.

A nurse is caring for a patient with Paget's disease and is reviewing the patient's most recent laboratory values. Which of the following values is most characteristic of Paget's disease? A) An elevated level of parathyroid hormone and low calcitonin levels B) A low serum alkaline phosphatase level and a low serum calcium level C) An elevated serum alkaline phosphatase level and a normal serum calcium level D) An elevated calcitonin level and low levels of parathyroid hormone

C Feedback: Patients with Paget's disease have normal blood calcium levels. Elevated serum alkaline phosphatase concentration and urinary hydroxyproline excretion reflect the increased osteoblastic activity associated with this condition. Alterations in PTH and calcitonin levels are atypical.

A patient is scheduled for CT scanning of the head because of a recent onset of neurologic deficits. What should the nurse tell the patient in preparation for this test? A) "No metal objects can enter the procedure room." B) "You need to fast for 8 hours prior to the test." C) "You will need to lie still throughout the procedure." D) "There will be a lot of noise during the test."

C Feedback: Preparation for CT scanning includes teaching the patient about the need to lie quietly throughout the procedure. If the patient were having an MRI, metal and noise would be appropriate teaching topics. There is no need to fast prior to a CT scan of the brain.

A nurse is taking a health history on a new patient who has been experiencing unexplained paresthesia. What question should guide the nurse's assessment of the patient's altered sensations? A) How does the strength in the affected extremity compare to the strength in the unaffected extremity? B) Does the color in the affected extremity match the color in the unaffected extremity? C) How does the feeling in the affected extremity compare with the feeling in the unaffected extremity? D) Does the patient have a family history of paresthesia or other forms of altered sensation?

C Feedback: Questions that the nurse should ask regarding altered sensations include How does this feeling compare to sensation in the unaffected extremity? Asking questions about strength and color are not relevant and a family history is unlikely.

The nurse is reviewing the medication administration record of a female patient who possesses numerous risk factors for stroke. Which of the woman's medications carries the greatest potential for reducing her risk of stroke? A) Naproxen 250 PO b.i.d. B) Calcium carbonate 1,000 mg PO b.i.d. C) Aspirin 81 mg PO o.d. D) Lorazepam 1 mg SL b.i.d. PRN

C Feedback: Research findings suggest that low-dose aspirin may lower the risk of stroke in women who are at risk. Naproxen, lorazepam, and calcium supplements do not have this effect.

A patient presents at a clinic complaining of back pain that goes all the way down the back of the leg to the foot. The nurse should document the presence of what type of pain? A) Bursitis B) Radiculopathy C) Sciatica D) Tendonitis

C Feedback: Sciatica nerve pain travels down the back of the thigh to the foot of the affected leg. Bursitis is inflammation of a fluid-filled sac in a joint. Radiculopathy is disease of a nerve root. Tendonitis is inflammation of muscle tendons.

A nurse's assessment of a teenage girl reveals that her shoulders are not level and that she has one prominent scapula that is accentuated by bending forward. The nurse should expect to read about what health problem in the patient's electronic health record? A) Lordosis B) Kyphosis C) Scoliosis D) Muscular dystrophy

C Feedback: Scoliosis is evidenced by an abnormal lateral curve in the spine, shoulders that are not level, an asymmetric waistline, and a prominent scapula, accentuated by bending forward. Lordosis is the curvature in the lower back; kyphosis is an exaggerated curvature of the upper back. This finding is not suggestive of muscular dystrophy.

A nurse is caring for a patient who has a leg cast. The nurse observes that the patient uses a pencil to scratch the skin under the edge of the cast. How should the nurse respond to this observation? A) Allow the patient to continue to scratch inside the cast with a pencil but encourage him to be cautious. B) Give the patient a sterile tongue depressor to use for scratching instead of the pencil. C) Encourage the patient to avoid scratching, and obtain an order for an antihistamine if severe itching persists. D) Obtain an order for a sedative, such as lorazepam (Ativan), to prevent the patient from scratching.

C Feedback: Scratching should be discouraged because of the risk for skin breakdown or damage to the cast. Most patients can be discouraged from scratching if given a mild antihistamine, such as diphenhydramine, to relieve itching. Benzodiazepines would not be given for this purpose.

A patient has been diagnosed with hearing loss related to damage of the end organ for hearing or cranial nerve VIII. What term is used to describe this condition? A) Exostoses B) Otalgia C) Sensorineural hearing loss D) Presbycusis

C Feedback: Sensorineural hearing loss is loss of hearing related to damage of the end organ for hearing or cranial nerve VIII. Exostoses refer to small, hard, bony protrusions in the lower posterior bony portion of the ear canal. Otalgia refers to a sensation of fullness or pain in the ear. Presbycusis is the term used to refer to the progressive hearing loss associated with aging. Both middle and inner ear age-related changes result in hearing loss.

During the performance of the Romberg test, the nurse observes that the patient sways slightly. What is the nurse's most appropriate action? A) Facilitate a referral to a neurologist. B) Reposition the patient supine to ensure safety. C) Document successful completion of the assessment. D) Follow up by having the patient perform the Rinne test.

C Feedback: Slight swaying during the Romberg test is normal, but a loss of balance is abnormal and is considered a positive Romberg test. Slight swaying is not a significant threat to the patient's safety. The Rinne test assesses hearing, not balance.

A young patient is being treated for a femoral fracture suffered in a snowboarding accident. The nurse's most recent assessment reveals that the patient is uncharacteristically confused. What diagnostic test should be performed on this patient? A) Electrolyte assessment B) Electrocardiogram C) Arterial blood gases D) Abdominal ultrasound

C Feedback: Subtle personality changes, restlessness, irritability, or confusion in a patient who has sustained a fracture are indications for immediate arterial blood gas studies due to the possibility of fat embolism syndrome. This assessment finding does not indicate an immediate need for electrolyte levels, an ECG, or abdominal ultrasound.

A patient is having a "fight or flight response" after receiving bad news about his prognosis. What affect will this have on the patient's sympathetic nervous system? A) Constriction of blood vessels in the heart muscle B) Constriction of bronchioles C) Increase in the secretion of sweat D) Constriction of pupils

C Feedback: Sympathetic nervous system stimulation results in dilated blood vessels in the heart and skeletal muscle, dilated bronchioles, increased secretion of sweat, and dilated pupils.

A trauma patient was admitted to the ICU with a brain injury. The patient had a change in level of consciousness, increased vital signs, and became diaphoretic and agitated. The nurse should recognize which of the following syndromes as the most plausible cause of these symptoms? A) Adrenal crisis B) Hypothalamic collapse C) Sympathetic storm D) Cranial nerve deficit

C Feedback: Sympathetic storm is a syndrome associated with changes in level of consciousness, altered vital signs, diaphoresis, and agitation that may result from hypothalamic stimulation of the sympathetic nervous system following traumatic brain injury. Alterations in cranial nerve or adrenal function would not have this result.

The nurse caring for an 80 year-old patient knows that she has a pre-existing history of dulled tactile sensation. The nurse should first consider what possible cause for this patient's diminished tactile sensation? A) Damage to cranial nerve VIII B) Adverse medication effects C) Age-related neurologic changes D) An undiagnosed cerebrovascular accident in early adulthood

C Feedback: Tactile sensation is dulled in the elderly person due to a decrease in the number of sensory receptors. While thorough assessment is necessary, it is possible that this change is unrelated to pathophysiological processes.

The nurse is caring for a patient in the ICU who has a brain stem herniation and who is exhibiting an altered level of consciousness. Monitoring reveals that the patient's mean arterial pressure (MAP) is 60 mm Hg with an intracranial pressure (ICP) reading of 5 mm Hg. What is the nurse's most appropriate action? A) Position the patient in the high Fowler's position as tolerated. B) Administer osmotic diuretics as ordered. C) Participate in interventions to increase cerebral perfusion pressure. D) Prepare the patient for craniotomy.

C Feedback: The cerebral perfusion pressure (CPP) is 55 mm Hg, which is considered low. The normal CPP is 70 to 100 mm Hg. Patients with a CPP of less than 50 mm Hg experience irreversible neurologic damage. As a result, interventions are necessary. A craniotomy is not directly indicated. Diuretics and increased height of bed would exacerbate the patient's condition.

Splints have been ordered for a patient who is at risk of developing footdrop following a spinal cord injury. The nurse caring for this patient knows that the splints are removed and reapplied when? A) At the patient's request B) Each morning and evening C) Every 2 hours D) One hour prior to mobility exercises

C Feedback: The feet are prone to footdrop; therefore, various types of splints are used to prevent footdrop. When used, the splints are removed and reapplied every 2 hours.

A 6-month-old infant is brought to the ED by his parents for inconsolable crying and pulling at his right ear. When assessing this infant, the advanced practice nurse is aware that the tympanic membrane should be what color in a healthy ear? A) Yellowish-white B) Pink C) Gray D) Bluish-white

C Feedback: The healthy tympanic membrane appears pearly gray and is positioned obliquely at the base of the ear canal. Any other color is suggestive of a pathological process.

A patient in the OR goes into malignant hyperthermia due to an abnormal reaction to the anesthetic. The nurse knows that the area of the brain that regulates body temperature is which of the following? A) Cerebellum B) Thalamus C) Hypothalamus D) Midbrain

C Feedback: The hypothalamus plays an important role in the endocrine system because it regulates the pituitary secretion of hormones that influence metabolism, reproduction, stress response, and urine production. It works with the pituitary to maintain fluid balance through hormonal release and maintains temperature regulation by promoting vasoconstriction or vasodilatation. The cerebellum, thalamus, and midbrain and not directly involved in temperature regulation.

The pathophysiology of an ischemic stroke involves the ischemic cascade, which includes the following steps: 1. Change in pH 2. Blood flow decreases 3. A switch to anaerobic respiration 4. Membrane pumps fail 5. Cells cease to function 6. Lactic acid is generated Put these steps in order in which they occur. A) 635241 B) 352416 C) 236145 D) 162534

C Feedback: The ischemic cascade begins when cerebral blood flow decreases to less than 25 mL per 100 g of blood per minute. At this point, neurons are no longer able to maintain aerobic respiration. The mitochondria must then switch to anaerobic respiration, which generates large amounts of lactic acid, causing a change in the pH. This switch to the less efficient anaerobic respiration also renders the neuron incapable of producing sufficient quantities of adenosine triphosphate (ATP) to fuel the depolarization processes. The membrane pumps that maintain electrolyte balances begin to fail, and the cells cease to function.

A patient injured in a motor vehicle accident has sustained a fracture to the diaphysis of the right femur. Of what is the diaphysis of the femur mainly constructed? A) Epiphyses B) Cartilage C) Cortical bone D) Cancellous bone

C Feedback: The long bone shaft, which is referred to as the diaphysis, is constructed primarily of cortical bone.

A nurse assesses clients at a community center. Which client is at greatest risk for lower back pain? a. A 24-year-old female who is 25 weeks pregnant b. A 36-year-old male who uses ergonomic techniques c. A 45-year-old male with osteoarthritis d. A 53-year-old female who uses a walker

C Osteoarthritis causes changes to support structures, increasing the client's risk for low back pain. The other clients are not at high risk.

A patient diagnosed with transient ischemic attacks (TIAs) is scheduled for a carotid endarterectomy. The nurse explains that this procedure will be done for what purpose? A) To decrease cerebral edema B) To prevent seizure activity that is common following a TIA C) To remove atherosclerotic plaques blocking cerebral flow D) To determine the cause of the TIA

C Feedback: The main surgical procedure for select patients with TIAs is carotid endarterectomy, the removal of an atherosclerotic plaque or thrombus from the carotid artery to prevent stroke in patients with occlusive disease of the extracranial arteries. An endarterectomy does not decrease cerebral edema, prevent seizure activity, or determine the cause of a TIA.

The results of a nurse's musculoskeletal examination show an increase in the lumbar curvature of the spine. The nurse should recognize the presence of what health problem? A) Osteoporosis B) Kyphosis C) Lordosis D) Scoliosis

C Feedback: The nurse documents the spinal abnormality as lordosis. Lordosis is an increase in lumbar curvature of the spine. Kyphosis is an increase in the convex curvature of the spine. Scoliosis is a lateral curvature of the spine. Osteoporosis is the significant loss of bone mass and strength with an increased risk for fracture.

he nurse has identified the diagnosis of Risk for Impaired Tissue Perfusion Related to Deep Vein Thrombosis in the care of a patient receiving skeletal traction. What nursing intervention best addresses this risk? A) Encourage independence with ADLs whenever possible. B) Monitor the patient's nutritional status closely. C) Teach the patient to perform ankle and foot exercises within the limitations of traction. D) Administer clopidogrel (Plavix) as ordered.

C Feedback: The nurse educates the patient how to perform ankle and foot exercises within the limits of the traction therapy every 1 to 2 hours when awake to prevent DVT. Nutrition is important, but does not directly prevent DVT. Similarly, independence with ADLs should be promoted, but this does not confer significant prevention of DVT, which often affects the lower limbs. Plavix is not normally used for DVT prophylaxis.

A patient is being admitted to the neurologic ICU following an acute head injury that has resulted in cerebral edema. When planning this patient's care, the nurse would expect to administer what priority medication? A) Hydrochlorothiazide (HydroDIURIL) B) Furosemide (Lasix) C) Mannitol (Osmitrol) D) Spirolactone (Aldactone)

C Feedback: The osmotic diuretic mannitol is given to dehydrate the brain tissue and reduce cerebral edema. This drug acts by reducing the volume of brain and extracellular fluid. Spirolactone, furosemide, and hydrochlorothiazide are diuretics that are not typically used in the treatment of increased ICP resulting from cerebral edema.

The nurse is preparing health education for a patient who is being discharged after hospitalization for a hemorrhagic stroke. What content should the nurse include in this education? A) Mild, intermittent seizures can be expected. B) Take ibuprofen for complaints of a serious headache. C) Take antihypertensive medication as ordered. D) Drowsiness is normal for the first week after discharge.

C Feedback: The patient and family are provided with information that will enable them to cooperate with the care and restrictions required during the acute phase of hemorrhagic stroke and to prepare the patient to return home. Patient and family teaching includes information about the causes of hemorrhagic stroke and its possible consequences. Symptoms of hydrocephalus include gradual onset of drowsiness and behavioral changes. Hypertension is the most serious risk factor, suggesting that appropriate antihypertensive treatment is essential for a patient being discharged. Seizure activity is not normal; complaints of a serious headache should be reported to the physician before any medication is taken. Drowsiness is not normal or expected.

A patient is being treated for a fractured hip and the nurse is aware of the need to implement interventions to prevent muscle wasting and other complications of immobility. What intervention best addresses the patient's need for exercise? A) Performing gentle leg lifts with both legs B) Performing massage to stimulate circulation C) Encouraging frequent use of the overbed trapeze D) Encouraging the patient to log roll side to side once per hour

C Feedback: The patient is encouraged to exercise as much as possible by means of the overbed trapeze. This device helps strengthen the arms and shoulders in preparation for protected ambulation. Independent logrolling may result in injury due to the location of the fracture. Leg lifts would be contraindicated for the same reason. Massage by the nurse is not a substitute for exercise.

A nurse is providing care for a patient who has osteomalacia. What major goal will guide the choice of medical and nursing interventions? A) Maintenance of skin integrity B) Prevention of bone metastasis C) Maintenance of adequate levels of activated vitamin D D) Maintenance of adequate parathyroid hormone function

C Feedback: The primary defect in osteomalacia is a deficiency of activated vitamin D, which promotes calcium absorption from the gastrointestinal tract and facilitates mineralization of bone. Interventions are aimed at resolving the processes underlying this deficiency. Maintenance of skin integrity is important, but is not the primary goal in care. Osteomalacia is not a malignant process. Overproduction (not underproduction) of PTH can cause the disease.

Cytomegalovirus (CMV) is the most common cause of retinal inflammation in patients with AIDS. What drug, surgically implanted, is used for the acute stage of CMV retinitis? A) Pilocarpine B) Penicillin C) Ganciclovir D) Gentamicin

C Feedback: The surgically implanted sustained-release insert of ganciclovir enables higher concentrations of ganciclovir to reach the CMV retinitis. Pilocarpine is a muscarinic agent used in open-angle glaucoma. Gentamicin and penicillin are antibiotics that are not used to treat CMV retinitis.

A neurologic nurse is reviewing seizures with a group of staff nurses. How should this nurse best describe the cause of a seizure? A) Sudden electrolyte changes throughout the brain B) A dysrhythmia in the peripheral nervous system C) A dysrhythmia in the nerve cells in one section of the brain D) Sudden disruptions in the blood flow throughout the brain

C Feedback: The underlying cause of a seizure is an electrical disturbance (dysrhythmia) in the nerve cells in one section of the brain; these cells emit abnormal, recurring, uncontrolled electrical discharges. Seizures are not caused by changes in blood flow or electrolytes.

The nurse planning the care of a patient with head injuries is addressing the patient's nursing diagnosis of "sleep deprivation." What action should the nurse implement? A) Administer a benzodiazepine at bedtime each night. B) Do not disturb the patient between 2200 and 0600. C) Cluster overnight nursing activities to minimize disturbances. D) Ensure that the patient does not sleep during the day.

C Feedback: To allow the patient longer times of uninterrupted sleep and rest, the nurse can group nursing care activities so that the patient is disturbed less frequently. However, it is impractical and unsafe to provide no care for an 8-hour period. The use of benzodiazepines should be avoided.

A nurse is caring for a patient in skeletal traction. In order to prevent bony fragments from moving against one another, the nurse should caution the patient against which of the following actions? A) Shifting one's weight in bed B) Bearing down while having a bowel movement C) Turning from side to side D) Coughing without splinting

C Feedback: To prevent bony fragments from moving against one another, the patient should not turn from side to side; however, the patient may shift position slightly with assistance. Bearing down and coughing do not pose a threat to bone union.

The nurse caring for a patient with a spinal cord injury notes that the patient is exhibiting early signs and symptoms of disuse syndrome. Which of the following is the most appropriate nursing action? A) Limit the amount of assistance provided with ADLs. B) Collaborate with the physical therapist and immobilize the patient's extremities temporarily. C) Increase the frequency of ROM exercises. D) Educate the patient about the importance of frequent position changes.

C Feedback: To prevent disuse syndrome, ROM exercises must be provided at least four times a day, and care is taken to stretch the Achilles tendon with exercises. The patient is repositioned frequently and is maintained in proper body alignment whether in bed or in a wheelchair. The patient must be repositioned by caregivers, not just taught about repositioning. It is inappropriate to limit assistance for the sole purpose of preventing disuse syndrome.

A patient with a spinal cord injury has experienced several hypotensive episodes. How can the nurse best address the patient's risk for orthostatic hypotension? A) Administer an IV bolus of normal saline prior to repositioning. B) Maintain bed rest until normal BP regulation returns. C) Monitor the patient's BP before and during position changes. D) Allow the patient to initiate repositioning.

C Feedback: To prevent hypotensive episodes, close monitoring of vital signs before and during position changes is essential. Prolonged bed rest carries numerous risks and it is not possible to provide a bolus before each position change. Following the patient's lead may or may not help regulate BP.

A nurse is admitting a patient with a severe migraine headache and a history of acute coronary syndrome. What migraine medication would the nurse question for this patient? A) Rizatriptan (Maxalt) B) Naratriptan (Amerge) C) Sumatriptan succinate (Imitrex) D) Zolmitriptan (Zomig)

C Feedback: Triptans can cause chest pain and are contraindicated in patients with ischemic heart disease. Maxalt, Amerge, and Zomig are triptans used in routine clinical use for the treatment of migraine headaches.

The nurse is caring for a patient with an upper motor neuron lesion. What clinical manifestations should the nurse anticipate when planning the patient's neurologic assessment? A) Decreased muscle tone B) Flaccid paralysis C) Loss of voluntary control of movement D) Slow reflexes

C Feedback: Upper motor neuron lesions do not cause muscle atrophy, flaccid paralysis, or slow reflexes. However, upper motor neuron lesions normally cause loss of voluntary control.

A nurse is collaborating with the interdisciplinary team to help manage a patient's recurrent headaches. What aspect of the patient's health history should the nurse identify as a potential contributor to the patient's headaches? A) The patient leads a sedentary lifestyle. B) The patient takes vitamin D and calcium supplements. C) The patient takes vasodilators for the treatment of angina. D) The patient has a pattern of weight loss followed by weight gain.

C Feedback: Vasodilators are known to contribute to headaches. Weight fluctuations, sedentary lifestyle, and vitamin supplements are not known to have this effect.

The nurse is assessing a patient with multiple sclerosis who is demonstrating involuntary, rhythmic eye movements. What term will the nurse use when documenting these eye movements? A) Vertigo B) Tinnitus C) Nystagmus D) Astigmatism

C Feedback: Vertigo is an illusion of movement where the individual or the surroundings are sensed as moving. Tinnitus refers to a subjective perception of sound with internal origin. Nystagmus refers to involuntary rhythmic eye movement. Astigmatism is a defect is visual acuity.

The nurse is planning the care of a patient with a diagnosis of vertigo. What nursing diagnosis risk should the nurse prioritize in this patient's care? A) Risk for disturbed sensory perception B) Risk for unilateral neglect C) Risk for falls D) Risk for ineffective health maintenance

C Feedback: Vertigo is defined as the misperception or illusion of motion, either of the person or the surroundings. A patient suffering from vertigo will be at an increased risk of falls. For most patients, this is likely to exceed the patient's risk for neglect, ineffective health maintenance, or disturbed sensation.

A nurse is assessing reflexes in a patient with hyperactive reflexes. When the patient's foot is abruptly dorsiflexed, it continues to "beat" two to three times before settling into a resting position. How would the nurse document this finding? A) Rigidity B) Flaccidity C) Clonus D) Ataxia

C Feedback: When reflexes are very hyperactive, a phenomenon called clonus may be elicited. If the foot is abruptly dorsiflexed, it may continue to "beat" two to three times before it settles into a position of rest. Rigidity is an increase in muscle tone at rest characterized by increased resistance to passive stretch. Flaccidity is lack of muscle tone. Ataxia is the inability to coordinate muscle movements, resulting in difficulty walking, talking, and performing self-care activities.

A patient has just arrived to the floor after an enucleation procedure following a workplace accident in which his left eye was irreparably damaged. Which of the following should the nurse prioritize during the patient's immediate postoperative recovery? A) Teaching the patient about options for eye prostheses B) Teaching the patient to estimate depth and distance with the use of one eye C) Assessing and addressing the patient's emotional needs D) Teaching the patient about his post-discharge medication regimen

C Feedback: When surgical eye removal is unexpected, such as in severe ocular trauma, leaving no time for the patient and family to prepare for the loss, the nurse's role in providing emotional support is crucial. In the short term, this is a priority over teaching regarding prostheses, medications, or vision adaptation.

An older adult patient has been diagnosed with macular degeneration and the nurse is assessing him for changes in visual acuity since his last clinic visit. When assessing the patient for recent changes in visual acuity, the patient states that he sees the lines on an Amsler grid as being distorted. What is the nurse's most appropriate response? A) Ask if the patient has been using OTC vasoconstrictors. B) Instruct the patient to repeat the test at different times of the day when at home. C) Arrange for the patient to visit his ophthalmologist. D) Encourage the patient to adhere to his prescribed drug regimen.

C Feedback: With a change in the patient's perception of the grid, the patient should notify the ophthalmologist immediately and should arrange to be seen promptly. This is a priority over encouraging drug adherence, even though this is also important. Vasoconstrictors are not a likely cause of this change and repeating the test at different times is not relevant.

A nurse cares for several clients on a neurologic unit. Which prescription for a client should direct the nurse to ensure that an informed consent has been obtained before the test or procedure? a. Sensation measurement via the pinprick method b. Computed tomography of the cranial vault c. Lumbar puncture for cerebrospinal fluid sampling d. Venipuncture for autoantibody analysis

C A lumbar puncture is an invasive procedure with many potentially serious complications. The other assessments or tests are considered noninvasive and do not require an informed consent.

A nurse is caring for a young male client with lymphoma who is to begin treatment. What teaching topic is a priority? a. Genetic testing b. Infection prevention c. Sperm banking d. Treatment options

C All teaching topics are important to the client with lymphoma, but for a young male, sperm banking is of particular concern if the client is going to have radiation to the lower abdomen or pelvis.

A nurse assesses a client with Alzheimer's disease who is recently admitted to the hospital. Which psychosocial assessment should the nurse complete? a. Assess religious and spiritual needs while in the hospital. b. Identify the client's ability to perform self-care activities. c. Evaluate the client's reaction to a change of environment. d. Ask the client about relationships with family members.

C As Alzheimer's disease progresses, the client experiences changes in emotional and behavioral affect. The nurse should be alert to the client's reaction to a change in environment, such as being hospitalized, because the client may exhibit an exaggerated response, such as aggression, to the event. The other assessments should be completed but are not as important as assessing the client's reaction to environmental change.

A nurse is teaching a client with chronic migraine headaches. Which statement related to complementary therapy should the nurse include in this client's teaching? a. "Place a warm compress on your forehead at the onset of the headache." b. "Wear dark sunglasses when you are in brightly lit spaces." c. "Lie down in a darkened room when you experience a headache." d. "Set your alarm to ensure you do not sleep longer than 6 hours at one time."

C At the onset of a migraine attack, the client may be able to alleviate pain by lying down and darkening the room. He or she may want both eyes covered and a cool cloth on the forehead. If the client falls asleep, he or she should remain undisturbed until awakening. The other options are not recognized therapies for migraines.

A nurse is teaching a client with cerebellar function impairment. Which statement should the nurse include in this client's discharge teaching? a. "Connect a light to flash when your door bell rings." b. "Label your faucet knobs with hot and cold signs." c. "Ask a friend to drive you to your follow-up appointments." d. "Use a natural gas detector with an audible alarm."

C Cerebellar function enables the client to predict distance or gauge the speed with which one is approaching an object, control voluntary movement, maintain equilibrium, and shift from one skilled movement to another in an orderly sequence. A client who has cerebellar function impairment should not be driving. The client would not have difficulty hearing, distinguishing between hot and cold, or smelling.

A nurse delegates care for a client with early-stage Alzheimer's disease to an unlicensed assistive personnel (UAP). Which statement should the nurse include when delegating this client's care? a. "If she is confused, play along and pretend that everything is okay." b. "Remove the clock from her room so that she doesn't get confused." c. "Reorient the client to the day, time, and environment with each contact." d. "Use validation therapy to recognize and acknowledge the client's concerns."

C Clients who have early-stage Alzheimer's disease should be reoriented frequently to person, place, and time. The UAP should reorient the client and not encourage the client's delusions. The room should have a clock and white board with the current date written on it. Validation therapy is used with late-stage Alzheimer's disease.

A nurse prepares to discharge a client with Alzheimer's disease. Which statement should the nurse include in the discharge teaching for this client's caregiver? a. "Allow the client to rest most of the day." b. "Place a padded throw rug at the bedside." c. "Install deadbolt locks on all outside doors." d. "Provide a high-calorie and high-protein diet."

C Clients with Alzheimer's disease have a tendency to wander, especially at night. If possible, alarms should be installed on all outside doors to alert family members if the client leaves. At a minimum, all outside doors should have deadbolt locks installed to prevent the client from going outdoors unsupervised. The client should be allowed to exercise within his or her limits. Throw rugs are a slip and fall hazard and should be removed. The client should eat a well-balanced diet. There is no need for a high-calorie or high-protein diet.

A nurse plans care for an 83-year-old client who is experiencing age-related sensory perception changes. Which intervention should the nurse include in this client's plan of care? a. Provide a call button that requires only minimal pressure to activate. b. Write the date on the client's white board to promote orientation. c. Ensure that the path to the bathroom is free from equipment. d. Encourage the client to season food to stimulate nutritional intake.

C Dementia and confusion are not common phenomena in older adults. However, physical impairment related to illness can be expected. Providing opportunities for hazard-free ambulation will maintain strength and mobility (and ensure safety). Providing a call button, providing the date, and seasoning food do not address the client's impaired sensory perception.

A nurse is teaching the daughter of a client who has Alzheimer's disease. The daughter asks, "Will the medication my mother is taking improve her dementia?" How should the nurse respond? a. "It will allow your mother to live independently for several more years." b. "It is used to halt the advancement of Alzheimer's disease but will not cure it." c. "It will not improve her dementia but can help control emotional responses." d. "It is used to improve short-term memory but will not improve problem solving."

C Drug therapy is not effective for treating dementia or halting the advancement of Alzheimer's disease. However, certain drugs may help suppress emotional disturbances and psychiatric manifestations. Medication therapy may not allow the client to safely live independently.

client has a sickle cell crisis with extreme lower extremity pain. What comfort measure does the nurse delegate to the unlicensed assistive personnel (UAP)? a. Apply ice packs to the client's legs. b. Elevate the client's legs on pillows. c. Keep the lower extremities warm. d. Place elastic bandage wraps on the client's legs.

C During a sickle cell crisis, the tissue distal to the occlusion has decreased blood flow and ischemia, leading to pain. Due to decreased blood flow, the client's legs will be cool or cold. The UAP can attempt to keep the client's legs warm. Ice and elevation will further decrease perfusion. Elastic bandage wraps are not indicated and may constrict perfusion in the legs.

A nurse assesses a client who has a history of migraines. Which clinical manifestation should the nurse identify as an early sign of a migraine with aura? a. Vertigo b. Lethargy c. Visual disturbances d. Numbness of the tongue

C Early warning of impending migraine with aura usually consists of visual changes, flashing lights, or diplopia. The other manifestations are not associated with an impending migraine with aura.

A nurse assesses a client with multiple sclerosis after administering prescribed fingolimod (Gilenya). For which adverse effect should the nurse monitor? a. Peripheral edema b. Black tarry stools c. Bradycardia d. Nausea and vomiting

C Fingolimod (Gilenya) is an antineoplastic agent that can cause bradycardia, especially within the first 6 hours after administration. Peripheral edema, black and tarry stools, and nausea and vomiting are not adverse effects of fingolimod.

A nurse plans care for a client with lower back pain from a work-related injury. Which intervention should the nurse include in this client's plan of care? a. Encourage the client to stretch the back by reaching toward the toes. b. Massage the affected area with ice twice a day. c. Apply a heating pad for 20 minutes at least four times daily. d. Advise the client to avoid warm baths or showers.

C Heat increases blood flow to the affected area and promotes healing of injured nerves. Stretching and ice will not promote healing, and there is no need to avoid warm baths or showers.

A nurse asks a client to take deep breaths during an electroencephalography. The client asks, "Why are you asking me to do this?" How should the nurse respond? a. "Hyperventilation causes vascular dilation of cerebral arteries, which decreases electoral activity in the brain." b. "Deep breathing helps you to relax and allows the electroencephalograph to obtain a better waveform." c. "Hyperventilation causes cerebral vasoconstriction and increases the likelihood of seizure activity." d. "Deep breathing will help you to blow off carbon dioxide and decreases intracranial pressures."

C Hyperventilation produces cerebral vasoconstriction and alkalosis, which increases the likelihood of seizure activity. The client is asked to breathe deeply 20 to 30 times for 3 minutes. The other responses are not accurate.

A nurse is caring for a client with a history of renal insufficiency who is scheduled for a computed tomography scan of the head with contrast medium. Which priority intervention should the nurse implement? a. Educate the client about strict bedrest after the procedure. b. Place an indwelling urinary catheter to closely monitor output. c. Obtain a prescription for intravenous fluids. d. Contact the provider to cancel the procedure.

C If a contrast medium is used, intravenous fluid may be given to promote excretion of the contrast medium. Contrast medium also may act as a diuretic, resulting in the need for fluid replacement. The client will not require bedrest. Although urinary output should be monitored closely, there is no need for an indwelling urinary catheter. There is no need to cancel the procedure as long as actions are taken to protect the kidneys.

After teaching a client who is scheduled for magnetic resonance imaging (MRI), the nurse assesses the client's understanding. Which client statement indicates a correct understanding of the teaching? a. "I must increase my fluids because of the dye used for the MRI." b. "My urine will be radioactive so I should not share a bathroom." c. "I can return to my usual activities immediately after the MRI." d. "My gag reflex will be tested before I can eat or drink anything."

C No postprocedure restrictions are imposed after MRI. The client can return to normal activities after the test is complete. There are no dyes or radioactive materials used for the MRI; therefore, increased fluids are not needed and the client's urine would not be radioactive. The procedure does not impact the client's gag reflex.

A nurse teaches an 80-year-old client with diminished touch sensation. Which statement should the nurse include in this client's teaching? a. "Place soft rugs in your bathroom to decrease pain in your feet." b. "Bathe in warm water to increase your circulation." c. "Look at the placement of your feet when walking." d. "Walk barefoot to decrease pressure ulcers from your shoes."

C Older clients with decreased sensation are at risk of injury from the inability to sense changes in terrain when walking. To compensate for this loss, the client is instructed to look at the placement of her or his feet when walking. Throw rugs can slip and increase fall risk. Bath water that is too warm places the client at risk for thermal injury. The client should wear sturdy shoes for ambulation.

A nurse is caring for a client with paraplegia who is scheduled to participate in a rehabilitation program. The client states, "I do not understand the need for rehabilitation; the paralysis will not go away and it will not get better." How should the nurse respond? a. "If you don't want to participate in the rehabilitation program, I'll let the provider know." b. "Rehabilitation programs have helped many clients with your injury. You should give it a chance." c. "The rehabilitation program will teach you how to maintain the functional ability you have and prevent further disability." d. "When new discoveries are made regarding paraplegia, people in rehabilitation programs will benefit first."

C Participation in rehabilitation programs has many purposes, including prevention of disability, maintenance of functional ability, and restoration of function. The other responses do not meet this client's needs.

A nurse is caring for a client who is about to receive a bone marrow transplant. To best help the client cope with the long recovery period, what action by the nurse is best? a. Arrange a visitation schedule among friends and family. b. Explain that this process is difficult but must be endured. c. Help the client find things to hope for each day of recovery. d. Provide plenty of diversionary activities for this time.

C Providing hope is an essential nursing function during treatment for any disease process, but especially during the recovery period after bone marrow transplantation, which can take up to 3 weeks. The nurse can help the client look ahead to the recovery period and identify things to hope for during this time. Visitors are important to clients, but may pose an infection risk. Telling the client the recovery period must be endured does not acknowledge his or her feelings. Diversionary activities are important, but not as important as instilling hope.

A nurse assesses a client with a brain tumor. The client opens his eyes when the nurse calls his name, mumbles in response to questions, and follows simple commands. How should the nurse document this client's assessment using the Glasgow Coma Scale shown below? a. 8 b. 10 c. 12 d. 14

C The client opens his eyes to speech (Eye opening: To sound = 3), mumbles in response to questions (Verbal response: Inappropriate words = 3), and follows simple commands (Motor response: Obeys commands = 6). Therefore, the client's Glasgow Coma Scale score is: 3 + 3 + 6 = 12.

A client has thrombocytopenia. What client statement indicates the client understands self-management of this condition? a. "I brush and use dental floss every day." b. "I chew hard candy for my dry mouth." c. "I usually put ice on bumps or bruises." d. "Nonslip socks are best when I walk."

C The client should be taught to apply ice to areas of minor trauma. Flossing is not recommended. Hard foods should be avoided. The client should wear well-fitting shoes when ambulating.

A nurse cares for a client with a spinal cord injury. With which interdisciplinary team member should the nurse consult to assist the client with activities of daily living? a. Social worker b. Physical therapist c. Occupational therapist d. Case manager

C The occupational therapist instructs the client in the correct use of all adaptive equipment. In collaboration with the therapist, the nurse instructs family members or the caregiver about transfer skills, feeding, bathing, dressing, positioning, and skin care. The other team members are consulted to assist the client with unrelated issues.

A client has been treated for a deep vein thrombus and today presents to the clinic with petechiae. Laboratory results show a platelet count of 42,000/mm3. The nurse reviews the client's medication list to determine if the client is taking which drug? a. Enoxaparin (Lovenox) b. Salicylates (aspirin) c. Unfractionated heparin d. Warfarin (Coumadin)

C This client has manifestations of heparin-induced thrombocytopenia. Enoxaparin, salicylates, and warfarin do not cause this condition.

A nurse assesses a client who is recovering from the implantation of a vagal nerve stimulation device. For which clinical manifestations should the nurse assess as common complications of this procedure? (Select all that apply.) a. Bleeding b. Infection c. Hoarseness d. Dysphagia e. Seizures

C, D Complications of surgery to implant a vagal nerve stimulation device include hoarseness (most common), dyspnea, neck pain, and dysphagia. The device is tunneled under the skin with an electrode connected to the vagus nerve to control simple or complex partial seizures. Bleeding is not a common complication of this procedure, and infection would not occur during the recovery period.

A patient is admitted to the orthopedic unit with a fractured femur after a motorcycle accident. The patient has been placed in traction until his femur can be rodded in surgery. For what early complications should the nurse monitor this patient? Select all that apply. A) Systemic infection B) Complex regional pain syndrome C) Deep vein thrombosis D) Compartment syndrome E) Fat embolism

C, D, E Feedback: Early complications include shock, fat embolism, compartment syndrome, and venous thromboemboli (deep vein thrombosis [DVT], pulmonary embolism [PE]). Infection and CRPS are later complications of fractures.

A nurse assesses a client who is recovering from a lumbar laminectomy. Which complications should alert the nurse to urgently communicate with the health care provider? (Select all that apply.) a. Surgical discomfort b. Redness and itching at the incision site c. Incisional bulging d. Clear drainage on the dressing e. Sudden and severe headache

C, D, E Bulging at the incision site or clear fluid on the dressing after a laminectomy strongly suggests a cerebrospinal fluid leak, which constitutes an emergency. Loss of cerebral spinal fluid may cause a sudden and severe headache, which is also an emergency situation. Pain, redness, and itching at the site are normal.

A client has received a bone marrow transplant and is waiting for engraftment. What actions by the nurse are most appropriate? (Select all that apply.) a. Not allowing any visitors until engraftment b. Limiting the protein in the client's diet c. Placing the client in protective precautions d. Teaching visitors appropriate hand hygiene e. Telling visitors not to bring live flowers or plants

C, D, E The client waiting for engraftment after bone marrow transplant has no white cells to protect him or her against infection. The client is on protective precautions and visitors are taught hand hygiene. No fresh flowers or plants are allowed due to the standing water in the vase or container that may harbor organisms. Limiting protein is not a healthy option and will not promote engraftment.

A nurse is caring for a patient who has had a plaster arm cast applied. Immediately postapplication, the nurse should provide what teaching to the patient? A) The cast will feel cool to touch for the first 30 minutes. B) The cast should be wrapped snuggly with a towel until the patient gets home. C) The cast should be supported on a board while drying. D) The cast will only have full strength when dry.

D Feedback: A cast requires approximately 24 to 72 hours to dry, and until dry, it does not have full strength. While drying, the cast should not be placed on a hard surface. The cast will exude heat while it dries and should not be wrapped.

A patient diagnosed with a cerebral aneurysm reports a severe headache to the nurse. What action is a priority for the nurse? A) Sit with the patient for a few minutes. B) Administer an analgesic. C) Inform the nurse-manager. D) Call the physician immediately.

D Feedback: A headache may be an indication that the aneurysm is leaking. The nurse should notify the physician immediately. The physician will decide whether administration of an analgesic is indicated. Informing the nurse-manager is not necessary. Sitting with the patient is appropriate, once the physician has been notified of the change in the patient's condition.

A nurse is assessing a child who has a diagnosis of muscular dystrophy. Assessment reveals that the child's muscles have greater-than-normal tone. The nurse should document the presence of which of the following? A) Tonus B) Flaccidity C) Atony D) Spasticity

D Feedback: A muscle with greater-than-normal tone is described as spastic. Soft and flabby muscle tone is defined as atony. A muscle that is limp and without tone is described as being flaccid. The state of readiness known as muscle tone (tonus) is produced by the maintenance of some of the muscle fibers in a contracted state.

A nurse is preparing to discharge an emergency department patient who has been fitted with a sling to support her arm after a clavicle fracture. What should the nurse instruct the patient to do? A) Elevate the arm above the shoulder 3 to 4 times daily. B) Avoid moving the elbow, wrist, and fingers until bone remodeling is complete. C) Engage in active range of motion using the affected arm. D) Use the arm for light activities within the range of motion.

D Feedback: A patient with a clavicle fracture may use a sling to support the arm and relieve the pain. The patient may be permitted to use the arm for light activities within the range of comfort. The patient should not elevate the arm above the shoulder level until the ends of the bones have united, but the nurse should encourage the patient to exercise the elbow, wrist, and fingers.

Which of the following nurse's actions carries the greatest potential to prevent hearing loss due to ototoxicity? A) Ensure that patients understand the differences between sensory hearing loss and conductive hearing loss. B) Educate patients about expected age-related changes in hearing perception. C) Educate patients about the risks associated with prolonged exposure to environmental noise. D) Be aware of patients' medication regimens and collaborate with other professionals accordingly

D Feedback: A variety of medications may have adverse effects on the cochlea, vestibular apparatus, or cranial nerve VIII. All but a few, such as aspirin and quinine, cause irreversible hearing loss. Ototoxicity is not related to age-related changes, noise exposure, or the differences between types of hearing loss.

A nurse is caring for a patient who has an MRI scheduled. What is the priority safety action prior to this diagnostic procedure? A) Assessing the patient for signs and symptoms of active infection B) Ensuring that the patient can remain immobile for up to 3 hours C) Assessing the patient for a history of nut allergies D) Ensuring that there are no metal objects on or in the patient

D Feedback: Absolutely no metal objects can be present during MRItheir presence constitutes a serious safety risk. The procedure takes up to 90 minutes. Nut allergies and infection are not contraindications to MRI.

The ED is notified that a 6-year-old is in transit with a suspected brain injury after being struck by a car. The child is unresponsive at this time, but vital signs are within acceptable limits. What will be the primary goal of initial therapy? A) Promoting adequate circulation B) Treating the child's increased ICP C) Assessing secondary brain injury D) Preserving brain homeostasis

D Feedback: All therapy is directed toward preserving brain homeostasis and preventing secondary brain injury, which is injury to the brain that occurs after the original traumatic event. The scenario does not indicate the child has increased ICP or a secondary brain injury at this point. Promoting circulation is likely secondary to the broader goal of preserving brain homeostasis.

A nurse in the ICU is providing care for a patient who has been admitted with a hemorrhagic stroke. The nurse is performing frequent neurologic assessments and observes that the patient is becoming progressively more drowsy over the course of the day. What is the nurse's best response to this assessment finding? A) Report this finding to the physician as an indication of decreased metabolism. B) Provide more stimulation to the patient and monitor the patient closely. C) Recognize this as the expected clinical course of a hemorrhagic stroke. D) Report this to the physician as a possible sign of clinical deterioration.

D Feedback: Alteration in LOC often is the earliest sign of deterioration in a patient with a hemorrhagic stroke. Drowsiness and slight slurring of speech may be early signs that the LOC is deteriorating. This finding is unlikely to be the result of metabolic changes and it is not expected. Stimulating a patient with an acute stroke is usually contraindicated.

A 72-year-old man has been brought to his primary care provider by his daughter, who claims that he has been experiencing uncharacteristic lapses in memory. What principle should underlie the nurse's assessment and management of this patient? A) Loss of short-term memory is normal in older adults, but loss of long-term memory is pathologic. B) Lapses in memory in older adults are considered benign unless they have negative consequences. C) Gradual increases in confusion accompany the aging process. D) Thorough assessment is necessary because changes in cognition are always considered to be pathologic.

D Feedback: Although mental processing time decreases with age, memory, language, and judgment capacities remain intact. Change in mental status should never be assumed to be a normal part of aging.

A patient with a documented history of seizure disorder experiences a generalized seizure. What nursing action is most appropriate? A) Restrain the patient to prevent injury. B) Open the patient's jaws to insert an oral airway. C) Place patient in high Fowler's position. D) Loosen the patient's restrictive clothing.

D Feedback: An appropriate nursing intervention would include loosening any restrictive clothing on the patient. No attempt should be made to restrain the patient during the seizure because muscular contractions are strong and restraint can produce injury. Do not attempt to pry open jaws that are clenched in a spasm to insert anything. Broken teeth and injury to the lips and tongue may result from such an action. If possible, place the patient on one side with head flexed forward, which allows the tongue to fall forward and facilitates drainage of saliva and mucus.

The nurse is caring for a patient with increased intracranial pressure (ICP). The patient has a nursing diagnosis of "ineffective cerebral tissue perfusion." What would be an expected outcome that the nurse would document for this diagnosis? A) Copes with sensory deprivation. B) Registers normal body temperature. C) Pays attention to grooming. D) Obeys commands with appropriate motor responses.

D Feedback: An expected outcome of the diagnosis of ineffective cerebral tissue perfusion in a patient with increased intracranial pressure (ICP) would include obeying commands with appropriate motor responses. Vitals signs and neurologic status are assessed every 15 minutes to every hour. Coping with sensory deprivation would relate to the nursing diagnosis of "disturbed sensory perception." The outcome of "registers normal body temperature" relates to the diagnosis of "potential for ineffective thermoregulation." Body image disturbance would have a potential outcome of "pays attention to grooming."

The nurse is caring for a patient who has undergone supratentorial removal of a pituitary mass. What medication would the nurse expect to administer prophylactically to prevent seizures in this patient? A) Prednisone B) Dexamethasone C) Cafergot D) Phenytoin

D Feedback: Antiseizure medication (phenytoin, diazepam) is often prescribed prophylactically for patients who have undergone supratentorial craniotomy because of the high risk of seizures after this procedure. Prednisone and dexamethasone are steroids and do not prevent seizures. Cafergot is used in the treatment of migraines.

The nurse is providing care for a patient who is withdrawing from heavy alcohol use. The nurse and other members of the care team are present at the bedside when the patient has a seizure. In preparation for documenting this clinical event, the nurse should note which of the following? A) The ability of the patient to follow instructions during the seizure. B) The success or failure of the care team to physically restrain the patient. C) The patient's ability to explain his seizure during the postictal period. D) The patient's activities immediately prior to the seizure.

D Feedback: Before and during a seizure, the nurse observes the circumstances before the seizure, including visual, auditory, or olfactory stimuli; tactile stimuli; emotional or psychological disturbances; sleep; and hyperventilation. Communication with the patient is not possible during a seizure and physical restraint is not attempted. The patient's ability to explain the seizure is not clinically relevant.

A patient is being discharged home from the ambulatory surgical center after cataract surgery. In reviewing the discharge instructions with the patient, the nurse instructs the patient to immediately call the office if the patient experiences what? A) Slight morning discharge from the eye B) Any appearance of redness of the eye C) A scratchy feeling in the eye D) A new floater in vision

D Feedback: Cataract surgery increases the risk of retinal detachment and the patient must be instructed to notify the surgeon of new floaters in vision, flashing lights, decrease in vision, pain, or increase in redness. Slight morning discharge, some redness, and a scratchy feeling may be expected for a few days after surgery.

The neurologic nurse is testing the function of a patient's cerebellum and basal ganglia. What action will most accurately test these structures? A) Have the patient identify the location of a cotton swab on his or her skin with the eyes closed. B) Elicit the patient's response to a hypothetical problem. C) Ask the patient to close his or her eyes and discern between hot and cold stimuli. D) Guide the patient through the performance of rapid, alternating movements.

D Feedback: Cerebellar and basal ganglia influence on the motor system is reflected in balance control and coordination. Coordination in the hands and upper extremities is tested by having the patient perform rapid, alternating movements and point-to-point testing. The cerebellum and basal ganglia do not mediate cutaneous sensation or judgment.

The nursing students are learning how to assess function of cranial nerve VIII. To assess the function of cranial nerve VIII the students would be correct in completing which of the following assessment techniques? A) Have the patient identify familiar odors with the eyes closed. B) Assess papillary reflex. C) Utilize the Snellen chart. D) Test for air and bone conduction (Rinne test).

D Feedback: Cranial nerve VIII is the acoustic nerve. It functions in hearing and equilibrium. When assessing this nerve, the nurse would test for air and bone conduction (Rinne) with a tuning fork. Assessment of papillary reflex would be completed for cranial nerves III (oculomotor), IV (trochlear), and VI (abducens). The Snellen chart would be used to assess cranial nerve II (optic).

A nurse is performing a musculoskeletal assessment of a patient with arthritis. During passive range-of-motion exercises, the nurse hears an audible grating sound. The nurse should document the presence of which of the following? A) Fasciculations B) Clonus C) Effusion D) Crepitus

D Feedback: Crepitus is a grating, crackling sound or sensation that occurs as the irregular joint surfaces move across one another, as in arthritic conditions. Fasciculations are involuntary twitching of muscle fiber groups. Clonus is the rhythmic contractions of a muscle. Effusion is the collection of excessive fluid within the capsule of a joint.

What should the nurse suspect when hourly assessment of urine output on a patient postcraniotomy exhibits a urine output from a catheter of 1,500 mL for two consecutive hours? A) Cushing syndrome B) Syndrome of inappropriate antidiuretic hormone (SIADH) C) Adrenal crisis D) Diabetes insipidus

D Feedback: Diabetes insipidus is an abrupt onset of extreme polyuria that commonly occurs in patients after brain surgery. Cushing syndrome is excessive glucocorticoid secretion resulting in sodium and water retention. SIADH is the result of increased secretion of ADH; the patient becomes volume-overloaded, urine output diminishes, and serum sodium concentration becomes dilute. Adrenal crisis is undersecretion of glucocorticoids resulting in profound hypoglycemia, hypovolemia, and hypotension.

The nurse is reviewing the health history of a newly admitted patient and reads that the patient has been previously diagnosed with exostoses. How should the nurse accommodate this fact into the patient's plan of care? A) The nurse should perform the Rinne and Weber tests. B) The nurse should arrange for audiometry testing as soon as possible. C) The nurse should collaborate with the pharmacist to assess for potential ototoxic medications. D) No specific assessments or interventions are necessary to addressing exostoses.

D Feedback: Exostoses are small, hard, bony protrusions found in the lower posterior bony portion of the ear canal; they usually occur bilaterally. They do not normally impact hearing and no treatments or nursing actions are usually necessary.

The nurse's musculoskeletal assessment of a patient reveals involuntary twitching of muscle groups. How would the nurse document this observation in the patient's chart? A) Tetany B) Atony C) Clonus D) Fasciculations

D Feedback: Fasciculation is involuntary twitching of muscle fiber groups. Clonus is a series of involuntary, rhythmic, muscular contractions and tetany is involuntary muscle contraction, but neither is characterized as twitching. Atony is a loss of muscle strength.

Following a motorcycle accident, a 17-year-old man is brought to the ED. What physical assessment findings related to the ear should be reported by the nurse immediately? A) The malleus can be visualized during otoscopic examination. B) The tympanic membrane is pearly gray. C) Tenderness is reported by the patient when the mastoid area is palpated. D) Clear, watery fluid is draining from the patient's ear.

D Feedback: For the patient experiencing acute head trauma, immediately report the presence of clear, watery drainage from the ear. The fluid is likely to be cerebrospinal fluid associated with skull fracture. The ability to visualize the malleus is a normal physical assessment finding. The tympanic membrane is normally pearly gray in color. Tenderness of the mastoid area usually indicates inflammation. This should be reported, but is not a finding indicating urgent intervention.

Radiographs of a boy's upper arm show that the humerus appears to be fractured on one side and slightly bent on the other. This diagnostic result suggests what type of fracture? A) Impacted B) Compound C) Compression D) Greenstick

D Feedback: Greenstick fractures are an incomplete fracture that results in the bone being broken on one side, while the other side is bent. This is not characteristic of an impacted, compound, or compression fracture.

A patient with a sudden onset of hearing loss tells the nurse that he would like to begin using hearing aids. The nurse understands that the health professional dispensing hearing aids would have what responsibility? A) Test the patient's hearing promptly. B) Perform an otoscopy. C) Measure the width of the patient's ear canal. D) Refer the patient to his primary care physician.

D Feedback: Health care professionals who dispense hearing aids are required to refer prospective users to a physician if the patient has sudden or rapidly progressive hearing loss. This would be a health priority over other forms of assessment, due to the possible presence of a pathologic process.

The nurse has implemented interventions aimed at facilitating family coping in the care of a patient with a traumatic brain injury. How can the nurse best facilitate family coping? A) Help the family understand that the patient could have died. B) Emphasize the importance of accepting the patient's new limitations. C) Have the members of the family plan the patient's inpatient care. D) Assist the family in setting appropriate short-term goals.

D Feedback: Helpful interventions to facilitate coping include providing family members with accurate and honest information and encouraging them to continue to set well-defined, short-term goals. Stating that a patient's condition could be worse downplays their concerns. Emphasizing the importance of acceptance may not necessarily help the family accept the patient's condition. Family members cannot normally plan a patient's hospital care, although they may contribute to the care in some ways.

31. The public health nurse is addressing eye health and vision protection during an educational event. What statement by a participant best demonstrates an understanding of threats to vision? A) I'm planning to avoid exposure to direct sunlight on my next vacation. B) I've never exercised regularly, but I'm going to start working out at the gym daily. C) I'm planning to talk with my pharmacist to review my current medications. D) I'm certainly going to keep a close eye on my blood pressure from now on.ease.

D Feedback: Hypertension is a major cause of vision loss, exceeding the significance of inactivity, sunlight, and adverse effects of medications

An older adult patient sought care for the treatment of a swollen, painful knee joint. Diagnostic imaging and culturing of synovial fluid resulted in a diagnosis of septic arthritis. The nurse should prioritize which of the following aspects of care? A) Administration of oral and IV corticosteroids as ordered B) Prevention of falls and pathologic fractures C) Maintenance of adequate serum levels of vitamin D D) Intravenous administration of antibiotics

D Feedback: IV antibiotics are the major treatment modality for septic arthritis; the nurse must ensure timely administration of these drugs. Corticosteroids are not used to treat septic arthritis and vitamin D levels are not necessarily affected. Falls prevention is important, but septic arthritis does not constitute the same fracture risk as diseases with decreased bone density.

35. A child has been experiencing recurrent episodes of acute otitis media (AOM). The nurse should anticipate that what intervention is likely to be ordered? A) Ossiculoplasty B) Insertion of a cochlear implant C) Stapedectomy D) Insertion of a ventilation tube

D Feedback: If AOM recurs and there is no contraindication, a ventilating, or pressure-equalizing, tube may be inserted. The ventilating tube, which temporarily takes the place of the eustachian tube in equalizing pressure, is retained for 6 to 18 months. Ossiculoplasty is not used to treat AOM and stapedectomy is performed to treat otosclerosis. Cochlear implants are used to treat sensorineural hearing loss.

22. An emergency department patient is diagnosed with a hip dislocation. The patient's family is relieved that the patient has not suffered a hip fracture, but the nurse explains that this is still considered to be a medical emergency. What is the rationale for the nurse's statement? A) The longer the joint is displaced, the more difficult it is to get it back in place. B) The patient's pain will increase until the joint is realigned. C) Dislocation can become permanent if the process of bone remodeling begins. D) Avascular necrosis may develop at the site of the dislocation if it is not promptly resolved.

D Feedback: If a dislocation or subluxation is not reduced immediately, avascular necrosis (AVN) may develop. Bone remodeling does not take place because a fracture has not occurred. Realignment does not become more difficult with time and pain would subside with time, not become worse.

A 13-year-old was brought to the ED, unconscious, after being hit in the head by a baseball. When the child regains consciousness, 5 hours after being admitted, he cannot remember the traumatic event. MRI shows no structural sign of injury. What injury would the nurse suspect the patient has? A) Diffuse axonal injury B) Grade 1 concussion with frontal lobe involvement C) Contusion D) Grade 3 concussion with temporal lobe involvement

D Feedback: In a grade 3 concussion there is a loss of consciousness lasting from seconds to minutes. Temporal lobe involvement results in amnesia. Frontal lobe involvement can cause uncharacteristic behavior and a grade 1 concussion does not involve loss of consciousness. Diagnostic studies may show no apparent structural sign of injury, but the duration of unconsciousness is an indicator of the severity of the concussion. Diffuse axonal injury (DAI) results from widespread shearing and rotational forces that produce damage throughout the brain—to axons in the cerebral hemispheres, corpus callosum, and brain stem. In cerebral contusion, a moderate to severe head injury, the brain is bruised and damaged in a specific area because of severe acceleration-deceleration force or blunt trauma.

A patient presents at the clinic with complaints of morning numbness, cramping, and stiffness in his fourth and fifth fingers. What disease process should the nurse suspect? A) Tendonitis B) A ganglion C) Carpal tunnel syndrome D) Dupuytren's disease

D Feedback: In cases of Dupuytren's disease, the patient may experience dull, aching discomfort, morning numbness, cramping, and stiffness in the affected fingers. This condition starts in one hand, but eventually both hands are affected. This clinical scenario does not describe tendonitis, a ganglion, or carpal tunnel syndrome.

On otoscopy, a red blemish behind the tympanic membrane is suggestive of what diagnosis? A) Acoustic tumor B) Cholesteatoma C) Facial nerve neuroma D) Glomus tympanicum

D Feedback: In the case of glomus tympanicum, a red blemish on or behind the tympanic membrane is seen on otoscopy. This assessment finding is not associated with an acoustic tumor, facial nerve neuroma, or cholesteatoma.

A nurse is caring for a patient who is recovering in the hospital following orthopedic surgery. The nurse is performing frequent assessments for signs and symptoms of infection in the knowledge that the patient faces a high risk of what infectious complication? A) Cellulitis B) Septic arthritis C) Sepsis D) Osteomyelitis

D Feedback: Infection is a risk after any surgery, but it is of particular concern for the postoperative orthopedic patient because of the risk of osteomyelitis. Orthopedic patients do not have an exaggerated risk of cellulitis, sepsis, or septic arthritis when compared to other surgical patients.

A patient for whom the nurse is caring has positron emission tomography (PET) scheduled. In preparation, what should the nurse explain to the patient? A) The test will temporarily limit blood flow through the brain. B) An allergy to iodine precludes getting the radio-opaque dye. C) The patient will need to endure loud noises during the test. D) The test may result in dizziness or lightheadedness.

D Feedback: Key nursing interventions for PET scan include explaining the test and teaching the patient about inhalation techniques and the sensations (e.g., dizziness, light-headedness, and headache) that may occur. A PET scan does not impede blood flow through the brain. An allergy to iodine precludes the dye for an MRI, and loud noise is heard in an MRI.

An older adult patient has come to the clinic for a regular check-up. The nurse's initial inspection reveals an increased thoracic curvature of the patient's spine. The nurse should document the presence of which of the following? A) Scoliosis B) Epiphyses C) Lordosis D) Kyphosis

D Feedback: Kyphosis is the increase in thoracic curvature of the spine. Scoliosis is a deviation in the lateral curvature of the spine. Epiphyses are the ends of the long bones. Lordosis is the exaggerated curvature of the lumbar spine.

A patient is undergoing diagnostic testing for osteomalacia. Which of the following laboratory results is most suggestive of this diagnosis? A) High chloride, calcium, and magnesium B) High parathyroid and calcitonin levels C) Low serum calcium and magnesium levels D) Low serum calcium and low phosphorus level

D Feedback: Laboratory studies will reveal a low serum calcium and low phosphorus level.

A patient presents to the ED complaining of a sudden onset of incapacitating vertigo, with nausea and vomiting and tinnitus. The patient mentions to the nurse that she suddenly cannot hear very well. What would the nurse suspect the patient's diagnosis will be? A) Ossiculitis B) Ménière's disease C) Ototoxicity D) Labyrinthitis

D Feedback: Labyrinthitis is characterized by a sudden onset of incapacitating vertigo, usually with nausea and vomiting, various degrees of hearing loss, and possibly tinnitus. None of the other listed diagnosis is characterized by a rapid onset of symptoms.

A patient has a poor prognosis after being involved in a motor vehicle accident resulting in a head injury. As the patient's ICP increases and condition worsens, the nurse knows to assess for indications of approaching death. These indications include which of the following? A) Hemiplegia B) Dry mucous membranes C) Signs of internal bleeding D) Loss of brain stem reflexes

D Feedback: Loss of brain stem reflexes, including pupillary, corneal, gag, and swallowing reflexes, is an ominous sign of approaching death. Dry mucous membranes, hemiplegia, and bleeding must be promptly addressed, but none of these is a common sign of impending death.

The nurse is admitting a patient to the unit who is diagnosed with a lower motor neuron lesion. What entry in the patient's electronic record is most consistent with this diagnosis? A) "Patient exhibits increased muscle tone." B) "Patient demonstrates normal muscle structure with no evidence of atrophy." C) "Patient demonstrates hyperactive deep tendon reflexes." D) "Patient demonstrates an absence of deep tendon reflexes."

D Feedback: Lower motor neuron lesions cause flaccid muscle paralysis, muscle atrophy, decreased muscle tone, and loss of voluntary control.

A patient is scheduled for a total hip replacement and the surgeon has explained the risks of blood loss associated with orthopedic surgery. The risk of blood loss is the indication for which of the following actions? A) Use of a cardiopulmonary bypass machine B) Postoperative blood salvage C) Prophylactic blood transfusion D) Autologous blood donation

D Feedback: Many patients donate their own blood during the weeks preceding their surgery. Autologous blood donations are cost effective and eliminate many of the risks of transfusion therapy. Orthopedic surgery does not necessitate cardiopulmonary bypass and blood is not salvaged postoperatively. Transfusions are not given prophylactically.

A nurse is teaching preventative measures for otitis externa to a group of older adults. What action should the nurse encourage? A) Rinsing the ears with normal saline after swimming B) Avoiding loud environmental noises C) Instilling antibiotic ointments on a regular basis D) Avoiding the use of cotton swabs

D Feedback: Nurses should instruct patients not to clean the external auditory canal with cotton-tipped applicators and to avoid events that traumatize the external canal such as scratching the canal with the fingernail or other objects. Environmental noise should be avoided, but this does not address the risk for ear infection. Routine use of antibiotics is not encouraged and rinsing the ears after swimming is not recommended.

An orthopedic nurse is caring for a patient who is postoperative day one following foot surgery. What nursing intervention should be included in the patient's subsequent care? A) Dressing changes should not be performed unless there are clear signs of infection. B) The surgical site can be soaked in warm bath water for up to 5 minutes. C) The surgical site should be cleansed with hydrogen peroxide once daily. D) The foot should be elevated in order to prevent edema.

D Feedback: Pain experienced by patients who undergo foot surgery is related to inflammation and edema. To control the anticipated edema, the foot should be elevated on several pillows when the patient is sitting or lying. Regular dressing changes are performed and the wound should be kept dry. Hydrogen peroxide is not used to cleanse surgical wounds.

A patient with a diagnosis of retinal detachment has undergone a vitreoretinal procedure on an outpatient basis. What subject should the nurse prioritize during discharge education? A) Risk factors for postoperative cytomegalovirus (CMV) B) Compensating for vision loss for the next several weeks C) Non-pharmacologic pain management strategies D) Signs and symptoms of increased intraocular pressure

D Feedback: Patients must be educated about the signs and symptoms of complications, particularly of increasing IOP and postoperative infection. CMV is not a typical complication and the patient should not expect vision loss. Vitreoretinal procedures are not associated with high levels of pain.

A nurse on the neurologic unit is providing care for a patient who has spinal cord injury at the level of C4. When planning the patient's care, what aspect of the patient's neurologic and functional status should the nurse consider? A) The patient will be unable to use a wheelchair. B) The patient will be unable to swallow food. C) The patient will be continent of urine, but incontinent of bowel. D) The patient will require full assistance for all aspects of elimination.

D Feedback: Patients with a lesion at C4 are fully dependent for elimination. The patient is dependent for feeding, but is able to swallow. The patient will be capable of using an electric wheelchair.

A patient with chronic open-angle glaucoma is being taught to self-administer pilocarpine. After the patient administers the pilocarpine, the patient states that her vision is blurred. Which nursing action is most appropriate? A) Holding the next dose and notifying the physician B) Treating the patient for an allergic reaction C) Suggesting that the patient put on her glasses D) Explaining that this is an expected adverse effect

D Feedback: Pilocarpine, a miotic drug used to treat glaucoma, achieves its effect by constricting the pupil. Blurred vision lasting 1 to 2 hours after instilling the eye drops is an expected adverse effect. The patient may also note difficulty adapting to the dark. Because blurred vision is an expected adverse effect, the drug does not need to be withheld, nor does the physician need to be notified. Likewise, the patient does not need to be treated for an allergic reaction. Wearing glasses will not alter this temporary adverse effect.

A patient is ready to be discharged home after a cataract extraction with intraocular lens implant and the nurse is reviewing signs and symptoms that need to be reported to the ophthalmologist immediately. Which of the patient's statements best demonstrates an adequate understanding? A) I need to call the doctor if I get nauseated. B) I need to call the doctor if I have a light morning discharge. C) I need to call the doctor if I get a scratchy feeling. D) I need to call the doctor if I see flashing lights.

D Feedback: Postoperatively, the patient who has undergone cataract extraction with intraocular lens implant should report new floaters in vision, flashing lights, decrease in vision, pain, or increase in redness to the ophthalmologist. Slight morning discharge and a scratchy feeling can be expected for a few days. Blurring of vision may be experienced for several days to weeks.

During the examination of an unconscious patient, the nurse observes that the patient's pupils are fixed and dilated. What is the most plausible clinical significance of the nurse's finding? A) It suggests onset of metabolic problems. B) It indicates paralysis on the right side of the body. C) It indicates paralysis of cranial nerve X. D) It indicates an injury at the midbrain level.

D Feedback: Pupils that are fixed and dilated indicate injury at the midbrain level. This finding is not suggestive of unilateral paralysis, metabolic deficits, or damage to CN X.

A nurse is providing discharge education to a patient who is going home with a cast on his leg. What teaching point should the nurse emphasize in the teaching session? A) Using crutches efficiently B) Exercising joints above and below the cast, as ordered C) Removing the cast correctly at the end of the treatment period D) Reporting signs of impaired circulation

D Feedback: Reporting signs of impaired circulation is critical; signs of impaired circulation must be reported to the physician immediately to prevent permanent damage. For this reason, this education is a priority over exercise and crutch use. The patient does not independently remove the cast.

A nurse is reviewing the care of a patient who has a long history of lower back pain that has not responded to conservative treatment measures. The nurse should anticipate the administration of what drug? A) Calcitonin B) Prednisone C) Aspirin D) Cyclobenzaprine

D Feedback: Short-term prescription muscle relaxants (e.g., cyclobenzaprine [Flexeril]) are effective in relieving acute low back pain. ASA is not normally used for pain control, due to its antiplatelet action and associated risk for bleeding. Calcitonin and corticosteroids are not used in the treatment of lower back pain.

A patient is brought to the emergency department by ambulance after stepping in a hole and falling. While assessing him the nurse notes that his right leg is shorter than his left leg; his right hip is noticeably deformed and he is in acute pain. Imaging does not reveal a fracture. Which of the following is the most plausible explanation for this patient's signs and symptoms? A) Subluxated right hip B) Right hip contusion C) Hip strain D) Traumatic hip dislocation

D Feedback: Signs and symptoms of a traumatic dislocation include acute pain, change in positioning of the joint, shortening of the extremity, deformity, and decreased mobility. A subluxation would cause moderate deformity, or possibly no deformity. A contusion or strain would not cause obvious deformities.

23. A nurse is assessing the neurovascular status of a patient who has had a leg cast recently applied. The nurse is unable to palpate the patient's dorsalis pedis or posterior tibial pulse and the patient's foot is pale. What is the nurse's most appropriate action? A) Warm the patient's foot and determine whether circulation improves. B) Reposition the patient with the affected foot dependent. C) Reassess the patient's neurovascular status in 15 minutes. D) Promptly inform the primary care provider.

D Feedback: Signs of neurovascular dysfunction warrant immediate medical follow-up. It would be unsafe to delay. Warming the foot or repositioning the patient may be of some benefit, but the care provider should be informed first.

The public health nurse is planning a health promotion campaign that reflects current epidemiologic trends. The nurse should know that hemorrhagic stroke currently accounts for what percentage of total strokes in the United States? A) 43% B) 33% C) 23% D) 13%

D Feedback: Strokes can be divided into two major categories: ischemic (87%), in which vascular occlusion and significant hypoperfusion occur, and hemorrhagic (13%), in which there is extravasation of blood into the brain or subarachnoid space.

A patient is exploring treatment options after being diagnosed with age-related cataracts that affect her vision. What treatment is most likely to be used in this patient's care? A) Antioxidant supplements, vitamin C and E, beta-carotene, and selenium B) Eyeglasses or magnifying lenses C) Corticosteroid eye drops D) Surgical intervention

D Feedback: Surgery is the treatment option of choice when the patient's functional and visual status is compromised. No nonsurgical (medications, eye drops, eyeglasses) treatment cures cataracts or prevents age-related cataracts. Studies recently have found no benefit from antioxidant supplements, vitamins C and E, beta-carotene, or selenium. Corticosteroid eye drops are prescribed for use after cataract surgery; however, they increase the risk for cataracts if used long-term or in high doses. Eyeglasses and magnification may improve vision in the patient with early stages of cataracts, but have limitations for the patient with impaired functioning.

What should be included in the patient's care plan when establishing an exercise program for a patient affected by a stroke? A) Schedule passive range of motion every other day. B) Keep activity limited, as the patient may be over stimulated. C) Have the patient perform active range-of-motion (ROM) exercises once a day. D) Exercise the affected extremities passively four or five times a day.

D Feedback: The affected extremities are exercised passively and put through a full ROM four or five times a day to maintain joint mobility, regain motor control, prevent development of a contracture in the paralyzed extremity, prevent further deterioration of the neuromuscular system, and enhance circulation. Active ROM exercises should ideally be performed more than once per day.

After mastoid surgery, an 81-year-old patient has been identified as needing assistance in her home. What would be a primary focus of this patient's home care? A) Preparation of nutritious meals and avoidance of contraindicated foods B) Ensuring the patient receives adequate rest each day C) Helping the patient adapt to temporary hearing loss D) Assisting the patient with ambulation as needed to avoid falling

D Feedback: The caregiver and patient are cautioned that the patient may experience some vertigo and will therefore require help with ambulation to avoid falling. The patient should not be expected to experience hearing loss and no foods are contraindicated. Adequate rest is needed, but this is not a primary focus of home care.

An ED nurse has just received a call from EMS that they are transporting a 17-year-old man who has just sustained a spinal cord injury (SCI). The nurse recognizes that the most common cause of this type of injury is what? A) Sports-related injuries B) Acts of violence C) Injuries due to a fall D) Motor vehicle accidents

D Feedback: The most common causes of SCIs are motor vehicle crashes (46%), falls (22%), violence (16%), and sports (12%).

A patient who has had an amputation is being cared for by a multidisciplinary rehabilitation team. What is the primary goal of this multidisciplinary team? A) Maximize the efficiency of care B) Ensure that the patient's health care is holistic C) Facilitate the patient's adjustment to a new body image D) Promote the patient's highest possible level of function

D Feedback: The multidisciplinary rehabilitation team helps the patient achieve the highest possible level of function and participation in life activities. The team is not primarily motivated by efficiency, the need for holistic care, or the need to foster the patient's body image, despite the fact that each of these are valid goals.

A nurse is emptying an orthopedic surgery patient's closed suction drainage at the end of a shift. The nurse notes that the volume is within expected parameters but that the drainage has a foul odor. What is the nurse's best action? A) Aspirate a small amount of drainage for culturing. B) Advance the drain 1 to 1.5 cm. C) Irrigate the drain with normal saline. D) Inform the surgeon of this finding.

D Feedback: The nurse should promptly notify the surgeon of excessive or foul-smelling drainage. It would be inappropriate to advance the drain, irrigate the drain, or aspirate more drainage.

A patient is currently being stimulated by the parasympathetic nervous system. What effect will this nervous stimulation have on the patient's bladder? A) The parasympathetic nervous system causes urinary retention. B) The parasympathetic nervous system causes bladder spasms. C) The parasympathetic nervous system causes urge incontinence. D) The parasympathetic nervous system makes the bladder contract.

D Feedback: The parasympathetic division of the nervous system causes contraction (stimulation) of the urinary bladder muscles and a decrease (inhibition) in heart rate, whereas the sympathetic division produces relaxation (inhibition) of the urinary bladder and an increase (stimulation) in the rate and force of the heartbeat.

The nurse is discharging a patient home after mastoid surgery. What should the nurse include in discharge teaching? A) "Try to induce a sneeze every 4 hours to equalize pressure." B) "Be sure to exercise to reduce fatigue." C) "Avoid sleeping in a side-lying position." D) "Don't blow your nose for 2 to 3 weeks."

D Feedback: The patient is instructed to avoid heavy lifting, straining, exertion, and nose blowing for 2 to 3 weeks after surgery to prevent dislodging the tympanic membrane graft or ossicular prosthesis. Side-lying is not contraindicated; sneezing could cause trauma.

A patient with a cerebral aneurysm exhibits signs and symptoms of an increase in intracranial pressure (ICP). What nursing intervention would be most appropriate for this patient? A) Range-of-motion exercises to prevent contractures B) Encouraging independence with ADLs to promote recovery C) Early initiation of physical therapy D) Absolute bed rest in a quiet, nonstimulating environment

D Feedback: The patient is placed on immediate and absolute bed rest in a quiet, nonstressful environment because activity, pain, and anxiety elevate BP, which increases the risk for bleeding. Visitors are restricted. The nurse administers all personal care. The patient is fed and bathed to prevent any exertion that might raise BP.

After a subarachnoid hemorrhage, the patient's laboratory results indicate a serum sodium level of less than 126 mEq/L. What is the nurse's most appropriate action? A) Administer a bolus of normal saline as ordered. B) Prepare the patient for thrombolytic therapy as ordered. C) Facilitate testing for hypothalamic dysfunction. D) Prepare to administer 3% NaCl by IV as ordered.

D Feedback: The patient may be experiencing syndrome of inappropriate antidiuretic hormone (SIADH) or cerebral salt-wasting syndrome. The treatment most often is the use of IV hypertonic 3% saline. A normal saline bolus would exacerbate the problem and there is no indication for tests of hypothalamic function or thrombolytic therapy.

A nursing student is writing a care plan for a newly admitted patient who has been diagnosed with a stroke. What major nursing diagnosis should most likely be included in the patient's plan of care? A) Adult failure to thrive B) Post-trauma syndrome C) Hyperthermia D) Disturbed sensory perception

D Feedback: The patient who has experienced a stroke is at a high risk for disturbed sensory perception. Stroke is associated with multiple other nursing diagnoses, but hyperthermia, adult failure to thrive, and post-trauma syndrome are not among these.

A female patient is diagnosed with a right-sided stroke. The patient is now experiencing hemianopsia. How might the nurse help the patient manage her potential sensory and perceptional difficulties? A) Keep the lighting in the patient's room low. B) Place the patient's clock on the affected side. C) Approach the patient on the side where vision is impaired. D) Place the patient's extremities where she can see them.

D Feedback: The patient with homonymous hemianopsia (loss of half of the visual field) turns away from the affected side of the body and tends to neglect that side and the space on that side; this is called amorphosynthesis. In such instances, the patient cannot see food on half of the tray, and only half of the room is visible. It is important for the nurse to remind the patient constantly of the other side of the body, to maintain alignment of the extremities, and if possible, to place the extremities where the patient can see them. Patients with a decreased field of vision should be approached on the side where visual perception is intact. All visual stimuli (clock, calendar, and television) should be placed on this side. The patient can be taught to turn the head in the direction of the defective visual field to compensate for this loss. Increasing the natural or artificial lighting in the room and providing eyeglasses are important in increasing vision. There is no reason to keep the lights dim.

An elderly patient is being discharged home. The patient lives alone and has atrophy of his olfactory organs. The nurse tells the patient's family that it is essential that the patient have what installed in the home? A) Grab bars B) Nonslip mats C) Baseboard heaters D) A smoke detector

D Feedback: The sense of smell deteriorates with age. The olfactory organs are responsible for smell. This may present a safety hazard for the patient because he or she may not smell smoke or gas leaks. Smoke detectors are universally necessary, but especially for this patient.

What term is used to describe the fibrous connective tissue that hugs the brain closely and extends into every fold of the brain's surface? A) Dura mater B) Arachnoid C) Fascia D) Pia mater

D Feedback: The term "meninges" describes the fibrous connective tissue that covers the brain and spinal cord. The meninges have three layers, the dura mater, arachnoid, and pia mater. The pia mater is the innermost membrane that hugs the brain closely and extends into every fold of the brain's surface. The dura mater, the outermost layer, covers the brain and spinal cord. The arachnoid, the middle membrane, is responsible for the production of cerebrospinal fluid.

The registered nurse taking shift report learns that an assigned patient is blind. How should the nurse best communicate with this patient? A) Provide instructions in simple, clear terms. B) Introduce herself in a firm, loud voice at the doorway of the room. C) Lightly touch the patient's arm and then introduce herself. D) State her name and role immediately after entering the patient's room.

D Feedback: There are several guidelines to consider when interacting with a person who is blind or has low vision. Identify yourself by stating your name and role, before touching or making physical contact with the patient. When talking to the person, speak directly at him or her using a normal tone of voice. There is no need to raise your voice unless the person asks you to do so and there is no particular need to simplify verbal instructions.

A patient has had a cast placed for the treatment of a humeral fracture. The nurse's most recent assessment shows signs and symptoms of compartment syndrome. What is the nurse's most appropriate action? A) Arrange for a STAT assessment of the patient's serum calcium levels. B) Perform active range of motion exercises. C) Assess the patient's joint function symmetrically. D) Contact the primary care provider immediately.

D Feedback: This major neurovascular problem is caused by pressure within a muscle compartment that increases to such an extent that microcirculation diminishes, leading to nerve and muscle anoxia and necrosis. Function can be permanently lost if the anoxic situation continues for longer than 6 hours. Therefore, immediate medical care is a priority over further nursing assessment. Assessment of calcium levels is unnecessary.

A nurse is caring for a patient who has had a total hip replacement. The nurse is reviewing health education prior to discharge. Which of the patient's statements would indicate to the nurse that the patient requires further teaching? A) "I'll need to keep several pillows between my legs at night." B) "I need to remember not to cross my legs. It's such a habit." C) "The occupational therapist is showing me how to use a 'sock puller' to help me get dressed." D) "I will need my husband to assist me in getting off the low toilet seat at home."

D Feedback: To prevent hip dislocation after a total hip replacement, the patient must avoid bending the hips beyond 90 degrees. Assistive devices, such as a raised toilet seat, should be used to prevent severe hip flexion. Using an abduction pillow or placing several pillows between the legs reduces the risk of hip dislocation by preventing adduction and internal rotation of the legs. Likewise, teaching the patient to avoid crossing the legs also reduces the risk of hip dislocation. A sock puller helps a patient get dressed without flexing the hips beyond 90 degrees.

A patient recovering from a stroke has severe shoulder pain from subluxation of the shoulder and is being cared for on the unit. To prevent further injury and pain, the nurse caring for this patient is aware of what principle of care? A) The patient should be fitted with a cast because use of a sling should be avoided due to adduction of the affected shoulder. B) Elevation of the arm and hand can lead to further complications associated with edema. C) Passively exercising the affected extremity is avoided in order to minimize pain. D) The patient should be taught to interlace fingers, place palms together, and slowly bring scapulae forward to avoid excessive force to shoulder.

D Feedback: To prevent shoulder pain, the nurse should never lift a patient by the flaccid shoulder or pull on the affected arm or shoulder. The patient is taught how to move and exercise the affected arm/shoulder through proper movement and positioning. The patient is instructed to interlace the fingers, place the palms together, and push the clasped hands slowly forward to bring the scapulae forward; he or she then raises both hands above the head. This is repeated throughout the day. The use of a properly worn sling when the patient is out of bed prevents the paralyzed upper extremity from dangling without support. Range-of-motion exercises are still vitally important in preventing a frozen shoulder and ultimately atrophy of subcutaneous tissues, which can cause more pain. Elevation of the arm and hand is also important in preventing dependent edema of the hand.

Several residents of a long-term care facility have developed signs and symptoms of viral conjunctivitis. What is the most appropriate action of the nurse who oversees care in the facility? A) Arrange for the administration of prophylactic antibiotics to unaffected residents. B) Instill normal saline into the eyes of affected residents two to three times daily. C) Swab the conjunctiva of unaffected residents for culture and sensitivity testing. D) Isolate affected residents from residents who have not developed conjunctivitis.

D Feedback: To prevent spread during outbreaks of conjunctivitis caused by adenovirus, health care facilities must set aside specified areas for treating patients diagnosed with or suspected of having conjunctivitis caused by adenovirus. Antibiotics and saline flushes are ineffective and normally no need to perform testing of individuals lacking symptoms.

A patient with a head injury has been increasingly agitated and the nurse has consequently identified a risk for injury. What is the nurse's best intervention for preventing injury? A) Restrain the patient as ordered. B) Administer opioids PRN as ordered. C) Arrange for friends and family members to sit with the patient. D) Pad the side rails of the patient's bed.

D Feedback: To protect the patient from self-injury, the nurse uses padded side rails. The nurse should avoid restraints, because straining against them can increase ICP or cause other injury. Narcotics used to control restless patients should be avoided because these medications can depress respiration, constrict the pupils, and alter the patient's responsiveness. Visitors should be limited if the patient is agitated.

A patient with a fractured femur is in balanced suspension traction. The patient needs to be repositioned toward the head of the bed. During repositioning, what should the nurse do? A) Place slight additional tension on the traction cords. B) Release the weights and replace them immediately after positioning. C) Reposition the bed instead of repositioning the patient. D) Maintain consistent traction tension while repositioning.

D Feedback: Traction is used to reduce the fracture and must be maintained at all times, including during repositioning. It would be inappropriate to add tension or release the weights. Moving the bed instead of the patient is not feasible.

A nurse is preparing to discharge a patient from the emergency department after receiving treatment for an ankle sprain. While providing discharge education, the nurse should encourage which of the following? A) Apply heat for the first 24 to 48 hours after the injury. B) Maintain the ankle in a dependent position. C) Exercise hourly by performing rotation exercises of the ankle. D) Keep an elastic compression bandage on the ankle.

D Feedback: Treatment of a sprain consists of resting and elevating the affected part, applying cold, and using a compression bandage. After the acute inflammatory stage (usually 24 to 48 hours after injury), heat may be applied intermittently. Rotation exercises would likely be painful.

A nurse is providing a class on osteoporosis at the local seniors' center. Which of the following statements related to osteoporosis is most accurate? A) Osteoporosis is categorized as a disease of the elderly. B) A nonmodifiable risk factor for osteoporosis is a person's level of activity. C) Secondary osteoporosis occurs in women after menopause. D) Slow discontinuation of corticosteroid therapy can halt the progression of the osteoporosis.

D Feedback: When corticosteroid therapy is discontinued, the progression of osteoporosis is halted, but restoration of lost bone mass does not occur. Osteoporosis is not a disease of the elderly because its onset occurs earlier in life, when bone mass peaks and then begins to decline. A person's level of physical activity is a modifiable factor that influences peak bone mass. Lack of activity increases the risk for the development of osteoporosis. Primary osteoporosis occurs in women after menopause.

A nurse assesses a client who is recovering from a diskectomy 6 hours ago. Which assessment finding should the nurse address first? a. Sleepy but arouses to voice b. Dry and cracked oral mucosa c. Pain present in lower back d. Bladder palpated above pubis

D A distended bladder may indicate damage to the sacral spinal nerves. The other findings require the nurse to provide care but are not the priority or a complication of the procedure.

nurse cares for a client with amyotrophic lateral sclerosis (ALS). The client states, "I do not want to be placed on a mechanical ventilator." How should the nurse respond? a. "You should discuss this with your family and health care provider." b. "Why are you afraid of being placed on a breathing machine?" c. "Using the incentive spirometer each hour will delay the need for a ventilator." d. "What would you like to be done if you begin to have difficulty breathing?"

D ALS is an adult-onset upper and lower motor neuron disease characterized by progressive weakness, muscle wasting, and spasticity, eventually leading to paralysis. Once muscles of breathing are involved, the client must indicate in the advance directive what is to be done when breathing is no longer possible without intervention. The other statements do not address the client's needs.

A client with multiple myeloma demonstrates worsening bone density on diagnostic scans. About what drug does the nurse plan to teach this client? a. Bortezomib (Velcade) b. Dexamethasone (Decadron) c. Thalidomide (Thalomid) d. Zoledronic acid (Zometa)

D All the options are drugs used to treat multiple myeloma, but the drug used specifically for bone manifestations is zoledronic acid (Zometa), which is a bisphosphonate. This drug class inhibits bone resorption and is used to treat osteoporosis as well.

A client with sickle cell disease (SCD) takes hydroxyurea (Droxia). The client presents to the clinic reporting an increase in fatigue. What laboratory result should the nurse report immediately? a. Hematocrit: 25% b. Hemoglobin: 9.2 mg/dL c. Potassium: 3.2 mEq/L d. White blood cell count: 38,000/mm3

D Although individuals with SCD often have elevated white blood cell (WBC) counts, this extreme elevation could indicate leukemia, a complication of taking hydroxyurea. The nurse should report this finding immediately. Alternatively, it could indicate infection, a serious problem for clients with SCD. Hematocrit and hemoglobin levels are normally low in people with SCD. The potassium level, while slightly low, is not as worrisome as the WBCs.

A nurse assesses a client's recent memory. Which client statement confirms that the client's remote memory is intact? a. "A young girl wrapped in a shroud fell asleep on a bed of clouds." b. "I was born on April 3, 1967, in Johnstown Community Hospital." c. "Apple, chair, and pencil are the words you just stated." d. "I ate oatmeal with wheat toast and orange juice for breakfast."

D Asking clients about recent events that can be verified, such as what the client ate for breakfast, assesses the client's recent memory. The client's ability to make up a rhyme tests not memory, but rather a higher level of cognition. Asking clients about certain facts from the past that can be verified assesses remote or long-term memory. Asking the client to repeat words assesses the client's immediate memory.

After teaching the wife of a client who has Parkinson disease, the nurse assesses the wife's understanding. Which statement by the client's wife indicates she correctly understands changes associated with this disease? a. "His masklike face makes it difficult to communicate, so I will use a white board." b. "He should not socialize outside of the house due to uncontrollable drooling." c. "This disease is associated with anxiety causing increased perspiration." d. "He may have trouble chewing, so I will offer bite-sized

D Because chewing and swallowing can be problematic, small frequent meals and a supplement are better for meeting the client's nutritional needs. A masklike face and drooling are common in clients with Parkinson disease. The client should be encouraged to continue to socialize and communicate as normally as possible. The wife should understand that the client's masklike face can be misinterpreted and additional time may be needed for the client to communicate with her or others. Excessive perspiration is also common in clients with Parkinson disease and is associated with the autonomic nervous system's response.

After teaching a client who is diagnosed with new-onset status epilepticus and prescribed phenytoin (Dilantin), the nurse assesses the client's understanding. Which statement by the client indicates a correct understanding of the teaching? a. "To prevent complications, I will drink at least 2 liters of water daily." b. "This medication will stop me from getting an aura before a seizure." c. "I will not drive a motor vehicle while taking this medication." d. "Even when my seizures stop, I will continue to take this drug."

D Discontinuing antiepileptic drugs can lead to the recurrence of seizures or status epilepticus. The client does not need to drink more water and can drive while taking this medication. The medication will not stop an aura before a seizure.

A nursing student is struggling to understand the process of graft-versus-host disease. What explanation by the nurse instructor is best? a. "Because of immunosuppression, the donor cells take over." b. "It's like a transfusion reaction because no perfect matches exist." c. "The client's cells are fighting donor cells for dominance." d. "The donor's cells are actually attacking the client's cells."

D Graft versus host disease is an autoimmune-type process in which the donor cells recognize the client's cells as foreign and begin attacking them. The other answers are not accurate.

A nurse assesses a client with a neurologic disorder. Which assessment finding should the nurse identify as a late manifestation of amyotrophic lateral sclerosis (ALS)? a. Dysarthria b. Dysphagia c. Muscle weakness d. Impairment of respiratory muscles

D In ALS, progressive muscle atrophy occurs until a flaccid quadriplegia develops. Eventually, the respiratory muscles are involved, which leads to respiratory compromise. Dysarthria, dysphagia, and muscle weakness are early clinical manifestations of ALS.

A nurse teaches a client with a lower motor neuron lesion who wants to achieve bladder control. Which statement should the nurse include in this client's teaching? a. "Stroke the inner aspect of your thigh to initiate voiding." b. "Use a clean technique for intermittent catheterization." c. "Implement digital anal stimulation when your bladder is full." d. "Tighten your abdominal muscles to stimulate urine flow."

D In clients with lower motor neuron problems such as spinal cord injury, performing a Valsalva maneuver or tightening the abdominal muscles are interventions that can initiate voiding. Stroking the inner aspect of the thigh may initiate voiding in a client who has an upper motor neuron problem. Intermittent catheterization and digital anal stimulation do not initiate voiding or bladder control.

A nursing student is caring for a client with leukemia. The student asks why the client is still at risk for infection when the client's white blood cell count (WBC) is high. What response by the registered nurse is best? a. "If the WBCs are high, there already is an infection present." b. "The client is in a blast crisis and has too many WBCs." c. "There must be a mistake; the WBCs should be very low." d. "Those WBCs are abnormal and don't provide protection."

D In leukemia, the WBCs are abnormal and do not provide protection to the client against infection. The other statements are not accurate.

A nurse cares for a client who is experiencing deteriorating neurologic functions. The client states, "I am worried I will not be able to care for my young children." How should the nurse respond? a. "Caring for your children is a priority. You may not want to ask for help, but you have to." b. "Our community has resources that may help you with some household tasks so you have energy to care for your children." c. "You seem distressed. Would you like to talk to a psychologist about adjusting to your changing status?" d. "Give me more information about what worries you, so we can see if we can do something to make adjustments."

D Investigate specific concerns about situational or role changes before providing additional information. The nurse should not tell the client what is or is not a priority for him or her. Although community resources may be available, they may not be appropriate for the client. Consulting a psychologist would not be appropriate without obtaining further information from the client related to current concerns.

A nurse obtains a focused health history for a client who is scheduled for magnetic resonance imaging (MRI). Which condition should alert the nurse to contact the provider and cancel the procedure? a. Creatine phosphokinase (CPK) of 100 IU/L b. Atrioventricular graft c. Blood urea nitrogen (BUN) of 50 mg/dL d. Internal insulin pump

D Metal devices such as internal pumps, pacemakers, and prostheses interfere with the accuracy of the image and can become displaced by the magnetic force generated by an MRI procedure. An atrioventricular graft does not contain any metal. CPK and BUN levels have no impact on an MRI procedure.

A nurse prepares a client for prescribed magnetic resonance imaging (MRI). Which action should the nurse implement prior to the test? a. Implement nothing by mouth (NPO) status for 8 hours. b. Withhold all daily medications until after the examination. c. Administer morphine sulfate to prevent claustrophobia during the test. d. Place the client in a gown that has cloth ties instead of metal snaps.

D Metal objects are a hazard because of the magnetic field used in the MRI procedure. Morphine sulfate is not administered to prevent claustrophobia; lorazepam (Ativan) or diazepam (Valium) may be used instead. The client does not need to be NPO, and daily medications do not need to be withheld prior to MRI.

A nurse cares for a client who presents with an acute exacerbation of multiple sclerosis (MS). Which prescribed medication should the nurse prepare to administer? a. Baclofen (Lioresal) b. Interferon beta-1b (Betaseron) c. Dantrolene sodium (Dantrium) d. Methylprednisolone (Medrol)

D Methylprednisolone is the drug of choice for acute exacerbations of the disease. The other drugs are not used to treat acute exacerbations of MS. Interferon beta-1b is used to treat and control MS, decrease specific symptoms, and slow the progression of the disease. Baclofen and dantrolene sodium are prescribed to lessen muscle spasticity associated with MS.

An emergency room nurse initiates care for a client with a cervical spinal cord injury who arrives via emergency medical services. Which action should the nurse take first? a. Assess level of consciousness. b. Obtain vital signs. c. Administer oxygen therapy. d. Evaluate respiratory status.

D The first priority for a client with a spinal cord injury is assessment of respiratory status and airway patency. Clients with cervical spine injuries are particularly prone to respiratory compromise and may even require intubation. The other assessments should be performed after airway and breathing are assessed.

After teaching a client newly diagnosed with epilepsy, the nurse assesses the client's understanding. Which statement by the client indicates a need for additional teaching? a. "I will wear my medical alert bracelet at all times." b. "While taking my epilepsy medications, I will not drink any alcoholic beverages." c. "I will tell my doctor about my prescription and over-the-counter medications." d. "If I am nauseated, I will not take my epilepsy medication

D The nurse must emphasize that antiepileptic drugs must be taken even if the client is nauseous. Discontinuing the medication can predispose the client to seizure activity and status epilepticus. The client should not drink alcohol while taking seizure medications. The client should wear a medical alert bracelet and should make the doctor aware of all medications to prevent complications of polypharmacy.

A nurse cares for a client who has been diagnosed with the Huntington gene but has no symptoms. The client asks for options related to family planning. What is the nurse's best response? a. "Most clients with the Huntington gene do not pass on Huntington disease to their children." b. "I understand that they can diagnose this disease in embryos. Therefore, you could select a healthy embryo from your fertilized eggs for implantation to avoid passing on Huntington disease." c. "The need for family planning is limited because one of the hallmarks of Huntington disease is infertility." d. "Tell me more specifically what information you need about family planning so that I can direct you to the right information or health care provider."

D The presence of the Huntington gene means that the trait will be passed on to all offspring of the affected person. Understanding options for contraception and conception (e.g., surrogacy options) and implications for children may require the expertise of a genetic counselor or a reproductive specialist. The other statements are not accurate.

The nurse assesses a client's oral cavity and makes the discovery shown in the photo below: What action by the nurse is most appropriate? a. Encourage the client to have genetic testing. b. Instruct the client on high-fiber foods. c. Place the client in protective precautions. d. Teach the client about cobalamin therapy.

D This condition is known as glossitis, and is characteristic of B12 anemia. If the anemia is a pernicious anemia, it is treated with cobalamin. Genetic testing is not a priority for this condition. The client does not need high-fiber foods or protective precautions.

A client with autoimmune idiopathic thrombocytopenic purpura (ITP) has had a splenectomy and returned to the surgical unit 2 hours ago. The nurse assesses the client and finds the abdominal dressing saturated with blood. What action is most important? a. Preparing to administer a blood transfusion b. Reinforcing the dressing and documenting findings c. Removing the dressing and assessing the surgical site d. Taking a set of vital signs and notifying the surgeon

D While some bloody drainage on a new surgical dressing is expected, a saturated dressing is not. This client is already at high risk of bleeding due to the ITP. The nurse should assess vital signs for shock and notify the surgeon immediately. The client may or may not need a transfusion. Reinforcing the dressing is an appropriate action, but the nurse needs to do more than document afterward. Removing the dressing increases the risk of infection; plus, it is not needed since the nurse knows where the bleeding is coming from.


Conjuntos de estudio relacionados

To what extent does technology have a negative effect on cognitive processes?

View Set

ch 19: postop nursing management

View Set

Chapter 1 - The Essentials of Human Communication

View Set